Sumario´mcabral/livros/livro-calculo/cursoCalculoI... · Curso de Cálculo Uma riável a V...

89

Transcript of Sumario´mcabral/livros/livro-calculo/cursoCalculoI... · Curso de Cálculo Uma riável a V...

Curso de Cál ulo de Uma Variável(Somente a Lista de Exer í ios)Primeira Edição V1.0Julho de 2010Mar o A. P. CabralPhD Indiana University EUAProfessor do Instituto de Matemáti aUniversidade Federal do Rio de Janeiro

Departamento de Matemáti a Apli adaInstituto de Matemáti aUniversidade Federal do Rio de JaneiroRio de Janeiro - BrasilCópias são autorizadas e bem vindas: divulgue nosso trabalho! Consulte o sítiowww.labma.ufrj.br/~m abral/livros ou entre em ontato om o autor emmap abral(at)ufrj(dot)br.

ii

Sumario

1 Enun iados dos Exer í ios 11.1 Exer í ios de Limite . . . . . . . . . . . . . . . . . . . . . . . . . . . . . . 11.1.1 Exer í ios de Fixação . . . . . . . . . . . . . . . . . . . . . . . . . 11.1.2 Problemas . . . . . . . . . . . . . . . . . . . . . . . . . . . . . . . 41.1.3 Extras . . . . . . . . . . . . . . . . . . . . . . . . . . . . . . . . . 61.1.4 Desaos . . . . . . . . . . . . . . . . . . . . . . . . . . . . . . . . 71.2 Exer í ios de Continuidade . . . . . . . . . . . . . . . . . . . . . . . . . . . 81.2.1 Exer í ios de Fixação . . . . . . . . . . . . . . . . . . . . . . . . . 81.2.2 Problemas . . . . . . . . . . . . . . . . . . . . . . . . . . . . . . . 91.2.3 Extras . . . . . . . . . . . . . . . . . . . . . . . . . . . . . . . . . 101.2.4 Desaos . . . . . . . . . . . . . . . . . . . . . . . . . . . . . . . . 111.3 Exer í ios de Derivada . . . . . . . . . . . . . . . . . . . . . . . . . . . . 121.3.1 Exer í ios de Fixação . . . . . . . . . . . . . . . . . . . . . . . . . 121.3.2 Problemas . . . . . . . . . . . . . . . . . . . . . . . . . . . . . . . 151.3.3 Extras . . . . . . . . . . . . . . . . . . . . . . . . . . . . . . . . . 161.3.4 Desaos . . . . . . . . . . . . . . . . . . . . . . . . . . . . . . . . 181.4 Exer í ios de Apli ação de Derivada . . . . . . . . . . . . . . . . . . . . . 181.4.1 Exer í ios de Fixação . . . . . . . . . . . . . . . . . . . . . . . . . 181.4.2 Problemas . . . . . . . . . . . . . . . . . . . . . . . . . . . . . . . 211.4.3 Extras . . . . . . . . . . . . . . . . . . . . . . . . . . . . . . . . . 251.4.4 Desaos . . . . . . . . . . . . . . . . . . . . . . . . . . . . . . . . 291.4.5 ⋆Problemas (Taxas Rela ionadas) . . . . . . . . . . . . . . . . . . . . 301.4.6 ⋆Problemas (Derivação Implí ita) . . . . . . . . . . . . . . . . . . . . 311.5 Exer í ios de Integral . . . . . . . . . . . . . . . . . . . . . . . . . . . . . 321.5.1 Exer í ios de Fixação . . . . . . . . . . . . . . . . . . . . . . . . . 321.5.2 Problemas . . . . . . . . . . . . . . . . . . . . . . . . . . . . . . . 341.5.3 Extras . . . . . . . . . . . . . . . . . . . . . . . . . . . . . . . . . 351.5.4 Desaos . . . . . . . . . . . . . . . . . . . . . . . . . . . . . . . . 371.5.5 ⋆Problemas (Integração por Frações Par iais) . . . . . . . . . . . . . . 371.6 Exer í ios de Apli ações da Integral . . . . . . . . . . . . . . . . . . . . . . 371.6.1 Exer í ios de Fixação . . . . . . . . . . . . . . . . . . . . . . . . . 371.6.2 Problemas . . . . . . . . . . . . . . . . . . . . . . . . . . . . . . . 391.6.3 Extras . . . . . . . . . . . . . . . . . . . . . . . . . . . . . . . . . 401.6.4 Desaos . . . . . . . . . . . . . . . . . . . . . . . . . . . . . . . . 421.6.5 ⋆Problemas (Substituição Trigonométri a) . . . . . . . . . . . . . . . 431.6.6 ⋆Problemas (Comprimento de Curvas no Plano) . . . . . . . . . . . . . 431.6.7 ⋆Problemas (Área de Superfí ie de Sólido de Revolução) . . . . . . . . 43iii

iv SUMÁRIO2 Respostas dos Exer í ios 452.1 Limite . . . . . . . . . . . . . . . . . . . . . . . . . . . . . . . . . . . . . 452.1.1 Exer í ios de Fixação . . . . . . . . . . . . . . . . . . . . . . . . . 452.1.2 Problemas . . . . . . . . . . . . . . . . . . . . . . . . . . . . . . . 482.1.3 Extras . . . . . . . . . . . . . . . . . . . . . . . . . . . . . . . . . 502.1.4 Desaos . . . . . . . . . . . . . . . . . . . . . . . . . . . . . . . . 512.2 Continuidade . . . . . . . . . . . . . . . . . . . . . . . . . . . . . . . . . . 522.2.1 Exer í ios de Fixação . . . . . . . . . . . . . . . . . . . . . . . . . 522.2.2 Problemas . . . . . . . . . . . . . . . . . . . . . . . . . . . . . . . 532.2.3 Extras . . . . . . . . . . . . . . . . . . . . . . . . . . . . . . . . . 542.2.4 Desaos . . . . . . . . . . . . . . . . . . . . . . . . . . . . . . . . 542.3 Derivada . . . . . . . . . . . . . . . . . . . . . . . . . . . . . . . . . . . . 542.3.1 Exer í ios de Fixação . . . . . . . . . . . . . . . . . . . . . . . . . 542.3.2 Problemas . . . . . . . . . . . . . . . . . . . . . . . . . . . . . . . 552.3.3 Extras . . . . . . . . . . . . . . . . . . . . . . . . . . . . . . . . . 572.3.4 Desaos . . . . . . . . . . . . . . . . . . . . . . . . . . . . . . . . 582.4 Apli ação de Derivada . . . . . . . . . . . . . . . . . . . . . . . . . . . . 592.4.1 Exer í ios de Fixação . . . . . . . . . . . . . . . . . . . . . . . . . 592.4.2 Problemas . . . . . . . . . . . . . . . . . . . . . . . . . . . . . . . 602.4.3 Extras . . . . . . . . . . . . . . . . . . . . . . . . . . . . . . . . . 662.4.4 Desaos . . . . . . . . . . . . . . . . . . . . . . . . . . . . . . . . 722.4.5 ⋆Problemas (Taxas Rela ionadas) . . . . . . . . . . . . . . . . . . . . 732.4.6 ⋆Problemas (Derivação Implí ita) . . . . . . . . . . . . . . . . . . . . 742.5 Integral . . . . . . . . . . . . . . . . . . . . . . . . . . . . . . . . . . . . . 742.5.1 Exer í ios de Fixação . . . . . . . . . . . . . . . . . . . . . . . . . 742.5.2 Problemas . . . . . . . . . . . . . . . . . . . . . . . . . . . . . . . 762.5.3 Extras . . . . . . . . . . . . . . . . . . . . . . . . . . . . . . . . . 782.5.4 Desaos . . . . . . . . . . . . . . . . . . . . . . . . . . . . . . . . 792.5.5 ⋆Problemas (Integração por Frações Par iais) . . . . . . . . . . . . . . 792.6 Apli ações da Integral . . . . . . . . . . . . . . . . . . . . . . . . . . . . . 792.6.1 Exer í ios de Fixação . . . . . . . . . . . . . . . . . . . . . . . . . 792.6.2 Problemas . . . . . . . . . . . . . . . . . . . . . . . . . . . . . . . 802.6.3 Extras . . . . . . . . . . . . . . . . . . . . . . . . . . . . . . . . . 822.6.4 Desaos . . . . . . . . . . . . . . . . . . . . . . . . . . . . . . . . 842.6.5 ⋆Problemas (Substituição Trigonométri a) . . . . . . . . . . . . . . . 852.6.6 ⋆Problemas (Comprimento de Curvas no Plano) . . . . . . . . . . . . . 852.6.7 ⋆Problemas (Área de Superfí ie de Sólido de Revolução) . . . . . . . . 85

Capıtulo 1

Enunciados dos Exercıcios

1.1 Exer í ios de Limite1.1.1 Exer í ios de FixaçãoExer í io 1.Considere o grá o de y = f(x) esboçada no grá o abaixo. Determine oslimites abaixo. Caso algum não exista, determine os limites laterais.(a) limx→a

f(x); (b) limx→b

f(x); ( ) limx→c

f(x).x

y

a b c

5

6

3

1

Exer í io 2.Determine se é Verdadeiro ou Falso. Se for falso dê um ontraexemplo ou orrija. Se for verdadeiro justique.(a) x ∈ R; |x− 3| ≤ 2 = [1, 5].(b) x ∈ R; |x+ 2| < 1 = (1, 3).( ) √x2 = x para todo x ∈ R.(d) se g(x) =

4; x 6= 2;

π; x = 2, então lim

x→2g(x) = g(2) = π.Exer í io 3.Determine se é Verdadeiro ou Falso. Se for falso dê um ontraexemplo ou orrija. Se for verdadeiro justique.(a) Se lim

x→3+f(x) = 5, então e lim

x→3f(x) = 5.(b) Se lim

x→2f(x) = 4, então e lim

x→2−f(x) = −4.022.jul.2010 1

2 CAPÍTULO 1. ENUNCIADOS DOS EXERCÍCIOS( ) Se limx→2

f(x) = 4, então f(2) = 4.(d) Existe uma função f tal que limx→3+

f(x) 6= limx→3−

f(x) = limx→3

f(x).Exer í io 4.Considere a função f dada por f(x) =

5; x ≤ 1

7; 1 < x ≤ 2

9; x > 2

. Determine limx→k

f(x)ou, aso não exista, os limites laterais para:(a) k = 1; (b) k = 0.9999; ( ) k = 1.0001;(d) k = 2; (e) k = 1.9999; (f) k = 2.0001.Exer í io 5.Aplique a denição do módulo para esboçar o o grá o de:(a) cosx

| cos(x)| ; (b) √|x|.Exer í io 6.Partindo de grá o de funções simples (±x2, ±1/x, ±1/x2, √x, sen(x), |x|,log(x), ex), utilizando translações verti ais e/ou horizontais e/ou reexões, esbo e o grá ode: (a) y = 1 +

√x (b) y = 2 + sen(x); ( ) y = log(x− 1) + 2;(d) y =

−1

(x+ 2)3; (e) y = |(x+ 1)(x+ 2)|; (f) y = |ex − 2|.Exer í io 7.Determine os limites:(a) lim

x→2

x− 2

(2− x)(3− x); (b) lim

x→0

x4 + x

x3 + 2x; ( ) lim

x→3

x− 3

x2 − 4.Exer í io 8.Dena lim

x→+∞f(x) = +∞ seguindo mutatis mutandis1 as denições dadas notexto.Exer í io 9. Faça o estudo de sinal do numerador e denominador para determinar os valoresde x que satisfazem as desigualdades:(a) 3− x2

x2 − 1≥ 0; (b) x3 − 1

x(x2 − 4)≤ 0.Exer í io 10. Faça o estudo de sinal e o esboço do grá o dos polinmios abaixo.(a) p(x) = (x− 2)(x+ 3)(1− x); (b) q(x) = (x− 2)2(x+ 1);( ) r(x) = (3− x)(x− 2)2(x− 5).Exer í io 11.Determine os limites:(a) lim

x→0−

1

x; (b) lim

x→0−

1

x2; ( ) lim

x→0−

x

|x| ; (d) limx→0

x3

|x| ;(e) limx→2

x2 + 1

x− 2; (f) lim

x→0−

(

x+1

x

)

; (g) limx→3+

x

x2 − 9.1latim para modique o que tem que ser modi ado

1.1. EXERCÍCIOS DE LIMITE 3Exer í io 12.Determine os limites:(a) limx→+∞

x√x2 + 1

; (b) limx→+∞

(

x+1

x

); ( ) limx→+∞

1 + 6x

x− 2;(d) lim

x→−∞

2x− x2

3x+ 5; (e) lim

x→+∞

2x3 − 4

5x+ 3; (f) lim

x→−∞

7x3 − 15x2

13x;(g) lim

x→−∞

3x5 + x− 1

x5 − 7;(h) lim

x→+∞

3x3 + 2x4 + 5x5 − 1

4x5 − 3x4 − 2x2 + x+ 3; (i) lim

x→+∞

5x10 − 3x7 + 9x6 − 12x2 − x+ 1

x9 − 7x2 − 21.Exer í io 13.Determine se é Verdadeiro ou Falso. Se for falso dê um ontraexemplo ou orrija. Se for verdadeiro justique. Se lim

x→1q(x) = 0, então(a) lim

x→1

3

q(x)= +∞; (b) lim

x→1

q(x)

f(x)= 0; ( ) lim

x→1

q(x)

x2 − 1= 0.Exer í io 14.Qual das Figuras abaixo pode representar o grá o de uma função g tal que:(i) lim

x→∞g(x) = 1 (ii) lim

x→−∞g(x) = −1(iii) lim

x→1+g(x) = +∞ (iv) lim

x→1−g(x) = −∞.

-1

1

1 x

y

-1

1

1 x

y

-1

1

1 x

y

-1

1

1 x

y

-1

1

1 x

y

-1 1 x

y

-1 1 x

y

-1 1 x

y

-1

1

1 x

y

-1

1

1 x

y

-1

1

1 x

y

-1

1

1 x

y

-1

1

1 x

y

-1

1

1 x

y

-1

1

1 x

y

-1

1

1 x

y

-1

1

1 x

y

-1

1

1 x

y

(a) (b) ( ) (d)Exer í io 15. Faça um esboço de um grá o de uma função f tal que limx→1−

f(x) = 2,

f(1) = 1 e, além disso (um grá o para ada item):(a) limx→1+

f(x) = −2, (b) limx→1+

f(x) não exista, ( ) limx→1+

f(x) = +∞,Exer í io 16.Determine os limites:(a) limx→0

|x| sen(1/x); (b) limh→0

sen(3x)

x; ( ) lim

x→+∞(1 + 1/x)5x;(d) lim

x→π/2+tan(x); (e) lim

x→0+(1− 2x)1/x.Exer í io 17.Estude o Teorema 4 da p.33 (Sanduí he) e responda:(a) É verdade que se 1 ≤ g(x) ≤ 2 então lim

x→3/2g(x) existe e é um número entre 1 e 2?(b) Explique, utilizando o Teorema do Sanduí he, omo al ular lim

x→+∞

cos(√x2 + 1)

x2.

4 CAPÍTULO 1. ENUNCIADOS DOS EXERCÍCIOS1.1.2 ProblemasProblema 1.Esbo e o grá o das seguintes funções:(a) f(x) = −√9− x2; |x| ≤ 3

|x| − 3; |x| > 3.(b) f(x) = √x− 1; x ≥ 1;

log(x) + 1; x < 1.Problema 2.Considere a função IZ ( hamada de função ara terísti a ou indi adora do onjunto Z) denida por IZ(x) = 0; x 6∈ Z

1; x ∈ Z.Esbo e o grá o e determine (se existir):(a) lim

x→3/4IZ(x); (b) lim

x→−3IZ(x); ( ) lim

x→+∞IZ(x).Problema 3.Cal ule os limites abaixo (quando eles existirem) justi ando seus passos (semutilizar a regra de L'Hospital) Limites om raízes:(a) lim

h→0

√1 + h−

√1− h

h(b) lim

x→4

|x| − 4√x− 2

; ( ) limh→−1

√h2 + 3− 2

h + 1;Problema 4.Determine os limites e, aso não exista, os limites laterais ( aso existam).(a) lim

x→−3sen

(

7

x+ 3

); (b) limx→2

log |x− 2|;( ) limx→2

|x− 2|(x+ 1)

x− 2; (d) lim

x→−5

x+ 3

x+ 5.Problema 5.Cal ule os limites abaixo (quando eles existirem) justi ando seus passos (semutilizar a regra de L'Hospital):(a) lim

x→2−

x

x2 − 4; (b) lim

x→1+

x+ 3

1− x( ) lim

x→0

(

1

x− 1

x2

)

; (d) limx→2−

|x− 2|x2 − 5x+ 6

;(e) limx→−2

x+ 2

|x| − 2; (f) lim

a→2

(a− 2)(a2 − 4)

a3 − 5a2 + 8a− 4; (g) lim

x→2

x2 − 3x+ 2

x2 − 3x+ 5;(h) lim

x→1

x3 − x

x2 − 3x+ 2; (i) lim

x→2

x2 + 3x− 1

x2 + 2x− 1; (j) lim

x→1

x+ 1− 2x

x− 1;(k) lim

x→−1

x2 + 2x+ 1

x+ 1(l) lim

x→−1

x3 + 1

x+ 1; (m) lim

x→1

2x2 − 3x+ 1

x− 1;Problema 6.Cal ule os limites abaixo (quando eles existirem) justi ando seus passos (semutilizar a regra de L'Hospital) Limites no innito:(a) lim

x→−∞

√x2 + 1

x+ 1; (b) lim

y→+∞

7− 2y√

5− 2y + 9y2; ( ) lim

x→+∞

√10x4 + 3x3 + 2x+ 5

5x2 − 10x− 100;(d) lim

x→+∞

√x2 + 1√x+ 1

; (e) limy→−∞

5− 3y3√

8− y + 10y4; (f) lim

x→−∞sen

(√16x6 − x+ 1

2x3 − x2 + 20

).Problema 7.Considere a, b ∈ R e c > 0. Determine os limites:(a) limx→0

(1+ax)b/x; (b) limx→−∞

(√cx2 + a−

√cx2 + b

); ( ) limx→+∞

(√cx2 + ax− bx

);

1.1. EXERCÍCIOS DE LIMITE 5(d) limx→+∞

(√cx2 + ax−

√cx2 + bx

); (e) limx→−∞

(√cx2 + ax−

√cx2 + bx

).Problema 8.Considere os polinmiosp(x) = axm + x2 − 3x + 1, q(x) = bxm + 2x5 − 4, r(x) = cx2m + 3x7 + 2 om m > 10,a, b 6= 0 e c > 0. Determine os limites:(a) lim

x→+∞

p(x)

r(x)(b) lim

x→+∞

q(x)

p(x)( ) lim

x→+∞

r(x)

x2p(x)(d) limx→+∞

xmp(x)

r(x)(e) lim

x→+∞

r(x)

p(x)(f) lim

x→+∞

r(x)

xq(x)Problema 9.Determine os limites laterais quando x → 0 para:(a) h(x) = 1

1 + e1/x; (b) h(x) = 1

x− 1

|x| .Problema 10. Sabendo que o quadro de sinais de f(x) é dado pela tabela abaixo e quelim

x→+∞f(x) = 4 e lim

x→−∞f(x) = +∞, esbo e o grá o de f(x) e determine TODAS asassíntotas verti ais e horizontais.

−3 −2 3 40 ±∞ 0 ±∞

f(x) + − − − +Problema 11.Esbo e o grá o de ada uma das funções abaixo seguindo o roteiro abaixo.(i) Faça um estudo do sinal da função (onde ela é zero, positiva e negativa).(ii) Determine assíntotas horizontais e verti ais.(iii) Baseado em (i) e (ii) esbo e o grá o.(a) y =x2 − 1

x− 1; (b) y =

1

x2 − 1; ( ) y =

x

x2 + 1(d) y =x2 − 1

x(x− 2); (e) y =

3x2 − 3

4− x2;Problema 12.Considere a função h(x) =

x; x ∈ Q

−x; x 6∈ Q.Esbo e o grá o e determine (seexistir):(a) lim

x→πh(x); (b) lim

x→1h(x); ( ) lim

x→0+

h(x)

x2; (d) lim

x→+∞

h(x)

x2; (e) lim

x→0

h(x)

x.Problema 13. (a) Suponha que h(x) satisfaz √

x

x3 + x≤ h(x) ≤ x

x2 + 1. Determine lim

x→+∞h(x).(b) Suponha que f(x) satisfaz |f(x)− 3| ≤ 2|x− 5|4. Cal ule lim

x→5f(x).Problema 14.Cal ule os limites abaixo (quando eles existirem) justi ando seus passos (semutilizar a regra de L'Hospital): Limites trigonométri os e exponen iais.(a) lim

x→0

(tan(3x))2 + sen(11x2)

x sen(5x); (b) lim

x→+∞3x2sen

(

1

x2

); ( ) limx→0

cosx− cos3 x

3x2;

6 CAPÍTULO 1. ENUNCIADOS DOS EXERCÍCIOS(d) limh→0+

sen(√h) tan(2

√h)

5h; (e) lim

x→1sen

(

7x+ 1

sen(πx/2)− 1

)

(ex−1 − 1);(f) limh→0+

(1− 5h3)2/h3 ; (g) lim

x→π

cosx

x− π; (h) lim

x→0

sen x

|x| .1.1.3 ExtrasExtra 1.Partindo de grá o de funções simples (±x2,±1/x,±1/x2,

√x, sen(x), |x|), utili-zando translações verti ais e/ou horizontais e/ou reexões, esbo e o grá o de:(a) y = | sen(x)| − 1; (b) y = ||x| − 1|; ( ) y = |x+ 2| − 1.Extra 2. Faça um esboço de um grá o de uma função f tal que, simultaneamente:

limx→−∞

f(x) = 4, limx→+∞

f(x) = −∞, limx→1−

f(x) = −∞, f(1) = 1, limx→1+

f(x) = −2.Extra 3.Determine limx→0

sengr(x)

x, onde sengr é a função seno do ângulo x medido em graus.Note que para a função seno utilizada em ál ulo, o ângulo é medido em radianos.Extra 4.Esbo e o grá o de:(a) y = x+ |x|; (b) x− ⌊x⌋.Extra 5.Determine os limites:(a) lim

x→1

|x| − 1

|x− 1| ; (b) limx→1

x3 + 1

(x− 1)2; ( ) lim

x→2

x2 + 2x

x3 − x; (d) lim

x→πcos

(

1

x− π

)

(x−π).Extra 6.Determine os limites:(a) limx→+∞

(√x2 + x− x2

); (b) limx→+∞

2x+ |x|x+ 1

; ( ) limx→−∞

2x+ |x|x+ 1

;(d) limx→−∞

x+ 1

x+ |x|+ 1.Extra 7.Considere a ∈ R. Determine os limites:(a) lim

x→+∞

(√x+ a−

√x); (b) lim

x→+∞

(√x2 + a− x

).Extra 8.Esbo e o grá o das seguintes funções:(a) f(x) = 1; x ∈ Q;

2; x 6∈ Q; (b) g(x) = x; x ∈ Q;

x2; x 6∈ Q;

1.1. EXERCÍCIOS DE LIMITE 71.1.4 DesaosDesao 1.A função parte inteira de x, denotada por ⌊x⌋ é denida na p. 12.(a) Cal ule, se existir: limx→∞

x

1

x

. (b) Cal ule, se existir: limx→−∞

x

1

x

.( ) Esbo e o grá o de f(x) = x

1

x

⌋. (d) Cal ule, se existir: limx→0

x

1

x

.Desao 2.Determine:(a) limx→+∞

(ex + x)1/x. (b) limx→+∞

(1 + x)α/ logx, om α 6= 0.Desao 3. Como al ular assíntotas oblíquas e generalizações?Dividindo os polinmios e separando em quo iente e resto.Assim, x2 − 3x+ 2

x− 1= q(x) +

r

x− 1. Para x grande, x2 − 3x+ 2

x− 1≈ q(x), sua assíntotaoblíqua. Plote uns grá os para ver omo de fato se pare em. O mesmo o orre quando adiferença entre os graus do numerador e denominador é maior que 1.Desao 4.Determine lim

x→0+

1

x sen(1/x). Tente esboçar o grá o perto do zero desta função.Utilize algum software para isso.Desao 5. (Cari atura de sen(1/x) do livro do Spivak de Cál ulo) Esbo e o grá o da função

f que satisfaz:(i) f(1/n) = (−1)(n+1),(ii) f é linear entre [1/(n+ 1), 1/n] (segmento de reta),(iii) f(x) = 1 para x > 1,(iv) f(−x) = f(x).Desao 6.Prove que a área do ír ulo de raio r é πr2 seguindo o seguinte roteiro:(a) Mostre que a área do polígono de n-lados ins rito no ír ulo é n

2r2 sen(2π/n).(b) Mostre que a área do polígono de n-lados ir uns rito no ír ulo é n

2r2 tan(π/n).( ) Faça n → +∞ e on lua o argumento.Desao 7. Sejam f e g duas funções tais que |f(x)| ≤ M para todo x ∈ R e lim

x→1g(x) = 0.Mostre que

limx→1

f(x)g(x) = 0.

Desao 8.Objetivo desta atividade é aproximar a função fatorial. É fá il ver que ()

n! =

(

1

2

)(

2

3

)2(3

4

)3(4

5

)4

· · ·(

n− 1

n

)n−1

nn.

8 CAPÍTULO 1. ENUNCIADOS DOS EXERCÍCIOSLogo n! = nn

n−1∏

j=1

(

j

j + 1

)j

= nn/

n−1∏

j=1

(1 + 1/j)j. Já sabemos que o termo (1 + 1/j)jtende para e quando j tende para innito. Portanto n! ≈ nn/en−1 = e(n/e)n (vide [Fe).Utilizando esta aproximação, determine os limites, quando n vai para innito, de:(a) n!

n; (b) n!

n5; ( ) n!

en; (d) n!

nn/2; (e) n!

nn.Obs: Podemos denir fatorial de não-inteiros (e até mesmo de omplexos) om a funçãogama de Euler.Obs: Utilizando outro aminho (vide [C p.361364 ou [Sp p.483) obtemos a fórmulade Stirling2: n! = √

2πn(n/e)neθn om |θ| ≤ 1/12.1.2 Exer í ios de Continuidade1.2.1 Exer í ios de FixaçãoExer í io 1.Determine se é Verdadeiro (provando a armativa) ou Falso (dando um ontra-exemplo):(a) Se lim

x→af(x) existe, então f é ontínua em a;(b) Se f é ontínua em a, então lim

x→a−f(x) existe.( ) Se f é des ontínua em a, então lim

x→a−f(x) 6= lim

x→a+f(x).Exer í io 2.Determine se f esboçada no grá o abaixo é ontínua ou não nos pontos

a, b, c, d. Explique, aso não seja ontínua, qual (quais) ondições são violadas.x

y

a b c dExer í io 3.Verique se as funções abaixo são ontínuas:(a) f(x) = x; x < 0;

0; x ≥ 0;(b) f(x) = x; x < 0;

1; x ≥ 0;Exer í io 4.Esbo e o grá o de uma função ontínua ujos pontos de des ontinuidade(úni os pontos onde a função não é ontínua) são:(a) 1, 2, 3; (b) N = 1, 2, . . ..2James Stirling: ⋆ 05/1692, Garden, Es ó ia † 05/12/1770, Edinburgh, Es ó ia.227.mar.2010

1.2. EXERCÍCIOS DE CONTINUIDADE 9Exer í io 5.Determine um valor para k ∈ R, se for possível, de modo que a função seja ontínua em R.(a) f(x) =

1

x; x 6= 0;

k; x = 0;(b) f(x) =

1

x2; x 6= 0;

k; x = 0;( ) f(x) =

x sen(

1x

)

; x 6= 0;

k; x = 0;Exer í io 6. Seja f uma função ontínua em [1, 4] tal que f(1) = 2, f(2) = 3, f(3) =−1 e f(4) = 2. Determine se é Verdadeiro (provando a armativa) ou Falso (dando um ontraexemplo):(a) f não tem raiz em [1, 2]; (b) f tem pelo menos duas raízes em [1, 4];( ) f tem exatamente uma raiz em [2, 3].Exer í io 7.Determine se é Verdadeiro (provando a armativa) ou Falso (dando um ontra-exemplo):(a) a função que representa o número de habitantes de uma idade em função do tempoé ontínua em todos os pontos;(b) a função que representa a altura de uma pessoa em função do tempo é ontínua emtodos os pontos;Exer í io 8.Estude o Teorema 10 da p.53 (TVI) e determine se é Verdadeiro (provando aarmativa) ou Falso (dando um ontraexemplo):(a) se f é ontínua om f(0) > 0 e f(1) > 0, então f(x) > 0 para todo x ∈ [0, 1];(b) Se f(1) < 0 < f(2), então f possui raiz em [0, 1].Exer í io 9.Estude o Teorema 10 da p.53 (TVI). Considere f : [−3,−1] → R ontínua omf(−3) = 5 e f(−1) = 2. Determine se esta orreto ou orrija.(a) se K ∈ [−3,−1], então existe c ∈ [2, 5] tal que f(c) = K;(b) se K ∈ [3, 4], então existe c ∈ [−3,−1] tal que f(c) = K;( ) se K ∈ [0, 3], então existe c ∈ [−3,−1] tal que f(c) = K;Exer í io 10.Estude o Lema 2 da p.51 e o Teorema 7 da p.52. Supondo que f é ontínua,prove, fazendo referên ia somente ao Lema 2 e o Teorema 7, que h(x) = 5[f(x)]3

x2 + 1é ontínua.

1.2.2 ProblemasProblema 1.Determine o onjunto dos pontos de des ontinuidade (úni os pontos onde afunção não é ontínua) de:(a) f(x) =

1

sen(x); x 6= kπ; k ∈ Z

1; x = kπ;(b) g(x) = 1

2 + cos(x);( ) h(x) = x− ⌊x⌋; (d) j(x) = x; x ∈ Q;

x3; x 6∈ Q.

10 CAPÍTULO 1. ENUNCIADOS DOS EXERCÍCIOSProblema 2.Determine se f(x) =

|x+ 2|; x < 0;

3; x = 0;

3− x; x > 0.

é ontínua e al ule limx→−∞

f(x).Problema 3. (Apli ação do TVI)(a) Seja f(x) = x4 − 2x3 + x2 + 7 sen(x). Mostre que existe a ∈ R tal que f(a) = 10;(b) Mostre que existe pelo menos um b > 0 tal que log(b) = e−b;( ) Considere f ontínua em [0, 1] om 0 ≤ f(x) ≤ 1. Mostre que existe c ∈ [0, 1] talque f(c) = c;(d) Suponha que f é ontínua em [0, 2] om f(1) = −3 e f(x) 6= 0 para todo x ∈ [0, 2].Prove que f(x) < 0 para todo x ∈ [0, 2].Problema 4.Determine um valor para a ∈ R, se for possível, de modo que a função seja ontínua em R.(a) f(x) =

(x− 2)2(x+ a)

x2 − 4 x+ 4; x 6= 2

7; x = 2.(b) f(x) =

2x+ 5 se x < −1,

a se x = −1,

x2 − 3 se x > −1.( ) f(x) =

x

|x| ; |x| ≥ 1

ax; |x| < 1.(d) f(x) =

sen(

1x

)

; x 6= 0;

a; x = 0;(e) f(x) = e1/x; x > 0

a; x ≤ 0.(f) f(x) =

sen(6x)

sen(8x); x 6= 0;

a; x = 0..Problema 5.Determine a, b ∈ R, se for possível, de modo que f seja ontínua em R.

f(x) =

ax+ b; |x| ≤ 2;

|x− 1|; |x| > 2.

1.2.3 ExtrasExtra 1.Determine o onjunto dos pontos de des ontinuidade (úni os pontos onde a funçãonão é ontínua) de f(x) =

1, x ∈ Q,

1 + |x|, x 6∈ Q.Extra 2.Determine um valor para a ∈ R, se for possível, de modo que a função seja ontínuaem R.(a) f(x) =

x3 − 4x2 + 5x− 2

x3 − 2x2 + xse x 6= 1,

a se x = 1.(b) f(x) =

x2 + 2 se x < 0,

a se x = 0,√x+ 4 se x > 0.( ) f(x) =

x3 + 1

x− 1; x 6= 1

a; x = 1. (d) f(x) = e1/x; x < 0

a; x ≥ 0.

1.2. EXERCÍCIOS DE CONTINUIDADE 11(e) f(x) = 2x+ a; x ≤ 1;

x2/a; x > 1. (f) f(x) =

a x; x < 0;

1; x ≥ 0;Extra 3.Determine se é Verdadeiro (provando a armativa) ou Falso (dando um ontrae-xemplo):(a) se f é ontínua om f(0) = 2 e f(3) = 5, então f(x) > 0 para todo x ∈ [0, 3].(b) se g é ontínua om g(1) = g(3) = −10 e g(2) = 10, então g possui exatamenteduas raízes no intervalo [1, 3];Extra 4. (Apli ação do TVI)(a) Mostre que existe pelo menos um x0 ∈ R tal que x0 + 2 sen(x0) = 1.(b) Mostre que todo polinmio de grau impar possui pelo menos uma raiz.( ) Mostre que a equação sen(π sen(x)) = sen(x) possui pelo menos uma solução em[π/6, π/2].(d) Considere h(x) = sen(x) + 1 − 2

π|x|. Prove que existem x0, x1 ∈ R distintos taisque h(x0) = h(x1) = 0.Extra 5.Determine a, b ∈ R, se for possível, de modo que f seja ontínua em R.

f(x) =

−2x; x ≤ 4;

ax+ b; 1 < x < 4;

x; x ≤ 1.1.2.4 DesaosDesao 1.Um montanhista ini ia a subida do Pi o das Agulhas Negras do abrigo Rebouçasas 8h da manhã e atinge o pi o as 15h deste dia. Ele dorme no pi o e retorna na manhãseguinte as 8h, hegando de volta ao abrigo Rebouças as 15h do mesmo dia.Mostre que ele passou por um ponto do per urso na mesma hora (em dias distintos)durante a subida e durante a des ida.Desao 2.Esbo e o grá o e determine os pontos de des ontinuidade de:(a) f(x) igual ao segundo algarismo da expansão de imal de x.(b) f(x) igual ao número de 7's da expansão de imal de x se este número é nito e zero aso ontrário.( ) f(x) = 0 se x ∈ R − Q, f(p/q) = 1/q se p/q é fração irredutível om q > 0 ef(0) = 0; Di a: esbo e o grá o para q = 2, 3, . . .(d) f(x) = 0 se 1 não apare e na expansão de imal de x e f(x) = n se 1 apare e naenésima posição.Desao 3.En ontre uma função f que seja des ontínua nos seguintes pontos, mas ontínuaem todos os outros:(a) 1, 1

2, 13, 14, . . .; (b) 0, 1, 1

2, 13, 14, . . .Desao 4. Suponha que lim

x→1log x = 0. Prove que log(x) é ontínua para x > 0.

12 CAPÍTULO 1. ENUNCIADOS DOS EXERCÍCIOSDesao 5. Prove (veja outra prova no Desao da p.18), utilizando as séries da exponen ial(p.57) e do seno e osseno (p.58), a relação de Euler:eiθ = cos θ + i sen θ.

Desao 6. Utilizando a relação de Euler eiθ = cos θ + i sen θ e a denição de senh e coshdadas na p.59, prove que:senh(ix) = i sen(x) e cosh(ix) = cos(x).Tome x = iθ e prove quecos(iθ) = cosh(θ) e sen(iθ) = i senh(θ).

1.3 Exer í ios de Derivada1.3.1 Exer í ios de FixaçãoExer í io 1.Determine a equação da reta tangente ao grá o de f(x) no ponto x = −2sabendo que f(−2) = 3 e f ′(−2) = 3.Exer í io 2.Determine se é Verdadeiro ou Falso. Se for falso dê um ontraexemplo ou orrija.(a) Se f é ontínua em x = 3, então f é derivável em x = 3.(b) Se f(2) = g(2), então f ′(2) = g′(2).( ) Se f ′(1) > 0, então f(1) > 0.Exer í io 3.Considere o grá o de f abaixo.(a) se f ′(x1) = 2 determine f ′(x2) e f ′(x3).(b) Coloque em ordem res ente f ′(x2), f′(x4), f

′(x5), f′(x6).

x

y

x1 x2 x3 x4 x5 x6

f(x)

225.julho.2010

1.3. EXERCÍCIOS DE DERIVADA 13Exer í io 4.Dado o grá o de f abaixo, faça o grá o exato de f ′.x

y

f(x)

4 12

3

−2Exer í io 5. Se f e g são funções diferen iáveis tais que f(2) = 3, f ′(2) = −1, g(2) = −5,g′(2) = 2, determine o valor de h′(2) se:(a) h(x) = f(x)g(x); (b) h(x) = f(x)

g(x).Exer í io 6.Considere f e g duas funções ujos grá os estão na gura abaixo. As retasque apare em são tangentes ao grá o. Determine o valor de h′(1) se:(a) h(x) = f(x) · g(x); (b) h(x) = 5f(x)− 3g(x).

-

6

x

y

1

1

f(x)

-

6

x

y

3

2

1

g(x)

Exer í io 7. Se um balonista joga um sa o de areia de um balão a 500m de altura então osa o de areia estará numa altura (em metros) h(t) = 500−16t2 após t segundos. Determine:(a) sua velo idade em t = 2;(b) em qual instante t o sa o atingirá o solo;( ) om qual velo idade o sa o atingirá o solo;(d) om qual a eleração o sa o atingirá o solo.Exer í io 8.Cal ule a derivada em relação a x das funções:(a) ex log x; (b) cosx

x+ 5( ) cos(x3 + 1);(d) eπ + log(π2 + 1). (e) log(1 + sen x); (f) |x− 2|;Exer í io 9.Cal ule:

14 CAPÍTULO 1. ENUNCIADOS DOS EXERCÍCIOS(a) d

dr

(

4

3πr3); (b) d

dk(3k2 − k−1); ( ) du

dtse u = t log t;(d) dv

dsse v = sπ; (e) dy

dxse y = (

√3)x; (f) d

dt(log π).Exer í io 10.Estude o Teorema do Valor Médio (Corolário 5 da p.83) e responda. Suponhaque f é derivável em R e −4 ≤ f ′(x) ≤ 3 para todo x ∈ R. Prove que:(a) −16 ≤ f(5)− f(1) ≤ 12; (b) −4h ≤ f(h)− f(0) ≤ 3h para todo h > 0.Exer í io 11.Um objeto ai do alto de um edifí io de 100m e atinge o solo em 5 segundos.Aplique o Teorema do Valor Médio (TVM) e prove que em algum instante o objeto estava om velo idade (em módulo) igual a 20m/s.Exer í io 12. Suponha que f ′′(x) = 0 para todo x ∈ R. Sabendo que f ′(−3) = 0 e

f(5) = π, aplique uma onsequên ia do Teorema do Valor Médio (TVM) duas vezes para on luir que f(x) = π para todo x ∈ R.Exer í io 13.Considere f e g duas funções ujos grá os estão na gura abaixo. As retasque apare em são tangentes ao grá o.(a) Se h(x) = f(g(x)), determine h′(2).(b) Se k(y) = g−1(y), determine k′(3).-

6

x

y

QQQQQQQQQQQ

QQ

QQQ

3

2f(x)

-

6

x

y

2

2

3

g(x)

Exer í io 14.Considere o grá o abaixo.x

y

Se o grá o representa f(x) determine maiores intervalos (indique no grá o) onde:(a) f ′ é positiva e negativa; (b) f é injetiva (possui inversa).Se o grá o representa f ′(x) determine maiores intervalos (indique no grá o) onde:( ) f é res ente e de res ente; (d) f é injetiva (possui inversa).

1.3. EXERCÍCIOS DE DERIVADA 151.3.2 ProblemasProblema 1.Cal ule, pela denição (utilizando limite), a derivada de:(a) f(x) = 1

x2; (b) f(x) = 1√

x.Problema 2.Determine a, b ∈ R tais que a função abaixo tenha derivada em todos os pontos.

f(x) =

x2; x < 1;

ax+ b; x ≥ 1.Problema 3. Suponha que |f(x)| ≤ |x|k om k > 1. Cal ule pela denição f ′(0).Di a: Veja o Exemplo 57 da p.72.Problema 4.Para ada uma das funções abaixo, determine onde possui derivada e al ule aderivada nestes pontos.(a) g(x) = 3; x < 2;

−4; x ≥ 2;(b) f(x) = |ex − 1|; ( ) h(x) = |(3− x)(x+ 1)|.Problema 5.Em ada um dos itens abaixo, s(t) representa a posição de uma partí ula semovendo em linha reta no instante t. Determine:(i) A velo idade e a eleração da partí ula no instante t = 0.(ii) Os instantes em que a partí ula esta parada.(a) s(t) = t2 − 1

t2 + 1; (b) s(t) = sen t.Problema 6.Considere a função f(x) = 2x3 − 2x2 + 5. Determine todos os pontos dográ o de f nos quais a reta tangente é:(a) horizontal; (b) paralela à reta 2y − 20x− 50 = 0.Problema 7.A he ondições sobre a, b, c ∈ R para que a urva:(a) y = ax3 + bx2 + cx+ π tenha uma úni a reta tangente horizontal;(b) y = ax2 + bx + c tenha x + y = 1 e y = −1 respe tivamente omo retas tangentesnos pontos x1 = −1 e x2 = 1.Problema 8.Cal ule as derivada (em relação a x) das funções:(a) (5x2 − 3x+ 4)300; (b) sen( 7

cos(x2) + 4); ( ) x2 + 1

e−x + 1; (d) 3

√x+ t

x2 + k;(e) log(sen(5ex)) · x4; (f) arctan(log(3x2 + 1)); (g) earcsen(4−5x).Problema 9.Dado que f(4) = 3, f ′(4) = −5 e g(x) = 3 log(f(x) + x), determine g′(4).Problema 10.Considere m0, T0, K, a, b, c, d ∈ R. Cal ule:(a) f ′(t) se f(t) = m0e

(T0−t)/K ; (b) f ′(x) se f(x) =

(

ax+ b

cx+ d

)2;

16 CAPÍTULO 1. ENUNCIADOS DOS EXERCÍCIOS( ) f ′(θ) se f(θ) = K sen(aθ3 + b); (d) f ′(t) se f(t) = eKt cos(at).Problema 11.Determine a equação da reta tangente ao grá o de:(a) y =sen(x2)

xpara x =

π/2; (b) y = esen(−2x) no ponto (π, 1).Problema 12.Mostre que:(a) ex ≥ 1 + x para x ≥ 0.(b) a equação 2x3 − 15x2 + 60x+ 4 = 0 possui exatamente uma raiz real.Problema 13. (Apli ações do Teorema do Valor Médio)(a) Dois orredores ini iaram a orrida no mesmo instante e terminaram empatados. Proveque em algum instante durante a orrida ele têm a mesma velo idade.(b) Considere f diferen iável om f(0) = 0 e f ′(x) ≤ 1 para todo x > 0. Mostre quef(x) ≤ x para todo x > 0.( ) Mostre que existe uma úni a função diferen iável h : R → R tal que: h′(x) = h(x);

h(0) = 1.Di a: Suponha que h1 e h2 são soluções. Dena f(x) =h1(x)

h2(x), al ule f ′(x) e f(0).(d) Considere f(x) = x2ex e g(x) = e

√x. Prove que existe um c ∈ (0, 1) tal que asretas tangentes ao grá o de f e de g são paralelas em x = c.Problema 14. Se f e g são funções diferen iáveis tais que f ′(2) = −1, f(2) = 3, g(−1) = 2,

g′(−1) = 6, determine o valor de h′(2) se:(a) h(x) = f(g(−x/2)); (b) h(y) = g−1(y).Problema 15. Sabendo que a equação da reta tangente ao grá o de y = f(x) no ponto(−1, 3) passa no ponto (0, 6), determine (f−1)′(3).Problema 16. Seja f(x) = cos(x5 + 2x + π/2). Sabendo que f(0) = 0, e que g(y) é ainversa de f perto de y = 0, determine g′(0).1.3.3 ExtrasExtra 1. Se f e g possuem derivada e são tais que: f(2) = 3, f ′(2) = −1, g(2) = −5,g′(2) = 2, determine:(a) w′(2) se w(x) =

4f(x)

g(x); (b) m′(0) se m(x) = exg(3x+ 2).Extra 2.Cal ule a derivada (em relação a x) das funções:(a) sen(x ex log x); (b) sen(sen(sen x))); ( ) 3arctan x; (d) √x+

√x;(e) sen(cosx sen x); (f) sen |1− x2|; (g) eex4 ; (h) log(sen(2x))√x2 + 1.

1.3. EXERCÍCIOS DE DERIVADA 17Extra 3.Determine todos os pontos do grá o de y(x) = x3 + 2x2 − 4x + 5 nos quais areta tangente:(a) é horizontal; (b) é paralela à reta 2y + 8x− 5 = 0.Extra 4.Determine a, b ∈ R tais que a função abaixo tenham derivada em todos os pontos.f(x) =

ax2 + b; x ≤ 1;1

x; x > 1

.Extra 5. Sabendo que g é ontínua em a e f(x) = (x− a)g(x), determine f ′(a).Extra 6.Determine a equação da reta tangente ao grá o da função no ponto indi ado.(a) y = x2 sen x no ponto (π, 0). (b) y = log(√x− 2), no ponto ((e + 2)2, 1).Extra 7.Determine:(a) os pontos da urva y =

1

xnos quais a reta tangente é paralela à reta 2x+ 3y = 0;(b) a(s) reta(s) tangente(s) ao grá o de y = e2x que ontem(êm) o ponto (5/2, 0).Extra 8. (Função res ente/de res ente) Mostre que:(a) x > log x para x > 1;(b) a equação −2x13 − 6x5 − x+ 10 = 0 possui exatamente uma raiz real.Extra 9. (Apli ações do Teorema do Valor Médio)(a) Suponha que f é uma função diferen iável em R e tenha duas raízes reais distintas.Mostre que f ′ tem, no mínimo, uma raiz real.(b) Considere uma função f diferen iável om f ′(x) ≤ 4 para todo x ∈ (2, 5). Prove que

f(5)− f(2) ≤ 12.( ) Mostre que existe uma úni a g função derivável em R tal que:g′′(x) = cos(2x+ log(x4 + 1)), g′(2) = −1 e g(3) = 5.Di a: Suponha que g1 e g2 são soluções. Dena f(x) = g1(x) − g2(x), al ule f ′′(x) e

f ′(2). Con lua que f ′(x) = · · · . Depois al ule f(3).Extra 10.Considere f(x) =

xk; x ∈ Q;

0; x 6∈ Q om k > 1. Cal ule pela denição f ′(0).Di a: Veja o Exemplo 57 da p.72.Extra 11.Determine uma fórmula para a derivada (fgh)′.

18 CAPÍTULO 1. ENUNCIADOS DOS EXERCÍCIOS1.3.4 DesaosDesao 1.Cal ule pela denição a derivada no ponto x = 0 de:(a) f(x) = x sen(1/x); x 6= 0;

0; x = 0;(b) g(x) = x2 sen(1/x); x 6= 0;

0; x = 0.Desao 2.Use o binmio de Newton para al ular pela denição a derivada de h(x) = xnpara n ∈ N.Desao 3.Considere f(x) = −x2

2. Determine uma função g tal que, para todo x > 0, areta tangente ao grá o de f em x seja paralela à reta normal ao grá o de g em x.Desao 4.Considere f uma função polinomial de grau 2. Mostre que a reta se ante aográ o de f nos pontos x1 e x2 é paralela à reta tangente no ponto médio x1 + x2

2quaisquerque sejam x1 e x2.Desao 5. Prove (veja outra prova no Desao da p.12) usando derivada de seno, osseno eexponen ial que

eiθ = cos θ + i sen θ.Di a: Dena f(θ) =cos θ + i sen θ

eiθe derive. Quando derivar, trate i ∈ C omo uma onstante.Desao 6.Prove que existe um úni o par de funções s(x) e c(x) deriváveis para todo x ∈ Rtais que s′(x) = c(x);

s(0) = 0;e

c′(x) = −s(x);

c(0) = 1.Di a: Suponha que existam s1, c1 e s2, c2. Dena f = (s1 − s2)2 + (c1 − c2)

2. Mostreque f ′(x) = 0 e f(0) = 0. Aplique o TVM.1.4 Exer í ios de Apli ação de Derivada1.4.1 Exer í ios de FixaçãoExer í io 1. Suponha que f(0) = 0, f ′ é ontínua e que f ′(0) = 5. Cal ule lim

x→0

f(x)

sen(x).Exer í io 2.Vamos al ular o limite lim

x→1

(x− 1)2

ex − eapli ando L'Hospital duas vezes. Assim,

limx→1

(x− 1)2

ex − e= lim

x→1

2(x− 1)

ex= lim

x→1

2

ex=

2

e2.226.juj.2010

1.4. EXERCÍCIOS DE APLICAÇO DE DERIVADA 19Na realidade o limite é zero. Qual o erro?Exer í io 3. Sabe-se que f ′(2) = 4 e que f(2) = 5. Cal ule de forma aproximada(a) f(2.1); (b) f(1.95).Exer í io 4. Sabe-se que p(x) é o polinmio do segundo grau que melhor aproxima f(x) =cos(x) perto do ponto x = π. Determine:(a) p(π); (b) p′(π); ( ) p′′(π).Exer í io 5.Esbo e o grá o de uma função ontínua para ada item abaixo que:(a) tenha um máximo lo al em x = −2 e um mínimo lo al em x = 1;(b) seja sempre res ente, mas até x = −2 om on avidade para ima e depois desteponto om on avidade para baixo.Exer í io 6.Considere uma f : [−3, 3] :→ R ujo quadro de sinais da função e das derivadasseja:

−2 −1 0 1 2f − − + + − −f ′ − + + − − +f ′′ + + − − + +Esbo e o grá o de y = f(x).Exer í io 7.Determine se é Verdadeiro ou Falso. Se for falso dê um ontraexemplo ou orrija.(a) se f ′′(x) > 0 para todo x ∈ [1, 2] então f ′ é res ente em [1, 2].(b) se f ′′(x) > 0 para todo x ∈ [1, 2] então f é possui on avidade para ima em [1, 2]( ) se h(x) = C para todo x ∈ [1, 2] então h não possui nenhum ponto do máximo nemmínimo lo al.Exer í io 8.Estude o Teorema 22 da p.110, o TVE (Teorema do Valor Extremo de Weiers-trass). Determine se é Verdadeiro ou Falso. Se for falso dê um ontraexemplo ou orrija.(a) Pelo TVE toda função ontínua em I = (−7, 100) possui um máximo em I.(b) Pelo TVE toda função ontínua em I = [0,+∞) possui um mínimo em I.( ) Pelo TVE toda função em I = [2, 3] possui um mínimo em I.(d) Pelo TVE toda função des ontínua em I = [2, 4] NO possui máximo neste intervalo.(e) Pelo TVE toda função ontínua em um intervalo ILIMITADO I NO possui máximoem I.Exer í io 9. Suponha que f é derivável em R e que f ′ se anula somente em 3 e 7.(a) É verdade que existe a ∈ [1, 10] tal que f(a) ≥ f(x) para todo x ∈ [1, 10]? Porque?(b) Explique omo podemos determinar a.( ) É verdade que existe c ∈ R tal que f(c) ≤ f(x) para todo x ∈ R? Porque?Exer í io 10. Sabendo f é ontínua em R e que f ′(x) > 0 para x < 0 e f ′(x) < 0, para

x > 0, determine (se for possível) a, b, c, d ∈ R tais que

20 CAPÍTULO 1. ENUNCIADOS DOS EXERCÍCIOS(a) minx∈[−5,−1]

f(x) = f(a);(b) maxx∈[−2,3]

f(x) = f(b);( ) minx∈[−1,2]

f(x) = f(c);(d) maxx∈[2,5]

f(x) = f(d);Exer í io 11.Considere f(x) =1

x. Determine, aso existam, para ada intervalo I abaixo,

maxx∈I

f(x), minx∈I

f(x) e os pontos xmax e xmin onde são atingidos o máximo e o mínimo.(a) I = [2, 3]; (b) I = (0, 1]; ( ) I = [−1,−4]; (d) I = [1,+∞); (e)I = (−∞, 0).Exer í io 12.Determine se é Verdadeiro ou Falso. Se for falso dê um ontraexemplo ou orrija. Suponha que todas as funções possuem derivadas em todos os pontos.(a) Se x = 4 é mínimo lo al de h então h′(4) = 0.(b) Se x = 2 é o máximo de f no intervalo [1, 4] então f ′(2) = 0.( ) Se x = 1 é o mínimo de f no intervalo [1, 4] então f ′(1) = 0.(d) Se g′(3) = 0 então x = 3 é o mínimo ou máximo lo al de g.Exer í io 13.Determine se é Verdadeiro ou Falso. Se for falso dê um ontraexemplo ou orrija.Sabendo que f e f ′ é derivável em I e a, b, c ∈ I:(a) f ′(b) = 0 e f ′′(b) = −1 então b é ponto de máximo lo al.(b) f ′(c) = 0 e f ′′(c) = 0 então c é NO é ponto de máximo nem mínimo de f em I.( ) se a é máximo lo al de f então a é máximo de f em I.Exer í io 14.Considere a função f esboçada na gura abaixo.(a) Determine os pontos de máximo e mínimo lo al de f .Determine os pontos de máximo e mínimo de f em:(b) [2, 4]; ( ) [−3, 1]; (d) [−1, 4].Determine o sinal de f ′′ em:(e) x = −1.8; (f) x = 0; (g) x = 4.(h) Dentre os inteiros −3,−2, . . . , 4, determine os que estão próximos de pontos deinexão (tro a de on avidade) de f .Considere g′(x) = f(x). Determine os pontos de:(i) máximo e mínimo lo al de g; (j) inexão de g.

x

y

−3 −2 −1 1 2 3 4f(x)

1.4. EXERCÍCIOS DE APLICAÇO DE DERIVADA 21Exer í io 15.Considere f(x) = x4 − x3. Determine TODOS os pontos de:(a) máximo/mínimo lo ais de f .(b) máximo/mínimo de f no intervalo [−1, 2].( ) máximo/mínimo de f no intervalo [−1, 0].(d) máximo/mínimo de f em R.(e) máximo/mínimo de f em (−∞, −1].1.4.2 ProblemasProblema 1.Cal ule os limites abaixo:(a) limx→0

sen(8x)

e2x − 1(b) lim

x→0+(ex + 3x)1/x ( ) lim

x→0

ex − e−x

sen(5x)(d) lim

x→0

ax − bx

xProblema 2.Estime, através de uma aproximação linear lo al:(a) √65; (b) log(e2 − 0.1); ( ) arctan(1.2).Problema 3.Considere a função f(x) = ax3 + bx2 + cx+ d onde a > 0.(a) Mostre que f admite nenhum ou dois extremos lo ais. Sob que ondições ada umdesses asos o orre?(b) No aso em que f não admite extremos lo ais, quantas raízes reais f pode ter?( ) No aso em que f admite dois extremos lo ais, quantas raízes reais f pode ter?(d) Baseado nos itens anteriores, des reva um pro edimento para determinar o número deraízes reais de f .Problema 4. (grá os triviais) Esbo e o grá o de f e de uma função g tal que:(a) f(x) = x3 − 3x2 + 3x+ 1; (b) g′(x) = x3 − 4x.Problema 5.Esbo e o grá o de uma função y = f(x) tal que f(0) = 2 e f ′ é dado pelográ o abaixo.-2 -1 0 1 2

x

y

f ’

-2 -1 0 1 2 x

y

f ’

-2 -1 0 1 2 x

y

f ’

-2 -1 0 1 2 x

y

f ’

Problema 6.Esbo e o grá o de uma função ontínua f nos maiores intervalos possíveisque verique todas as ondições indi adas simultaneamente.(a)• lim

x→0−f(x) = −∞, lim

x→0+f(x) = 1, f(0) = −1,

• limx→−∞

f(x) = 2, limx→+∞

f(x) = −1,• f ′(x) > 0 para x < −1, f ′(x) < 0 para −1 < x < 0, f ′(−1) = 0, f ′(x) < 0para x > 0.

22 CAPÍTULO 1. ENUNCIADOS DOS EXERCÍCIOS(b)• f(0) = 2, f(−2) = 1 e f ′(0) = 0.• lim

x→+∞f(x) = −1 e lim

x→−∞f(x) = 0.

• limx→2+

f(x) = +∞ e limx→2−

f(x) = −∞.• f ′(x) > 0 se x < 0 e f ′(x) < 0 se x > 0.• f ′′(x) < 0 se |x| < 2 e f ′′(x) > 0 se |x| > 2.Problema 7.Para as questões de esboço de grá o, antes do esboço deverá ser determinado:(a) TODOS os pontos de interseção om os eixos x e y;(b) os limites de no innito e TODAS as assíntotas;( ) os intervalos de res imento e de res imento;(d) TODOS os pontos de máximo e mínimo lo ais;(e) os intervalos om on avidade para ima e para baixo;Esbo e o grá o de ada uma das funções abaixo:(a) f(x) = 2x2 − 2x

(x− 2)(x+ 1). Di a: f ′(x) =

4 (1− 2x)

(x− 2)2 (x+ 1)2e

f ′′(x) =24 (x2 − x+ 1)

(x− 2)3 (x+ 1)3.(b) g(x) = 1 + x2

1− x2. Di a: g′(x) = 4

x

(1− x2)2e g′′(x) = 4

1 + 3 x2

(1− x2)3.( ) h(x) = x

(x− 1)2. Di a: h′(x) = − x+ 1

(x− 1)3e h′′(x) =

2 (x+ 2)

(x− 1)4.Problema 8.Esbo e o grá o da função:(a) f(x) = ex

x. Di a: f ′(x) =

(x− 1) ex

x2e f ′′(x) =

(x2 − 2 x+ 2) ex

x3.(b) f(x) = log(1− x2) + 1. Di a: f ′(x) =

2 x

x2 − 1e f ′′(x) = − 2 x2 + 2

(x2 − 1)2,

√1− e−1 ≈ 0.79.( ) f(x) = e(2−x)(x−1) + 1. Di a: f ′(x) = (3− 2x) e(2−x)(x−1) e

f ′′(x) = (4x2−12x+7) e(2−x)(x−1), 3/2−√2/2 ≈ 0.79 e 3/2+

√2/2 ≈ 2.20.(d) f(x) = x3 ex. Di a: f ′(x) = (x3 + 3 x2) ex e f ′′(x) = (x3 + 6 x2 + 6 x) ex,

−3 −√3 ≈ −4.7 e −3 +

√3 ≈ 1.26.Problema 9.Para ada função f e ada intervalo I abaixo, determine max

x∈If(x) e min

x∈If(x)e, se for possível, os pontos xmax e xmin onde o máximo/mínimo é atingidos.(a) f(x) = 1

sen(x)+

1

cos(x), I = (0, π/2).Di a: f ′(x) =

sen3 x− cos3 x

cos2 x sen2 x(b) f(x) = x+ 1x, I = (0, +∞), I = (0, 3], I = [3, 4].Di a: f ′(x) = 1− 4

x2.

1.4. EXERCÍCIOS DE APLICAÇO DE DERIVADA 23( ) f(x) = 3x4 − 4x3 + 12x2, I = [−1, 1] e [1, 2].Di a: f ′(x) = 12x(x2 − x+ 2) x.(d) f(x) = x

x+ 1em I = (−1, 1], I = [0, 1].Di a: f ′(x) =1

(x+ 1)2.Problema 10.Determine todos K ∈ R tais que a equação x

x4 + 3= K tenha pelo menosuma solução.Problema 11.En ontre dois números uja diferença seja 100 e ujo produto seja um mínimo.Problema 12.Uma hapa de metal de largura L deve ter duas bandas, de igual largura,dobradas ao longo do omprimento de maneira a formar uma alha retangular.Como devem ser feitas as dobras de tal forma que a alha omporte o maior volumepossível?

LProblema 13.Dispõe-se de 40m de o de arame para er ar um anteiro em um jardim ujaforma é a de um setor ir ular (fatia de pizza). Qual deve ser o raio do ír ulo para que o anteiro tenha a maior área possível ?Obs: A área de um setor ir ular é θr2/2, onde r é o raio do ír ulo e θ é o ângulo dosetor ir ular.θ

r

Problema 14.A tela do inema CABRALPLEX está a uma distân ia K do hão e possuialtura L. Um espe tador vai se sentar nesta sala, que é plana (não possui in linação), demodo que sentado em qualquer assento a distân ia entre seus olhos e o solo é h. A quedistân ia d da tela ele deve ar sentado para que per eba a maior imagem possível da tela?

24 CAPÍTULO 1. ENUNCIADOS DOS EXERCÍCIOSNote que a imagem é propor ional ao ângulo subentendido por seu olho e os extremos datela. Assumimos que a altura K > h, aso ontrário o melhor seria d = 0.h

L

K

B

d

θϕ

Problema 15.A página de um artaz deve ser retangular e ter uma área de A m2 ommargens laterais iguais a M m, e margens superior e inferior de N m. Determine asdimensões do artaz que permitirão a maior área impressa.

N

MM

N

Problema 16.Um tanque ilíndri o tem a forma de um ilindro om duas semiesferas em ada extremidade. Determine a forma do ilindro que:(a) maximizará o seu volume, sabendo que sua área de superfí ie é A,(b) minimizará o seu usto de fabri ação sabendo que seu volume é V .Problema 17.(a) Sejam f(x) = 2 +

√6x− 2x2 e P = (2, 2). Determine a maior e a menor distân iade P aos pontos do grá o de f .(b) Qual a menor distân ia verti al entre as urvas y = x2 e y = − 1

x2?

1.4. EXERCÍCIOS DE APLICAÇO DE DERIVADA 25Problema 18.Determine as dimensões do retângulo ins rito num ír ulo de raio R que possuio menor e o maior perímetro;Problema 19.En ontre as dimensões do retângulo de maior área que tem sua base sobre oeixo x e seus dois outros vérti es a ima do eixo x e sobre a parábola y = 27− x2.Problema 20.Maximize o volume do:(a) one reto ins rito numa esfera de raio R;(b) ilindro ir ular reto ins rito num one ir ular reto de raio R e altura H .

1.4.3 ExtrasExtra 1.Cal ule os limites abaixo.(a) limx→1

log(4x− 3)

x− 1(b) lim

x→+∞

x2

log x( ) lim

x→1

4√x− 1

5√x− 1

(d) limx→+∞

x1/x

Extra 2.Estime, através de uma aproximação linear lo al:(a) tan(0.05). (b) 3√28.Extra 3.Esbo e o grá o de uma função f e de uma função g tal que:(a) g′(x) = x2 − 5x+ 6 omo derivada; (b) f(x) = (x− 1)2(x+ 1)2.Extra 4.Esbo e o grá o de uma função y = f(x) tal que f(0) = 2 e que tenha omoderivada o seguinte grá o:

26 CAPÍTULO 1. ENUNCIADOS DOS EXERCÍCIOS-

6

x

y

0−2 −1 1 2 3 4

Extra 5.Esbo e o grá o de uma função ontínua f que verique todas as ondições indi- adas simultaneamente.(a) f(0) = 3, f(2) = 1, f ′(0) = f ′(2) = 0f ′(x) > 0 se |x− 1| > 1 f ′(x) < 0 se |x− 1| < 1f ′′(x) < 0 se x < 1 f ′′(x) > 0 se x > 1(b) f(2) = 4; f(4) = −1.f ′(2) = 0; f ′(x) > 0 se x < 2; f ′(x) < 0 se x > 2;f ′′(4) = 0; f ′′(x) < 0 se x < 4; f ′′(x) > 0 se x > 4;lim

x→−∞f(x) = −∞; lim

x→+∞f(x) = −3.Extra 6.Para as questões de esboço de grá o, antes do esboço deverá ser determinado:(a) TODOS os pontos de interseção om os eixos x e y;(b) os limites de no innito e TODAS as assíntotas;( ) os intervalos de res imento e de res imento;(d) TODOS os pontos de máximo e mínimo lo ais;(e) os intervalos om on avidade para ima e para baixo;(função ra ional) Esbo e o grá o de ada uma das funções abaixo:PS: Ignore on avidade, não al ule f ′′.(a) f(x) = (x+ 1)(x+ 2)

(x+ 3)(x− 1). Di a: f ′(x) = − x2 + 10 x+ 13

(x− 1)2 (x+ 3)2.(b) g(x) = x

(x− 3)(1− x)+ 3. Di a: g′(x) = x2 − 3

(x− 3)2 (x− 1)2.Extra 7. (função ra ional) Esbo e o grá o de ada uma das funções abaixo:(a) f(x) = x2

x2 + 3.Di a: f ′(x) = 6x

(x2 + 3)2e f ′′(x) = 18

1− x2

(x2 + 3)3.(b) g(x) = x2

4− x2+ 2.Di a: g′(x) = 8 x

(x− 2)2 (x+ 2)2e g′′(x) = 8

4 + 3 x2

(4− x2)3.

1.4. EXERCÍCIOS DE APLICAÇO DE DERIVADA 27Extra 8. (função não-ra ional) Esbo e o grá o da função:(a) f(x) = x log xDi a: f ′(x) = log x+ 1, f ′′(x) =1

x.(b) f(x) = x e1−x2Di a: f ′(x) = (1− 2x2) e1−x2, f ′′(x) = 2x(2x2 − 3) e1−x2,

1/√2 ≈ 0.707, √

3/2) ≈ 1.22.( ) f(x) = x2 exDi a: f ′(x) = x(x+ 2) ex, f ′′(x) = (x2 + 4 x+ 2) ex,−2−

√2 ≈ −3.41, −2 +

√2 ≈ −0.58).Extra 9.Para ada função f e ada intervalo I abaixo, determine max

x∈If(x) e min

x∈If(x) e,se for possível, os pontos xmax e xmin onde o máximo/mínimo é atingidos.(a) f(x) = 8x2 − x4 em I = R; I = [−1, 1].Di a: f ′(x) = 4x(4− x2).(b) f(x) = 1

x2 + 1em I = [1, 2] e I = [−1,+∞).Di a: f ′(x) = − 2x

(x2 + 1)2.( ) f(x) = sen(cosx) em [0, 2π].Extra 10. (problema om modelo simples) Suponha que uma janela tenha a forma de umretângulo om um triângulo equilátero no topo. Assumindo que o perímetro é de 12m,determine as dimensões da janela para que penetre o máximo de luz possível.Di a: área do triângulo equilátero é L2

√3/4.Extra 11.Modele os seguintes problemas e depois resolva-os.(a) Determine as dimensões do retângulo om perímetro P > 0 que possui a maior área.(b) Um ampo retangular está limitado por uma er a em três de seus lados e por um órrego reto no quarto lado. A he as dimensões do ampo om área máxima que pode ser er ado om uma er a de omprimento total P .( ) Um terreno retangular deve ser er ado om dois tipos de er a. Dois lados opostosdevem re eber uma er a reforçada que usta R$3,00 por metro, enquanto os outros doislados uma er a que usta R$2,00 por metro. Quais as dimensões do terreno de maior áreaque pode ser er ado om R$6.000,00?Extra 12.(a) A he os números x e y, om soma igual a S, uja soma dos quadrados seja o menorpossível.(b) Determine o número positivo tal que a diferença entre ele e o seu ubo seja a menore a maior possível.( ) Suponha que o produto de dois número reais positivos é igual a P > 0. Determine omínimo e máximo da soma destes dois números.Extra 13.Queremos fazer uma aixa em forma de paralelepípedo de base quadrada e abertaem ima, isto é, uma aixa sem tampa de base quadrada.

28 CAPÍTULO 1. ENUNCIADOS DOS EXERCÍCIOS(a) Se o volume desta aixa é V cm3, determine as dimensões que minimizam a quantidadede material.(b) Se temos A cm2 de material para fazer a aixa, determine o maior volume possívelpara esta aixa.( ) Se o volume desta aixa é V cm3 e o usto do material da base é duas vezes mais aro que o usto do material dos lados, determine as dimensões que minimizam o usto defabri ação.Extra 14. (guras e parte do texto retirados da Wikipédia) Em ópti a, o prin ípio de Fermatou prin ípio do menor tempo diz que o aminho de um raio de luz entre dois pontos deveser o que pode ser feito no menor tempo possível. Deste prin ípio pode ser deduzido a lei dereexão e a lei de refração de Snell. Vamos deduzir ambos neste exer í io.(a) Considere um raio de luz que parte de P e vai até Q depois de reetir no espelho emum ponto O. Determine a relação entre o ângulo de in idên ia θi e o ângulo de reexão θrpara que o tempo per orrido pelo raio seja o menor possível.

(b) Considere um raio de luz que parte de P e vai até Q passando de um meio onde aluz possui velo idade v1 para um meio onde a velo idade é v2. Determine a relação entre osângulos θ1 e θ2 e as velo idades (a Lei de Snell) para que o tempo per orrido pelo raio sejao menor possível.

Extra 15.Determine o ponto da urva indi ada mais próximo do ponto indi ado.(a) urva x2 − y2 = 1 e ponto (0, 2); (b) urva y = x3 e ponto (4, 0);( ) elipse 4x2 + y2 = 8 e ponto (1, 0); (d) urva y =√x e ponto (2, 0).Extra 16.Determine as dimensões do retângulo ins rito: em um semi ír ulo de raio R quepossui a maior área.

1.4. EXERCÍCIOS DE APLICAÇO DE DERIVADA 29Extra 17.Um ilindro é gerado ao se girar um retângulo de perímetro P em torno de umde seus lados. Qual deve ser a razão entre os lados do retângulo de tal forma que o ilindrotenha o maior volume possível?Extra 18.Maximize o volume do:(a) ilindro ir ular reto ins rito numa esfera de raio R;(b) one reto ins rito, de abeça para baixo, om vérti e no entro da base de um one ir ular reto de raio R e altura H .Extra 19.Uma er a de altura H a em volta de um prédio bem alto. Se a er a está a umadistân ia L do prédio, qual a menor es ada que vai do hão por ima da er a até a parededo prédio?L H

1.4.4 DesaosDesao 1. (formas indeterminadas) Vamos mostrar que 00, (+∞)0 e 1+∞ podem dar qual-quer número. Cal ule os limites abaixo (use L'Hospital) assumindo que k > 0:(a) limx→0+

x(log k)/(1+log x) = 00;(b) limx→+∞

x(log k)/(1+log x) = (+∞)0;( ) limx→0

(x+ 1)(log k/x) = 1+∞.Desao 2. Sua asa possui um orredor longo de largura a que termina num orredor per-pendi ular a este de largura b. Vo ê deseja mover um sofá de largura c (menor que a e b !).Desprezando a altura ( onsidere o sofá omo um retângulo), qual o omprimento máximo dosofá que pode fazer a urva ? (Somente monte o problema, as ontas são ompli adas, epodem ser resolvidos somente numeri amente).Desao 3.Prove que a menor distân ia entre o ponto (a, b) até o grá o de y = f(x) émedido na reta normal ao grá o de f .Desao 4.Prove que a distân ia entre o ponto (x0, y0) e a reta ax + by + c = 0 é igual a|ax0 + by0 + c|√

a2 + b2.

30 CAPÍTULO 1. ENUNCIADOS DOS EXERCÍCIOSDesao 5.Podemos aproximar log a (onde a > 1) pela soma das áreas de dois trapézios, on-forme a gura abaixo. Determine o ponto x de maneira que o erro da aproximação seja mínimo.1 x a

x

y

y=1/x

1 x a x

y

y=1/x

1 x a x

y

y=1/x

1 x a x

y

y=1/x

1 x a x

y

y=1/x

1 x a x

y

y=1/x

Desao 6.Maximize a área lateral do:(a) one reto ins rito numa esfera de raio R;(b) ilindro ir ular reto ins rito num one ir ular reto de raio R e altura H .

Desao 7.Deseja-se atravessar um lago ir ular até um ponto diametralmente oposto. Pode-se nadar e orrer. A velo idade orrendo é o dobro da velo idade nadando. Qual deve ser oper urso para se hegar mais rapidamente ao lado oposto?Desao 8. ( urvatura) Dado uma urva y = f(x) queremos determinar o raio do ír uloque os ula esta urva no ponto x = c. Mais pre isamente, queremos determinar o raio do ír ulo que passa por (c, f(c)) om mesma tangente e mesma derivada segunda que f .Mostre que se η = f(c), η1 = f ′(c) e η2 = f ′′(c) então o raio do ír ulo é igual a(1 + η21)

3/2

η2. O inverso do raio é hamado de urvatura da urva no ponto x = c.

1.4.5 ⋆Problemas (Taxas Rela ionadas)Problema 1.Um balão esféri o é esvaziado da tal forma que seu raio de res e a uma taxa onstante de 15 m/min. Com que taxa o ar estará saindo do balão quando o raio for iguala 9 m ?Problema 2.Um balão eleva-se verti almente do solo à razão de 3m/s. Quando o balãoestá a 48m do solo, passa, exatamente sobre ele um arro viajando a velo idade de 20m/s.Quatro segundos após este instante, om que velo idade varia a distân ia entre eles?

1.4. EXERCÍCIOS DE APLICAÇO DE DERIVADA 31Problema 3.Uma fonte luminosa aproxima-se perpendi ularmente de uma parede om velo- idade onstante de a metros/segundo, projetando uma imagem ir ular sobre esta. Sabe-seque a abertura do fa ho de luz (o ângulo entre os raios limites) é de 90o. Cal ule a velo idade om que a área iluminada sobre a parede está diminuindo quando a distân ia da fonte até aparede é de k metros.Problema 4.Um tanque ni o om água e vérti e para baixo tem raio R metros no topo ealtura H metros. Se a água uir para dentro do tanque om taxa onstante de V m3/s, omque velo idade em m/s a profundidade da água vai res er quando o tanque estiver om Lmetros de profundidade?Problema 5.Uma es ada de tamanho L está apoiada numa parede. Se a base for puxadaao longo do hão, afastando-se da parede om velo idade V , om que velo idade o topo daes ada estará se movendo para baixo quando ele está a uma altura H do solo?Problema 6.Mostre que:(a) se o raio de um ír ulo res e a uma taxa onstante, então sua área res e a uma taxapropor ional ao omprimento do raio.(b) se a aresta de um ubo res e a uma taxa onstante, então seu volume res e a umataxa propor ional à área da superfí ie.( ) se o volume de uma esfera está res endo a uma taxa propor ional à área de suasuperfí ie, então seu raio res e a uma taxa onstante.1.4.6 ⋆Problemas (Derivação Implí ita)Problema 1. Seja y = f(x) denida impli itamente em ada item abaixo. Determine aequação da reta tangente no ponto indi ado:(a) y3 + x2y = 130 em (1, 5); (b) x2 =x+ y

x− yem (−1, 0).Problema 2.Considere a urva x3 + y3 = 3xy. Determine os pontos onde a reta tangenteé verti al e onde é horizontal.Problema 3. Seja y = f(x) denida impli itamente por x2 − y2 +

√xy = 2 próximo aoponto (2, 2).(a) Cal ule f ′(2).(b) Determine a equação da reta tangente ao grá o de f(x) no ponto (2, 2).( ) Determine a equação da reta tangente ao grá o de g(x) = f(x)/x no ponto (2, 1).Problema 4.Para ada uma das funções y = f(x) denidas impli itamente perto de (x, y) =

(a, b) determine:• se a função é res ente ou de res ente perto de x = a;• f ′(a);• f ′′(a).

32 CAPÍTULO 1. ENUNCIADOS DOS EXERCÍCIOS(a) x5 + xy + y5 = 3 em (a, b) = (1, 1).(b) x cos(xy) = 0 em (a, b) = (1, π/2)Problema 5.En ontre o máximo e o mínimo de y = f(x) denida impli itamente porx4 − xy + y4 = 253.Problema 6.Determine a, b ∈ R tais que (1, 1) perten e a urva denida impli itamente porx2y + ay2 = b e que a reta tangente nesse ponto é 4x+ 3y = 7.1.5 Exer í ios de Integral1.5.1 Exer í ios de FixaçãoExer í io 1.Determine se é Verdadeiro (provando a armativa) ou Falso (dando um ontra-exemplo):(a) Se ∫ b

a

f(x) dx = 0, então f(x) = 0 para todo x ∈ [a, b].(b) Se f(x) ≤ 0 para todo x ∈ [a, b], então ∫ b

a

f(x) dx ≤ 0.( ) Se ∫ 3

0

h(x) dx = 9 e mudarmos o valor da função em x = 1 e em x = 2, a integralvai mudar de valor.Exer í io 2.Estude a Denição 25 da p.139 e o Lema 8 da p.139 e resolva.Sabendo que ∫ 2

−1

f(x) dx = 5,

∫ 2

−1

g(x) dx = −3 e ∫ 0

−1

f(x) dx = 7, al ule:(a) ∫ −1

2

f(x) dx; (b) ∫ 2

−1

(

f(x) + 2g(x))

dx; ( ) ∫ 1

1

g(sen(x2)) dx;(d) ∫ 2

0

f(x) dx; (e) ∫ 2

−1

(∫ 0

−1

f(s)g(t) ds

)

dt.(f) ∫ 2

−1

h(x) dx se h(x) =

f(x); x 6= 1;

5; x = 1.Exer í io 3.Considere a função f(x) representada na gura abaixo.x

y

f(x)

1 2 3 4 5

2

−1226.jul.2010

1.5. EXERCÍCIOS DE INTEGRAL 33Dena F (x) =

∫ x

0

f(s) ds. Usando a ideia de que a integral é área om sinal respondaaos seguintes itens.(a) Determine F (0), F (1), F (2), F (3).(b) Determine os intervalos onde F res e e de res e.( ) Determine os pontos de máximo e mínimo lo al de F .Exer í io 4.Estude o Teorema 24 da p.141 (TFC). Considere h(x) =

∫ x

2

(5− t)5

t4 + 6dt. De-termine:(a) h(2); (b) intervalos onde h res e e de res e; ( ) pontos de máximo e mínimolo al.Exer í io 5.Estude o Corolário 10 da p.141 (TFC). Sabendo que h(s) = g′(s) para todo

s ∈ R e que g(x) = Kex3

+Bx− C, determine ∫ 1

−1

h(s) ds.Exer í io 6.Cal ule:(a) ∫ 1

0

(2x3 − 3x2 + 5) dx; (b) ∫ 1

0

|y2 − 1| dy; ( ) ∫ (3x+ et − 7x sen t) dt.Exer í io 7.Estude a Seção 5.3 da p.144 (Integrais Impróprias). Sem al ular as integraisabaixo, es reva ada uma omo o limite de uma integral própria:(a) ∫ 5

−∞e−s2 ds; (b) ∫ 2

0

dx

log(5− x2); ( ) ∫ +∞

0

dy

1 + y4; (d) ∫ 2

1

dx

x10 − 1.Exer í io 8. Faça mudança de variáveis para provar que:(a) ∫ bc

ac

f(t) dt = c

∫ b

a

f(x) dx; (b) ∫ b+c

a+c

f(x− c) dx =

∫ b

a

f(u) du.Exer í io 9.Cal ule as seguintes integrais (por substituição):(a) ∫ 4√K − 3x dx; (b) ∫ 3x2 cos(x3) dx; ( ) ∫ log2(t)

tdt;(d) ∫ x

√3− 2x2 dx; (e) ∫ sen(θ)

cos(θ)dθ ; (f) ∫ cosx e5 senx dx.Exer í io 10.Cal ule as seguintes integrais (por partes):(a) ∫ x log x dx; (b) ∫ arctanx dx.Exer í io 11.Cal ule as seguintes integrais denidas:(a) ∫ 2

1

(3− 2x)4 dx; (b) ∫ +∞

log 3

e−x/4 dx;( ) ∫ π

π/4

sen(2θ) dθ; (d) ∫ +∞

1

1

s3ds.

34 CAPÍTULO 1. ENUNCIADOS DOS EXERCÍCIOS1.5.2 ProblemasProblema 1.Considere f(x) =

2x se 0 < x ≤ 2,

−1 se 2 < x ≤ 4,

5− x se 4 < x ≤ 5.

Determine:(a) ∫ 3

0

f(x) dx; (b) ∫ 3

2

f(x) dx; ( ) ∫ 5

1

f(x) dx.Problema 2.Estude o Lema 8 da p.139 e prove que:(a) se f(x) ≤ M , então ∫ b

a

f(x) dx ≤ M(b− a);(b) ∫ 6e

e

4 sen(ex + 5x2 + x) log x dx ≥ −20e.Problema 3.Considere g(x) =

∫ e2x

0

sen(t2) dt. Cal ule g′(x).Problema 4.Determine a equação da reta tangente ao grá o dey(x) = log(2 + sen(x2 − π)) +

∫ x

√π

cos(s2) dsno ponto (√π, log 2).Problema 5.Cal ule:(a) f ′(1) se f(y) =

∫ y

1

et3

dt

4

cos(1 + s2) ds;(b) g′(8) se g(y) =

∫ 5

y

(∫ x

8

log(t3 + 1) dt

)

dx.Problema 6.Determine para quais p > 0 as integrais abaixo são nitas.(a) ∫ +∞

1

dx

xp; (b) ∫ 1

0

dx

xp.Problema 7. (integral indenida)(a)∫ cos(

√k)√

kdk; (b) ∫ 3x√

1− 3 x2dx; ( ) ∫ x sen(3x+ 1) dx;(d) ∫ cosx sen x dx; (e) ∫ sen(

√t) dt; (f) ∫ e2x cosx dx;(g) ∫ sen(log x) dx; (h) ∫ e3

√s ds; (i) ∫ ex

1 + e2xdx.Problema 8. (integral denida)(a) ∫ 1

0

xe−x2

dx; (b) ∫ 1

0

se−3s ds; ( ) ∫ +∞

e

dt

t(log t)3dt; (d) ∫ +∞

0

se−s/2 ds;

1.5. EXERCÍCIOS DE INTEGRAL 35(e) ∫ log 3

0

ex√1 + ex dx; (f) ∫ 1

0

x

1 + x2dx; (g) ∫ 8

1

1

x2

1 +1

xdx.Problema 9. (integral om módulo)(a) ∫ 4

0

x2|x− 2| dx; (b) ∫ 2

1/2

| log s| ds; ( ) ∫ 2

−2

|es−1 − 1| ds.Problema 10.Determine y(x) sabendo que:(a) dy

dx=

2x+ 1√x

e y(1) = 0; (b) dy

dx= xex

2+1 e y(1) = e2.Problema 11. (Integrais Impróprias)(a) ∫ 16

0

dx4√x. (b) ∫ +∞

e

dx

x log3 x; ( ) ∫ 0

−∞

ex

3− 2exdx;Problema 12.Determine:(a) lim

x→+∞e−x2

∫ x

0

log(t9 + 3) dt;(b) uma função f tal que f(0) = 1 e que ∫ x

−π

e−sf ′(s) ds = 3x para todo x ∈ R.1.5.3 ExtrasExtra 1.Determine TODOS os valores de x ∈ R onde a função Si(x) = ∫ x

0

sen t

tdt possuipontos de máximo lo al.Extra 2.Determine a equação da reta tangente ao grá o de ada função no ponto indi ado:(a) f(x) = ∫ x

1

log(et + t− 1)et2

dx no no ponto x = 1;(b) h(x) = 7−∫ x

2

es

s2 + 1ds no ponto x = 2.Extra 3.Considere um móvel preso a uma mola e deslizando sobre uma superfí ie sem atrito(veja gura abaixo). Sua a eleração é dada por a(t) = Aω2 cos(ωt) ∀t ≥ 0 (onde A e ω são onstantes). No instante t = 0 o móvel está na posição x(0) = 0 e tem velo idade v(0) = 0.Determine a função x(t) que determina a posição do orpo ao longo do tempo.

x

A0−A

36 CAPÍTULO 1. ENUNCIADOS DOS EXERCÍCIOSExtra 4. Seja f(s) =

∫ s2

s

sen(t)

t2dt. Determine f ′(s).Extra 5.Considere F (x) =

∫ x

2

t2 − 1

t2 + 1dt. A he:(a) os intervalos nos quais F é res ente e onde é de res ente;(b) os intervalos nos quais o grá o de F possui on avidade para baixo e onde é para ima;( ) o valor de x onde F atinge um mínimo lo al e o valor onde atinge um máximo lo al.Extra 6. Sabendo que ∫ 3

−1

f(s) ds = 7 e ∫ 3

1

f(s) ds = 3 determine ∫ 0

−1

f(2x+ 1) dx.Extra 7.Determine a função y(θ) sabendo que:(a) dy

dθ= cos(5θ) + 3 e y(π) = 5π; (b) dy

dx=

1

2x+ 1e y(0) = 3.Extra 8. Suponha que um ponto move-se ao longo de uma urva y = f(x) no plano xy detal forma que a ada ponto (x, y) da urva a reta tangente tem in linação √

x+ 1. A he aequação da urva sabendo que ela passa pelo ponto (0, 1).Extra 9. (integral om módulo)(a) ∫ 4

0

|x2 − 3x+ 2| dx; (b) ∫ 3

−3

1 + |x| dx; ( ) ∫ 2

−2

x∣

∣x2 − 2x∣

∣ dx.Extra 10. (integral indenida)(a) ∫ x2 log x dx; (b) ∫ √x log x dx; ( ) ∫ x(log x)2 dx; (d) ∫ (cos(2x))2 dx;(e) ∫ ex

e2x + 2ex + 1dx; (f) ∫ e3 sen(x)+4 cos(x) dx; (g) ∫ ex cos(ex + 3) dx.Extra 11. (integral denida)(a) ∫ 1

1/2

e1/x

x2dx; (b) ∫ √

log 2

1

x3ex2

dx; ( ) ∫ 4

1

log x√x

dx;(d) ∫ π/2

0

cos θ√sen θ dθ; (e) ∫ 1

0

y2√1− y

dy; (f) ∫ 4π2

π2

sen√x√

xdx;(g) ∫ log 3

log 2

ex

ex + 4dx; (h) 1

0

t√t2 + 1

dt.Extra 12. (Integrais Impróprias)(a) ∫ +∞

1

log x

x2dx; (b) ∫ 2

−∞

dx

(4− x)2.

1.6. EXERCÍCIOS DE APLICAÇÕES DA INTEGRAL 37Extra 13.Use integração por partes para provar as fórmula de redução de integral:(a) Se Im =

xmex dx, então Im = xmex −mIm−1.(b) Se Im =

xm sen x dx e Jm =

xm cosx dx, então Im = −xm cosx + mJm−1 eJm = xm sen x−mIm−1.Obs: Existem outras fórmulas de redução para ál ulo de integrais. Veja os Desaos.1.5.4 DesaosDesao 1.Use integração por partes para provar as fórmula de redução de integral:(a) Se Im =

dx

(x2 + 1)m, então Im =

x

2(m− 1)(x2 + 1)m−1+

2m− 3

2(m− 1)Im−1;(b) Se Im =

senmx dx, então Im = − 1

msenm−1x cosx+

m− 1

mIm−2.Obs: Existem fórmulas de redução para ál ulo de integrais de potên ias inteiras do osseno, tangente, se ante.

1.5.5 ⋆Problemas (Integração por Frações Par iais)Problema 1.Cal ule ∫ +∞

−∞

dx

x2 + bx+ cse b2 − 4c < 0.Problema 2. Suponha que a 6= b e a 6= 0. Cal ule:(a) ∫ dx

(x− a)(x− b), (b) ∫ dx

x2(x− a).Problema 3. [Ha, p.252 Determine ∫

Ax+B

ax2 + 2bx+ cdx se:(a) ∆ = b2 − ac > 0; (b) ∆ < 0; ( ) ∆ = 0.

1.6 Exer í ios de Apli ações da Integral1.6.1 Exer í ios de FixaçãoExer í io 1.Esbo e o grá o e al ule a área da região delimitada por:(a) y = ex + 2, y = ex, x = 0, x = 5.(b) y = 0, y = cos(x), x = −π/2, x = π/2.Exer í io 2.Considere os grá os de y = f(x) e y = g(x) representadas na gura abaixo.227.jul.2010

38 CAPÍTULO 1. ENUNCIADOS DOS EXERCÍCIOSx

y

f(x)

g(x)

−1 1 2

3

6

Es reva uma (ou a soma de) integral(is) denida(s) que al ule a área delimitada por:(a) y = f(x) e y = g(x) para x ∈ [1, 2];(b) y = f(x) e y = g(x) para x ∈ [−1, 2];Exer í io 3.Considere os grá os de y = f(x) e y = g(x) do exer í io anterior. Es revauma (ou a soma de) integral(is) denida(s) que al ule o volume do sólido de revolução obtidopela rotação em torno do:(a) eixo x da região delimitada por y = 0 e y = f(x) para x ∈ [−1, 1];(b) eixo x da região delimitada por y = f(x) e y = g(x) para x ∈ [1, 2];( ) eixo y da região do item (b).Exer í io 4.Considere a região do plano delimitada por x = f(y) e x = g(y) indi ada nagura abaixo. Es reva uma integral que determine a área da região.x

y

x = f(y)

x = g(y)

3

−2

1 2

Exer í io 5. Suponha que Π(s) é o plano y = s em R3. Seja Ω ⊂ R3 um sólido ontidoentre os planos y = −2 e y = 4. Seja A(s) a área da interseção de Π(s) om Ω. Es revauma integral que determine o volume de Ω.Exer í io 6.Considere g : [1, 7] → R tal que −4 ≤ g(x) ≤ 5 para todo x ∈ [1, 7]. Sabendoque o valor médio de g no intervalo [1, 7] igual a K, prove que −4 ≤ K ≤ 5.

1.6. EXERCÍCIOS DE APLICAÇÕES DA INTEGRAL 391.6.2 ProblemasProblema 1.Cal ule as áreas ha huradas das guras (a) e (b) abaixo. x

y

x

y

x

y

x

y

x

y

x

y

x

y

x

y

x

y

x

y

x

y

PSfrag repla ements

00000000000 11111111111

y = x2y = x2y = x2y = x2y = x2y = x2y = x2y = x2y = x2y = x2y = x2

y = x− x2y = x− x2y = x− x2y = x− x2y = x− x2y = x− x2y = x− x2y = x− x2y = x− x2y = x− x2y = x− x2 x

y

x

y

x

y

x

y

x

y

x

y

x

y

x

y

x

y

x

y

x

y

x

y

x

y

x

y

x

y

x

y

x

y

x

y

x

y

x

y

x

y

x

y

PSfrag repla ements

y = sen(x)y = sen(x)y = sen(x)y = sen(x)y = sen(x)y = sen(x)y = sen(x)y = sen(x)y = sen(x)y = sen(x)y = sen(x)y = sen(x)y = sen(x)y = sen(x)y = sen(x)y = sen(x)y = sen(x)y = sen(x)y = sen(x)y = sen(x)y = sen(x)y = sen(x)

y = cos(x)y = cos(x)y = cos(x)y = cos(x)y = cos(x)y = cos(x)y = cos(x)y = cos(x)y = cos(x)y = cos(x)y = cos(x)y = cos(x)y = cos(x)y = cos(x)y = cos(x)y = cos(x)y = cos(x)y = cos(x)y = cos(x)y = cos(x)y = cos(x)y = cos(x)(a) (b)Problema 2.Esbo e e al ule a área da região limitada:(a) y − x = 6, y − x3 = 0 e 2y + x = 0. Di a: x = 2 é raiz de x3 = x+ 6.(b) por y2 = 2x+ 4 e por y = x− 2.( ) inferiormente por y =√x, superiormente por y = 1 e lateralmente por x = 0.Problema 3.Considere a região do plano limitada superiormente por y = 4 +

√16− x2 einferiormente por y = 4 e y = 6 − x2, onforme indi ada na gura abaixo. Determine suaárea.

x

y

y = 4

y = 6− x2

y = 4 +√16− x2

Problema 4.Cal ule o volume do sólido de revolução gerado quando a região ha hurada nagura abaixo é girada em torno do eixo x e do eixo y. x

y

x

y

x

y

x

y

x

y

x

y

x

y

x

y

x

y

x

y

x

y

x

y

x

y

x

y

x

y

x

y

x

y

x

y

x

y

x

y

x

y

x

y

PSfrag repla ements

0000000000000000000000 1

1

1

1

1

1

1

1

1

1

1

1

1

1

1

1

1

1

1

1

1

1

1

1

1

1

1

1

1

1

1

1

1

1

1

1

1

1

1

1

1

1

1

1

2222222222222222222222

Problema 5.Esbo e a região do plano, determine sua área e al ule o volume do sólido derevolução obtido pela rotação em torno do eixo x da região do plano delimitada:

40 CAPÍTULO 1. ENUNCIADOS DOS EXERCÍCIOS(a) por y = 3√x, y = 2 e x = 0;(b) a ima por y = e−x, abaixo pelo eixo x e a esquerda pela reta x = 1 (uma regiãoinnita).Problema 6. (sólido de revolução girado em torno de outros eixos) Determine o volume dosólido de revolução obtido quando a região limitada por x = y2 e x = y é girada em tornoda reta:(a) y = −1; (b) x = −1.Problema 7.Na gura abaixo, seja A o ponto de interseção da urva y = ex

2 om a reta L,e seja B o vérti e da parábola 4y = (x− 2)2. Suponha que a reta L passe por A e B. A retaL, a parábola e o grá o de y = ex

2 delimitam uma região Ω. Es reva uma soma de integraisque determine o volume do sólido de revolução obtido ao girar Ω em torno do eixo y.x

y

y = ex2

4y = (x− 2)2

L

Ω

1

1

B

A

Problema 8.Um bura o ilíndri o de raio a é feito passando pelo entro de uma esfera deraio r. Determine o volume do sólido (esfera om bura o no meio) remanes ente.Problema 9.Determine o volume do sólido uja base é o ír ulo (no plano xy) x2 + y2 = r2e ujas seções perpendi ulares ao eixo x são quadrados om um lado na base (no plano xy).Problema 10.Determine o volume do sólido uja base é limitada por y = x e y = x2 e ujasseções perpendi ulares ao eixo x são quadrados om um lado na base (no plano xy).Problema 11.Determine o valor médio das funções abaixo nos intervalos indi ados:(a) f(x) = x2 em [0, K]; (b) g(x) = sen(x) em [0, π].1.6.3 ExtrasExtra 1.Esbo e e al ule a área da região limitada por:(a) y = x2, y = 1/x, y = −2, x = −1 e x = 2.(b) y = 6πx− 3x2, y = cos(x)− 1( ) por y = x2 e y = 1− x2.

1.6. EXERCÍCIOS DE APLICAÇÕES DA INTEGRAL 41Extra 2.Esbo e e es reva integrais que al ulem a área da região limitada por:(a) y = x3 − x e y = sen(πx) om x ∈ [−1, 1].(b) y = x3 − 3x2 + 2x e y = 3x− x2 − 2 (interseção em x = −1, 1, 2).Extra 3.Neste exer í io vamos mostrar omo denir log para depois denir a exponen ial.Dena f(x) =

∫ x

1

dx/x. Fingindo que vo ê não sabe a primitiva de 1/x, somente mudandovariável, prove que f(ab)f(a) + f(b).Extra 4. Seja R a região do plano delimitada pelas urvas y = c− x2 e 2x2 − 2c para c > 0.(a) Esbo e R; (b) Determine c > 0 tal que a área de R seja igual a 32.Extra 5.Esbo e a região do plano e al ule o volume do sólido de revolução obtido pelarotação em torno do eixo x da região do plano delimitada:(a) a ima pelo grá o de f(x) = √

log(x)− 1

x, abaixo pelo eixo x e a esquerda por x = e(região innita).(b) por y = log(x), por y = 0 e para x ∈ [1, e2].Extra 6.Esbo e a região do plano e es reva integrais que al ulem o volume do sólido derevolução obtido pela rotação em torno do eixo x e em torno do eixo y da região do planodelimitada por:(a) y = 1/(x2 + 5), y = 0, x = 0, x = 2.(b) y = x/2 e y =

√x.( ) y =

√x, y = 6− x e y = 0.Extra 7.A base de um sólido é a região (do plano xy) limitada por y2 = 4x e a reta x = 9.Cada plano perpendi ular ao eixo x intersepta o sólido num quadrado om um lado na base(no plano xy). Cal ule seu volume.Extra 8.Uma alota esféri a é uma porção da esfera obtida através de um orte por umplano de uma esfera (veja gura abaixo) . Se o raio da esfera é r, a altura da alota é h e oraio da alota é a, determine o volume desta alota.PSfrag repla ements

42 CAPÍTULO 1. ENUNCIADOS DOS EXERCÍCIOSExtra 9.Considere o sólido de revolução gerado pela rotação da região limitada por y =√x+ 1, y = 0, x = 0 e x = 2 em torno do eixo x. Determine o valor de a tal que oplano x = a orta este sólido em duas partes de mesmo volume.Extra 10.Considere a elipse de equação x2

a2+

y2

b2= 1. Determine o volume do elipsoideobtido quando se gira esta elipse em torno do eixo x.1.6.4 DesaosDesao 1.Os eixos de dois ilindros, ada um om raio r se inter eptam fazendo um ânguloreto (veja gura abaixo). Determine o volume da região omum aos dois ilindros.PSfrag repla ements

Este é onhe ido omo sólido de Steinmetz. Di a: Considere planos paralelos ao planoque ontem os eixos.Uma generalização é dada na gura abaixo.PSfrag repla ements

Desao 2.Cal ule a área da lúnula (interseção de dois ír ulos), um de raio r e outro R, ujos entros estão afastados uma distân ia L. Assumimos que L,R, r > 0. Veja nas gurasabaixo ilustrações de lúnulas em inza:PSfrag repla ements PSfrag repla ements

1.6. EXERCÍCIOS DE APLICAÇÕES DA INTEGRAL 431.6.5 ⋆Problemas (Substituição Trigonométri a)Problema 1.Cal ule (por substituição trigonométri a):(a) ∫ dx√x2 − 1

; (b) ∫ √x2 − 1

xdx;

1.6.6 ⋆Problemas (Comprimento de Curvas no Plano)Problema 1.Determine o omprimento do grá o da função:(a) y = f(x) = log(x+√x2 − 1) para x ∈ [1, 2].(b) y = f(x) = log x para x ∈ [1, 2];( ) y = f(x) =

√16− x2 para x ∈ [0, 4].Problema 2.Deduza a fórmula do omprimento do grá o de y = f(x) para x ∈ [a, b].1.6.7 ⋆Problemas (Área de Superfí ie de Sólido de Revolução)Problema 1.Cal ule a área da superfí ie de revolução gerada pela rotação em torno doeixo x da urva:(a) y = x3 para x ∈ [0, 1]. (b) y = x2 para x ∈ [0, 1]. ( ) y = e−x para x ≥ 0.

44 CAPÍTULO 1. ENUNCIADOS DOS EXERCÍCIOS

Capıtulo 2

Respostas dos Exercıcios2.1 Limite2.1.1 Exer í ios de FixaçãoExer í io 1. (a) 3; (b) o limite não existe. Cal u-lando os laterais: limx→b−

f(x) = 6; limx→b+

f(x) = 1.( ) 5.não existe limite em c: o grá o possui umaquebra.Exer í io 2. (a) Verdadeiro. (b) Falso: é inter-valo entrado (x − (−2) = x + 2) em −2 omraio 1: (−4,−1). ( ) Falso para x < 0. Corretoé √x2 = |x|. (d) Falso: o limite é 4. O valor dafunção no ponto não importa para o ál ulo dolimite.Exer í io 3. (a) Falso. Tome f(x) = 4; x ≤ 3;

5; x > 3,então quando x → 3− o limite é 4. Assim, neste aso o limite não existe.(b) Falso. O limite quando x → 2− é 4 poisa existên ia do limite impli a na existên ia doslimites laterais ( om o mesmo valor).( ) Falso. Tome f(x) =

4; x 6= 2;

5; x = 2, entãoo limite quando x → 2 é 4 mas f(2) = 5.(d) Falso. Se o limite quando x → 3 existe,os laterais existem e assumem o mesmo valor.Exer í io 4. (a) lim

x→1−f(x) = 5, lim

x→1+f(x) = 7,

limx→1−

f(x) não existe.(b) todos limites são 5.( ) todos limites são 7.(d) limx→2−

f(x) = 7, limx→2+

f(x) = 9, limx→2−

f(x)não existe.(e) todos limites são 7.(f) todos limites são 9.022.jul.2010

Exer í io 5. (a) a função alterna entre 1, quandocos(x) > 0, e −1, quando cos(x) < 0. Nospontos onde cos(x) = 0 ela não está denida.

x

y

f(x) =cos(x)

| cos(x)|

−5π2 −3π

2−π

2π2

3π2

5π2

y = 1

y = −1

(b)x

yf(x) =

|x|

Exer í io 6.(a) Translação verti al de uma unidade dográ o de √x.

x

y

(a) y = 1 +√x

1

(b) Translação verti al de duas unidades dográ o de sen(x).45

46 CAPÍTULO 2. RESPOSTAS DOS EXERCÍCIOSx

y

(b) y = 2 + senx

1

2

3

( ) translação horizontal do log por uma uni-dade seguido por translação verti al de duas uni-dades (faça duas guras antes de obter a respostaabaixo).x

y

( ) y = log(x− 1) + 2

1 2

2

(d) Translação horizontal do grá o de−1/x3,que é pare ido om o grá o de −1/x.x

y

(d) y =−1

(x+ 2)3y = −2

−2

(e) Raízes do polinmio: −1,−2. Esbo e ográ o da parábola (x+1)(x+2) e depois reitaem torno do eixo x (efeito do módulo).

x

y

−2 −1(e) y = |(x+ 1)(x+ 2)|(f) Esbo e o grá o da parábola ex, transladeverti almente em 2 unidades e depois reita emtorno do eixo x (efeito do módulo). A hamos oponto de reexão resolvendo ex − 2 = 0, o queimpli a que x = log(2).x

y

(f) y = |ex − 2|

log(2)y = −2Exer í io 7. (a) −1 ( an ele termos iguais). (b)1/2 ( an ele x no numerador e denominador). ( )0 (somente numerador se anula).Exer í io 8.Dizemos que o limite de f(x) quandox tende a +∞ é +∞ se f(x) a tão grande epositivo quanto quisermos para todo x grande osu iente.Exer í io 9. (a) Análise simples de dois termosquadráti os. Será positiva em [−

√3,−1) e em

(1,√3]. (b) O termo x3 − 1 possui a raiz 1.Pelo Teorema D'Alembert pode ser fatorado por

x − 1. Fazendo divisão de polinmios obtemosque x3 − 1 = (x − 1)(x2 + x + 1). Cal ulandoDelta, vemos que o segundo polinmio possui 2raízes omplexas. Como a > 0, o termo x2+x+1 ≥ 0. Fazendo quadro de sinais om x− 1, x ex2−4 (podemos ignorar o termo sempre positivox2+x+1) obtemos que será negativa em (−2, 0)e [1, 2).Exer í io 10. (a) Raízes são −3, 1, 2.

−3 1 2

x− 2 − − − +

x+ 3 − + + +

1− x + + − −0 0 0

p(x) + − + −

2.1. LIMITE 47x

y

−3 1 2

(a) p(x) = (x− 2)(x + 3)(1 − x)(b) Raízes são −1, 2.−1 2

(x− 2)2 + + +

x+ 1 − + +

0 0q(x) − + +

x

y

−1 2

(b) q(x) = (x− 2)2(x+ 1)( ) Raízes são 2, 3, 5.2 3 5

3− x + + − −(x− 2)2 + + + +

x− 5 − − − +

0 0 0r(x) − − + −

x

y

2 3 5

( ) r(x) = (3− x)(x− 2)2(x− 5);Exer í io 11. (a) −∞. (b) +∞. ( ) −1. (d) (afunção vale x2 para x > 0 e −x2 para x < 0)0. (e) não existe pois depende de qual lado seaproxima. (f) −∞ (0 + 1/0− = 0−∞ = −∞).(g) +∞.Exer í io 12. (a) 1. (b) +∞. ( ) 6. (d) +∞.(e) +∞. (f) +∞. (g) 3. (h) 5/4. (i) +∞.Exer í io 13. (a) Falso. Se q(x) = x−1 o limitenão existe; se q(x) = −(x− 1)2 o limite é −∞.(b) Falso. Se f(x) = q(x) então o limite será1. ( ) Verdadeiro. O denominador vai para −1.Assim, 0/(−1) = 0 (não é indeterminação).Exer í io 14.A ondição (i) ex lui a letra (b).Tanto (iii) quanto (iv) ex lui letra (d). Final-mente a letra ( ) não representa uma função:qual valor de f(0.99999)? São três possibilida-des: logo não é função. Resposta: (a).Exer í io 15.

x

y

(a) 1

1

2

−2

x

y

(b) 1

1

2

48 CAPÍTULO 2. RESPOSTAS DOS EXERCÍCIOSx

y

( ) 1

1

2

Exer í io 16. (a) omo seno é limitado por ±1,temos que −√

|x| ≤√

|x| sen(1/x) ≤√

|x|.Apli ando o Teorema do Sanduí he, on luímosque o limite é 0.(b) substituindo variável, o limite é 3. ( )substituindo variável, o limite é e5. (d) −∞. (e)e−2 (fazendo y = −2x).Exer í io 17. (a) É falso. O limite pode não exis-tir. Por exemplo g des ontínua em x = 3/2:g(x) = 1 para x ≤ 3/2 e g(x) = 2 aso ontrá-rio. (b) Como −1 ≤ cos(y) ≤ 1,

− 1

x2≤ cos(

√x2 + 1)

x2≤ 1

x2.Assim, pelo Teorema do Sanduí he, omo

limx→+∞

−1

x2= lim

x→+∞1

x2= 0,

limx→+∞

cos(√x2 + 1)

x2= 0.2.1.2 ProblemasProblema 1.

x

y

−3 3

−3(a) f(x) = −√9− x2; |x| ≤ 3

|x| − 3; |x| > 3.

x

y

1

1

(b) f(x) = √x− 1; x ≥ 1;

log(x) + 1; x < 1.Problema 2. (a) e (b) o limite é 0. Em ( ) olimite não existe pois os ila entre 0 e 1.Problema 3. (a) 1 (ra ionalize o numerador). (b)4 (note que para x próximo de 4, |x| = x e ra i-onalize). ( ) −1/2 (ra ionalize).Problema 4. (a) não existe pois o valor os ilaentre 1 e −1.(b) −∞.( ) para x > 2, omo |x− 2| = x− 2, an e-lamos os termos e a função é x+ 1. para x < 2, omo |x − 2| = 2 − x obtemos que a função é−(x+1). Assim para x → 2+ o limite é 2+1 = 3;para x → 2− o limite é −(2 + 1) = −3. Logo olimite não existe.(d) o limite não existe.Problema 5. (a) −∞. (b) −∞. ( ) −∞ ( 1x −1x2 = x−1

x2 ). (d) 1 (para x → 2−, |x − 2| =2 − x. (e) −1 (para x → −2, |x| = −x). (f) 4(note que 2 é raiz dupla: a3 − 5a2 + 8a − 4 =(a − 1)(a − 2)2). (g) 0 (o limite é 0/3 = 0).(h) −2. (i) 9/7 (trivial). (j) 3 (rearrumando onumerador obtemos (x2 + x − 2)/x). (k) 0. (l)3 (x3 + 1 = (x+ 1)(x2 − x+ 1)). (m) 1.Problema 6. (a) −1 (para x pequeno, numeradorvale √

x2 = −x). (b) −2/3. ( ) √10/5;(d) +∞. (e) +∞ (para x pequeno, vale−3y3/(

√10y2)). (f) sen(−2) (para x pequeno,numerador vale 4

√x6 = −4x3).Problema 7. (a) eab (mude variável para y =

ax). (b) 0. ( ) se b < 0 obtemos+∞ (+∞+∞).O aso interessante é se b > 0 (+∞−∞). Nesta aso, se c > b2 o limite é +∞, se c < b2 olimite é −∞, se c = b2 o limite é a/(2b). (d)a−b2√c(ra ionalizando). (e) b−a

2√c(ra ionalizando: uidado que aqui √x2 = −x!).Problema 8. (a) 0. (b) b/a. ( ) +∞ se c > 0,(note que 2m > m + 2 se m > 10) . (d) a/c.(e) √c/a. (f) 0.Problema 9. (a) quando x → 0− é 1, quando

x → 0+ é 0.

2.1. LIMITE 49(b) para x > 0 a função vale 1/x− 1/x = 0,para x < 0 vale 1/x − (−1/x) = 2/x. Assimquando x → 0+ é 0, quando x → 0− é −∞.Problema 10.Assintotas verti ais: x = −2 ex = 4. Assintota horizontal: y = 4.

x

y

−3 3

y = 4

x = 4x = −2Problema 11. (a) É uma pegadinha, pois pode-mos simpli ar a função para (x+1)(x−1)/(x−1) = x+ 1 para x 6= 1 (função não esta denidano 1). Assim a função é a reta y = x + 1 massem estar denida em x = 1.

x

yy = x+ 1

(a) y =x2 − 1

x− 1

−1 1

2

(b) O sinal da função é dado pelo denomina-dor, já que o numerador é sempre positivo (igual a1). O sinal é: |x| > 1 a função é positiva, |x| < 1é negativa. Assintotas verti ais (quando denomi-nador se anula): x = ±1. A assíntota horizontalé y = 0 (o eixo x) pois o no ±∞ é 0.x

y

(b) y =1

x2 − 1

x = 1x = −1( ) Como o denominador é sempre positivo

(x2 + 1 > 0 para todo x), o sinal da função éo mesmo do numerador: positiva para x > 0 enegativa para x < 0. Como o denominador nun ase anula, não possui assintotas verti ais. Como olimite no ±∞ é 0, possui assintota horizontal y =0 (eixo x). A função passa no (0, 0). Note queela tem que ser positiva para x > 0 e onvergirpara 0 no +∞. Com estas informações zemoso esboço mais simples possível.

x

y

( ) y =x

x2 + 1

(d) Assintotas verti ais (denominador se anula):x = 0 e x = 2. Assíntotas horizontais (limite no±∞): y = 1. Fazendo o quadro de sinais obte-mos o omportamento perto das assintotas.

x

y

(d) y =x2 − 1

x(x− 2)

x = 2

y = 1

(e) Assintotas verti ais (denominador se anula):x = 2 e x = −2. Assíntotas horizontais (limiteno ±∞): y = −3. Fazendo o quadro de sinaisobtemos o omportamento perto das assintotas.

50 CAPÍTULO 2. RESPOSTAS DOS EXERCÍCIOSx

y

(e) y =3x2 − 3

4− x2

x = 2x = −2

y = −3−1 1

Problema 12.Para (a) e (b). O grá o de h(x)é formado por duas retas pontilhadas: uma emy = x, a ima dos ra ionais e outra no y = −x,a ima dos irra ionais (vide gura abaixo). Logoem (a) e (b) o limite não existe.

x

yy = xy = −x

h(x)Para ( ) e (d): O grá o de h(x)/x2 é for-mado por duas retas pontilhadas: uma em y =1/x, a ima dos ra ionais e outra no y = −1/x,a ima dos irra ionais

x

y

h(x)

x2Logo em ( ) o limite não existe: nos ra ionaisvai para +∞, nos irra ionais para −∞. Em (d)o limite é 0.Em (e): O grá o de h(x)/x é formado por

duas retas pontilhadas: uma em y = 1, a imados ra ionais e outra no y = −1, a ima dos irra- ionaisx

y

y = 1

y = −1

h(x)

xLogo em (e) o limite não existe.Problema 13. (a) Pelo Teorema do Sanduí he olimite é 0.(b) quando x → 5, |f(x) − 3| → 0. Logof(x) → 3.Problema 14. (a) 4. (b) 3 (troque variável paray = 1/x2). ( ) 1/3 ( oloque o cos em evidên ia).(d) 2/5. (e) 0 (use Teorema do sanduí he e limiteo seno ompli ado por ±1). (f) e−10; (g) Troquevariável para y = π−x. Assim, x = π−y. Assimcos(π − y) = cos π cos(−y) + sen(−y) senπ =

− sen y. Pelo limite fundamental, limy→0

− sen y

y=

−1.(h) Pelo limite fundamental e pela deniçãode módulo, dará 1 se x → 0+ e −1 se x → 0−.2.1.3 ExtrasExtra 1.(a) Come e om o grá o de sen e faça ree-xão em torno do eixo x obtendo grá o de | sen |.Depois faça translação verti al por uma unidade.x

y

π2

3π2

π2−π

2−3π2 (a) y = | sen(x)| − 1(b) Come e om y = |x|. Faça translaçãoverti al de uma unidade. Reita o grá o no eixo

x novamente.x

y

(b) y = ||x| − 1|

2.1. LIMITE 51( ) Come e transladando horizontalmente ográ o de módulo por duas unidades. Depoistranslade verti almente por uma unidade.x

y

( ) y = |x+ 2| − 1

−2

Extra 2.x

y

x = 1

y = 4

1

−2

Extra 3.Como sengr(x) = sen(πx/180) (assimsengr(90) = sen(π90/180) = sen(π/2), substi-tuindo variável obtemos que o limite vale π/180.Extra 4. (a) Para x > 0, y = x+ |x| = x+ x =2x, para x < 0, y = x + |x| = x + (−x) = 0.Assim o grá o é:

x

yy = 2x

(a) y = x+ |x|(b) Por denição, x ≥ ⌊x⌋. Além disso adiferença 0 ≤ f(x) = x−⌊x⌋ < 1. Por exemplo,para x no intervalo [0, 1), ⌊x⌋ = 0 e portantof(x) = x − 0 = x. Para x no intervalo [1, 2),⌊x⌋ = 1 e portanto f(x) = x − 1. Para x nointervalo [−1, 0), ⌊x⌋ = −1 e portanto f(x) =x− (−1) = x+ 1. Assim o grá o é:

x

y

(b) y = x− ⌊x⌋−2 −1 1 2

y = 1

Extra 5. (a) Não existe pois quando x → 1+ vale1, x → 1− vale −1. (b) +∞. ( ) 8/6 = 4/3.(d) 0 (aplique Teorema do Sanduí he e use queseno é limitado por 1 e −1).Extra 6. (a) 0; (b) 3 (para x grande, 2x+ |x| =2x + x = 3x). ( ) 1 (para x pequeno, 2x +|x| = 2x − x = x). (d) −∞ (para x pequeno,x+ |x|+ 1 = x− x+ 1 = 1).Extra 7. (a) 0. (b) 0.Extra 8.

x

y

y = 1

y = 2

(a) f(x)x

y

y = x

y = x2

(b) g(x)2.1.4 DesaosDesao 1. (a) Para x grande (basta que x > 1)temos que 0 < 1x < 1. Assim para x > 1, ⌊ 1x⌋ =

0. Logo o limite vale zero pois a função vale zeropara x > 1.(b) Para x pequeno (basta que x < −1) te-mos que −1 < 1x < 0. Assim para x < −1,

⌊ 1x⌋ = −1. Logo o limite vale −∞ pois a funçãovale −x para x < −1.

52 CAPÍTULO 2. RESPOSTAS DOS EXERCÍCIOS

x

y

y = 1

y = −x

y = 1− x11

213

−1 −12( ) y = x⌊ 1x⌋(d) Vamos utilizar o Teorema do Sanduí he.Para x > 0, observe que ⌊ 1x⌋ vale no máximo

1x e, no mínimo 1

x − 1. Logo, 1x − 1 ≤ ⌊ 1x⌋ ≤

1x . Multipli ando ambos os lados por x (que épositivo e não altera as desigualdades) obtemosque 1 − x ≤ x⌊ 1x⌋ ≤ 1. Passando ao limite x →0+ e apli ando o Teorema do Sanduí he obtemosa onvergên ia para 1.Para x < 0 fazemos um ra io ínio similar para on luir que 1 ≤ x⌊ 1x⌋ ≤ 1 − x. Passando ao li-mite x → 0− e apli ando o Teorema do Sanduí heobtemos a onvergên ia para 1. Como os limiteslaterais são iguais, o limite existe e é igual a 1.Desao 2. (a) Note que trata-se de uma indeter-minação do tipo ∞0. Em uma linha: ex res emuito mais rápido do que x. Assim para x grande,(ex + x)1/x ≈ (ex)1/x = e. Com rigor, olo-que ex em evidên ia: (ex(1 + x/ex))1/x = e(1 +x/ex)1/x. Agora o termo (1 + x/ex) → 1 e1/x → 0. Assim (1 + x/ex)1/x → 10 = 1.(b) Note que trata-se de uma indeterminaçãodo tipo ∞0. Em uma linha: Como em (a), para xgrande (1 + x) ≈ x. Assim temos que al ular olimite xα/ log x. Troque variável para y = log(x)(assim x = ey): al ule o limite (ey)α/y → eα.Desao 4.Embora o denominador se anule nolimite, o sinal dele alterna sempre. Assim o nolimite a função os ila entre +∞ e −∞. Pertodo zero teremos um innidade de pontos onde afunção se aproxima de ±∞.Desao 7.Como |f(x)| ≤ M , −M ≤ f(x) ≤M . Logo, −M |g(x)| ≤ f(x)g(x) ≤ M |g(x)|.Como lim

x→1g(x) = 0, lim

x→1|g(x)| = 0. Apliqueo Teorema do Sanduí he depois de veri ar que

limx→1

−M |g(x)| = limx→1

M |g(x)| = 0.Desao 8. (a) +∞; (b) +∞; ( ) +∞; (d) +∞;(e) 02.2 Continuidade2.2.1 Exer í ios de FixaçãoExer í io 1. (a) Falso. O limite deve ser igualao valor da função no ponto. Exemplo: f(x) =x

x; x 6= 0;

2; x = 0;O limite no zero é 1 mas f(0) = 2.(b) Verdade. Se f é ontínua o limite existe.Se o limite existe, ambos limites laterais existem.( ) Falso. O limite pode ser igual, omo no ontraexemplo do item (a) deste exer í io.Exer í io 2. Somente é ontínua em a. Em b e

d, embora o limite exista, ele difere do valor dafunção no ponto: o grá o possui um salto. Emc, os limites laterais existem mas diferem entre si.Assim não existe limite em c: o grá o possuiuma quebra.Exer í io 3. (a) ontínua. (b) des ontínua no 0.Exer í io 4. (a) f(x) =

1; x ≤ 1;

2; 1 < x < 2;

3; 2 ≤ x.(b) A função parte inteira (veja Figura nap. 2.1.3) para x > 0: f(x) =

0; x ≤ 0;

⌊x⌋; x > 0.Exer í io 5. (a) não existe valor possível pois oslimites laterais são distintos: a des ontinuidadenão é removível.(b) Como o limite é +∞, que não é um nú-mero, não existe k. Se pudéssemos olo ar valorinnito, olo aríamos k = +∞.( ) Pelo Teorema do Sanduí he o limite quandox → 0 é zero. Assim oloque k = 0 para tornara função ontínua.Exer í io 6. (a) Falso. Pode ter. Basta os ilarentre estes pontos.(b) Verdadeiro: pelo menos uma em [2, 3] epelo menos uma e, [3, 4], onde a função tro a desinal.( ) Falso. O TVI garante pelo menos uma,mais pode ter mais de uma.027.mar.2010

2.2. CONTINUIDADE 53Exer í io 7. (a) Falso. Quando nas e uma ri-ança a função dá um pulo de uma unidade ins-tantaneamente: não existe 1/5 de habitante et .(b) Verdadeiro. Nos res emos diariamenteuma quantidade innitamente pequena. Nossaaltura não dá pulos.Exer í io 8. (a) Falso. Nada impede que f(1/2) =−10. Neste aso teríamos vários pontos om va-lor negativo.(b) Falso. Se f for des ontínua pode não terraiz.Exer í io 9. (a) Errado. O orreto é se K ∈[2, 5], então existe c ∈ [−3,−1] tal que f(c) =K; (b) Correto pois se K ∈ [3, 4] então K ∈[2, 5]. Logo, pelo TVI, existe c ∈ [−3,−1] talque f(c) = K.( ) Errado. O intervalo [0, 3] NO está on-tido em [2, 5].Exer í io 10.Como f é ontínua, pelo Teorema,f ·f = f2 (produto de funções ontínuas) é ontí-nua. Assim, pelo Teorema novamente, f · (f2) =f3 (produto de funções ontínuas) é ontínua.Também pelo Teorema, 5f3 ( onstante vezes fun-ção ontínua) é ontínua. Pelo Lema, x é ontí-nua. Pelo Teorema x ·x = x2 produto de funções ontínuas) é ontínua. Pelo Lema 1 (função ons-tante) é ontínua. Pelo Teorema, x2 + 1 (somade funções ontínuas) é ontínua. Finalmente,pelo Teorema h, que é o quo iente de funções ontínuas, é ontínua.2.2.2 ProblemasProblema 1. (a) Nos pontos onde o denominadorse anula f(x) → ±∞. Nestes pontos a funçãoé des ontínua. Nos outros pontos, omo se tratada divisão de funções ontínuas (1 e sen(x)), elaé ontínua. R: 0,±π,±2π,±3π,.(b) O denominador nun a se anula pois cos(x)vale no mínimo −1: assim 2 − 1 = 1 ≤ 2 +cos(x) ≤ 2 + 1 = 3 para todo x ∈ R. Portanto, omo g é quo iente de funções ontínuas om de-nominador que nun a se anula, g é ontínua emR e o onjunto dos pontos de des ontinuidade é∅ (vazio).( ) Veja o grá o na Figura da p. 2.1.3. O onjunto dos pontos de des ontinuidade é Z.(d) Esbo e o grá o: uma úbi a pontilhadae uma reta pontilhada. É ontínua onde elas se ruzam (porque?) nos pontos onde x3 = x, isto

é, em x = 0, x = 1 e x = −1. É des ontínua emR− 0, 1,−1.Problema 2.Cal ulando os limites no 0:lim

x→0−f(x) = |0+2| = 2, lim

x→0+f(x) = 3−0 = 3.Como eles diferem no 0, não existe lim

x→0f(x) eportanto a função é des ontínua no 0. Nos outrospontos é ontínua.Para x grande e negativo, f(x) = |x − 2|.Assim lim

x→−∞f(x) = lim

x→−∞|x− 2| = +∞.Problema 3. (a) Note que f(0) = 0 < 10 e que

limx→+∞

f(x) = +∞ (veja Exemplo 43 da p.54).Logo existe M > 0 tal que f(M) > 10. PeloTVI existe c ∈ [0,M ] tal que f(c) = 10.(b) Dena h(x) = log(x) − e−x. Queremosen ontrar b > 0 tal que h(b) = 0. Quandox → 0−, log(x) → −∞ e e−x → 1. Logo,lim

x→0−h(x) = −∞. Quando x → +∞, log(x) →

+∞ e e−x → 0. Logo, limx→+∞

h(x) = +∞. As-sim existem M,N om 0 < M < N e tais queh(M) < 0 e h(N) > 0. Como h é ontínua, peloTVI existe d ∈ [M,N ] tal que h(b) = 0.( ) Dena g(x) = f(x) − x. Se g(c) = 0,então f(c) = c. Note que g(0) = f(0) − 0 =f(0) ≥ 0 e g(1) = f(1) − 1 ≤ 0. Se em umdos extremos g se anular nos teremos obtido oc. Caso ontrário, g(1) < 0 < g(0). Pelo TVI(g é ontínua pois é a subtração de duas funções ontínuas), existe c ∈ [0, 1] om g(c) = 0. Esteresultado é uma versão simpli ado do Teoremado Ponto Fixo de Brower.(d) Suponha, por ontradição, que não é ver-dade que f(x) < 0. Assim, existiria um t ∈ [0, 2] om f(t) ≥ 0. Como f não se anula em [0, 2], naverdade f(t) > 0. Como f(−1) = −3, apli andoo TVI em [1, t] (f é negativa em 1 e positiva emt) on luímos que existe um c ∈ [1, 2] tal quef(c) = 0. Como isto é um absurdo, on luímosque f(x) < 2 no intervalo [0, 2].Problema 4. (a) Simplique o (x−2)2 no nume-rador e denominador. a = 5.(b) Impossível. Teríamos que ter a = 3 e −2ao mesmo tempo.( ) a = 1.(d) Impossível pois o limite em x = 0 nãoexiste.(e) Impossível pois teríamos que ter a = +∞,que não é um número real.(f) a = 3/4.

54 CAPÍTULO 2. RESPOSTAS DOS EXERCÍCIOSProblema 5.Temos que resolver o sistema

2a+ b = |2− 1| = 1,−2a+ b = | − 2− 1| = 3.Obtemos a = −1/2, b = 2.2.2.3 ExtrasExtra 1.Ela somente é ontínua em x = 0 poisse x está próximo de 0 e x ∈ Q então f(x) = 1e se x 6∈ Q então f(x) ≈ 1 + |0| = 1. Logo olimite quando x → 0 é 1, que é igual ao valor dafunção. Logo é ontínua em x = 0.Em qualquer x 6= 0 o limite não existe pois seestá próximo de x 6= 0 e x ∈ Q então f(x) = 1e se x 6∈ Q então f(x) ≈ 1 + |x| 6= 1. Logoo onjunto dos pontos de des ontinuidade é R−

1.Extra 2. (a) Dividindo-se por (x− 1) duas vezeso numerador e o denominador, vamos obter o li-mite. Logo a = −1. (b) a = 2. ( ) Impossível.(d) a = 0. (e) a = −1 +√2 ou a = −1 −

√2.(f) Impossível. Geometri amente, um reta saindoda origem não temo omo ompletar de forma ontínua uma função que valia 1 para x < 0.Extra 3. (a) Falso. Pode ter raiz no meio (penseem algo do tipo seno, que os ila).(b) Não. O TVI garante pelo menos duasraízes, mas não exatamente duas.Extra 4. (a) Dena f(x) = x + 2 sen(x) − 1.Como f(0) = −1 e f(π) = π − 1 > 0, pelo TVI

f possui raiz.(b) Se o polinmio p é de grau impar omtermo de maior grau axk então, se k > 0,lim

x→−∞p(x) = −∞ e lim

x→+∞p(x) = +∞. Assimexistem M,N tais que p(M) < 0 e p(N) > 0.Como p é ontínua, pelo TVI existe c ∈ [M,N ] ⊂

R tal que p(c) = 0 ∈ [p(M), p(N)]. Se k < 0então (os limites se invertem) limx→−∞

f(x) = +∞e limx→+∞

f(x) = −∞ e o resto é análogo.( ) Dena h(x) = sen(π sen(x)) − sen(x).Como h(π/6) = 1/2 e h(π/2) = −1 (sinaisopostos), pelo TVI existe c ∈ [π/6, π/2] tal queh(c) = 0, isto é, tal que sen(π sen(c)) = sen(c).(d) Como h(0) = 1, h(π) = h(−π) = 1−2 =−1, apli ando o TVI nos intervalos [−π, 0] e [0, π]vamos obter duas raízes distintas para h.Extra 5.Temos que resolver o sistema

a+ b = 1,4a+ b = −8.

Obtemos a = −3, b = 4.2.2.4 DesaosDesao 1.Esbo e dois grá os olo ando no eixox a hora e no eixo y os pontos do per urso. Noprimeiro dia a função omeça do iní io do per- urso e termina no m. No dia seguinte, omeçano m do per urso e termina no iní io. Comoos per ursos são funções ontínuas, os grá os se ruzam em pelo menos um ponto, o que signi- a passar na mesma hora (em dias distintos) nomesmo ponto do per urso.Desao 2.Ver [Sp p.70 no.17.Desao 3.Ver [Sp p.98 no.6.Desao 4.Pelas propriedades do logaritmo, o-lo ando a em evidên ia,log(a+h) = log(a(1+h/a)) = log(a)+log(1+h/a).Quando h → 0, tro ando variável vemos quelog(1 + h/a) → log(1) = 0. Assim obtemos a ontinuidade de log.Desao 5.Basta apli ar as expansões em série daexponen ial, seno e osseno. Depois basta agru-par os termos om e sem i e utilizar as identi-dades: i0 = i4 = i8 = · · · = 1, i1 = i5 =i9 = · · · = i, i2 = i6 = i10 = · · · = −1,i3 = i7 = i11 = · · · = −i. Assim,eiθ = 1 + (iθ) +

(iθ)2

2+

(iθ)3

3!+

(iθ)4

4!+

(iθ)5

5!· · ·

= 1 + iθ − θ2

2− i

θ3

3!+

θ4

4!+ i

θ5

5!· · ·

= 1− θ2

2+

θ4

4!− · · · + i

(

θ − θ3

3!+

θ5

5!· · ·)

= cos θ + i sen θ.Desao 6.Basta fazer ontas an elando a partereal ou a imaginária.2.3 Derivada2.3.1 Exer í ios de FixaçãoExer í io 1. y − 3 = (x− (−2)) · 3 = 3(x+ 2).Assim a reta tangente é y = 3x+ 9.Exer í io 2. (a) Falso. f(x) = |x− 3| possui umbi oem x = 3.025.julho.2010

2.3. DERIVADA 55(b) Falso. f(x) = 0 e g(x) = x − 2. Entãof(2) = g(2) = 0 mas f ′(2) = 0 e g′(2) = 1.( ) Falso. f(x) = x−10. f ′(1) = 1 e f(1) =−9.Exer í io 3. (a) Como no intervalo [x1, x3] a fun-ção é um segmento de reta, f ′(x1) = 2 = f ′(x2).Note que f ′(x3) não existe pois grá o possui umbi o.(b) Note que f ′(x5) = 0 ou algo próximo eque f ′(x6) > f ′(x2) pois a in linação da reta tan-gente é maior em x6. Também f ′(x4) < 0 poisa função de res e ai. Assim, f ′(x4) < f ′(x5) <f ′(x2) < f ′(x6).Exer í io 4.

x

y

f ′(x)

4 12−5

8Exer í io 5. (a) h′(2) = f ′(2)g(2)+f(2)g′(2) =−1(−5) + 3(2) = 11.(b) h′(2) = f ′(2)g(2) − f(2)g′(2)

g(2)2.Logo h′(2) =

−1(−5)− 3(2)

(−5)2=

−1

25.Exer í io 6.Cal ulando o oe iente angular dareta tangente, f ′(1) = 1 e g′(1) = 0 (reta tan-gente horizontal). Assim:(a) f ′(1)g(1)+g′(1)f(1) = 1(3/2)+0(1) = 3/2.(b) 5f ′(1)− 3g′(1) = 5(1) − 3(0) = 5.Exer í io 7. (a) Velo idade é h′(t) = −32t. As-sim h′(2) = −64.(b) Quando h(t) = 0? Para t =√125/2.( ) Velo idade h′(

√125/2) = −16

√125.(d) A eleração h′′(t) = (−32t)′ = −32. As-sim a a eleração é −32 para todo t.Exer í io 8. (a) ex log x+ ex

x . (b) − sen x(x+5)−cos x(x+5)2

.( ) − sen(x3 + 1)(3x2). (d) 0 (a função é ons-tante em relação a x). (e) cos x

1 + senx. (f) Para

x > 2 a derivada é 1, para x < 2 a derivada é−1. Em x = 2 a derivada não existe.Exer í io 9. (a) 4πr2. (b) 6k+ 1

k2. ( ) log t+1.(d) πsπ−1. (e) √

3 = elog√3. Logo (

√3)x =

ex log√3. Assim a derivada é log(

√3)ex log

√3 =

(log√3)(

√3)x. (f) 0.Exer í io 10. (a) Pelo TVM, existe c ∈ [1, 5] talque f(5) − f(1) = f ′(c)4. Multipli ando por

4 a desigualdade −4 ≤ f ′(x) ≤ 3 obtemos oresultado.

( ) Pelo TVM, para todo h existe c ∈ [0, h] talque f(h)−f(0) = f ′(c)h. Como h > 0 podemosmultipli ar a desigualdade −4 ≤ f ′(x) ≤ 3 semalterar os sinais das desigualdades.Exer í io 11. Seja S(t) a altura do objeto emfunção do tempo. Então S(0) = 100, S(5) = 0.Assim, S(5)− S(0)

5− 0=

−100

5= −20. Pelo TVMexiste um instante t ∈ (0, 5) tal que S′(t) = −20,a velo idade do objeto.Exer í io 12.Como (f ′(x))′ = 0 para todo x ∈

R, f ′(x) = onstante. Como f ′(−3) = 0, a ons-tante é zero. Assim on luímos que f ′(x) = 0para todo x ∈ R. Logo f(x) = onstante. Comof(5) = π, a onstante é π. Assim on luímosque f(x) = π para todo x ∈ R.Exer í io 13.(a) h′(2) = f ′(g(2))g′(2) = f ′(3)g′(2). Comog′(2) é o oe iente angular da tangente, g′(2) =(3−2)/(2−0) = 1/2. Do mesmo modo, f ′(3) =(0−2)/(3−0) = −2/3. Assim, h′(2) = −2

3 · 12 =−2

6 = −13 .(b) Como g(2) = 3, g−1(3) = k(3) = 2.Como k é a inversa de g, k(g(x)) = x. Logo,

k′(g(x))g′(x) = 1. Assim, k′(g(2))g′(2) = 1 ouk′(3)g′(2) = 1. Como g′(2) = 1

2 pelo item (a),k′(3) = 1/g′(2) = 2.Exer í io 14.Mar amos no grá o os pontos ondeele ruza o zero e onde a reta tangente é horizon-tal.

x

y

a b c d e f g(a) f ′ é positiva em (−∞, b), (c, d) e (f,+∞).f ′ é negativa em (b, c) e (d, f).(b) f é injetiva em (−∞, b), ou (b, c), ou(c, d), ou (d, f), ou (f,+∞).( ) f é res ente em (a, e) e (g,+∞). f éde res ente em (−∞, a) e (e, g).(d) f é injetiva em (−∞, a) ou (a, e) ou (e, g)ou (g,+∞).2.3.2 ProblemasProblema 1. (a) f(x + h) − f(x) = 1

(x+h)2 −1x2 = x2−(x+h)2

x2(x+h)2 = −2xh+h2

x2(x+h)2 . Dividindo por hobtemos f(x+h)−f(x)h = −2x+h

x2(x+h)2. Fazendo h →

0 obtemos, f ′(x) = −2xx2(x)2

= − 2x3 .

56 CAPÍTULO 2. RESPOSTAS DOS EXERCÍCIOS(b) f(x+h)−f(x) = 1√x+h

− 1√x=

√x−

√x+h√

x+h√x.Multipli ando por √x+

√x+ h obtemos:

x−(x+h)√x+h

√x(

√x+

√x+h)

= −h√x+h

√x(

√x+

√x+h)

.Dividindo por h obtemos:f(x+h)−f(x)

h = −1√x+h

√x(

√x+

√x+h)

.Quando h → 0 obtemos:f ′(x) = −1√

x√x(√x+

√x)

= − 12x

√x.Problema 2.Para garantir ontinuidade em x =

1 devemos ter: (1)2 = a(1) + b, ou a + b = 1.Para que as derivadas laterais sejam iguais emx = 1 devemos ter 2x = a em x = 1, ou a = 2.Assim b = 1− a = −1.Problema 3.Primeiro note que 0 ≤ |f(0)| ≤|0|k = 0. Assim |f(0)| = 0, isto é, f(0) = 0.Agora pela denição, f ′(0) = lim

h→0

f(h)− f(0)

h=

limh→0

f(h)

h.Observe que 0 ≤ |f(h)|

|h| ≤ |h|kh = |h|k−1.Como k > 1, k − 1 > 0. Assim, lim

h→0|h|k−1 = 0.Logo, pelo teorema do Sanduí he, lim

h→0

|f(h)||h| =

0. Logo f ′(0) = limh→0

f(h)

h= 0.Problema 4. (a) Possui derivada em todos ospontos x 6= 2 igual a zero pois é onstante. Em

x = 2 é des ontínua e portanto também não éderivável em x = 2.(b) f(x) = ex − 1 se ex − 1 > 0, isto é, seex > 1. Tomando log dos dois lados, se x > 0.Assim, f(x) = ex−1 se x > 0 e f ′(x) = ex. Poroutro lado, f(x) = −(ex− 1) = 1− ex se x < 0.Assim f ′(x) = −ex se x < 0. Em x = 0 o grá opossui um bi o e a função não é derivável.( ) Fazendo análise de sinal do polinmio dosegundo grau (3−x)(x+1) (parábola om raízes3 e −1 om on avidade para baixo), on luímosque h(x) = (3 − x)(x + 1) se −1 < x < 3 eh(x) = −(3 − x)(x + 1) aso ontrário. Assim,h′(x) = −2x + 2 se −1 < x < 3 e h′(x) =2x − 2 se x < −1 ou x > 3. Em x = −1 ex = 3 o grá o possui um bi o e a função nãoé derivável.Problema 5. (a) A velo idade é s′(t) = 4t

(t2 + 1)2.A a eleração é s′′(t) = 4(t2 + 1)2 − 16t2(t2 + 1)

(t2 + 1)4.Logo s′(0) = 0 e s′′(0) = 4. Ela vai parar quandoa velo idade s′(t) = 0, ou seja, quando t = 0.(b) A velo idade é s′(t) = cos t. A a eleraçãoé s′′(t) = − sen t. Logo s′(0) = 1 e s′′(0) = 0.

Ela vai parar quando a velo idade s′(t) = cos t =0, ou seja, quando t = 2kπ ± π/2 para k ∈ Z.Problema 6. (a) Nos pontos onde f ′(x) = 6x2−4x = 0, isto é, x = 0 ou x = 2/3.(b) Rees revendo a reta 2y − 20x − 50 = 0 omo y = 10x + 25, observamos que o oe i-ente angular é 10. Assim queremos saber quandof ′(x) = 6x2 − 4x = 10, isto é, x = −1, x = 5/3.Problema 7. (a) y′ = 3ax2 + 2bx+ c. Para quetenha uma úni a tangente horizontal, queremosque a equação y′ = 3ax2 + 2bx + c = 0 tenhasolução úni a. Para isto basta que ∆ = (2b)2 −4(3a)c = 0, isto é, que b2 = 3ac.(b) y′(x) = 3ax + b. O oe iente angularde x + y = 1 é 1 = y′(−1) = b − 3a. O oe- iente angular de y = −1 é 0 = y′(1) = 3a + b.Resolvendo o sistema obtemos que b = 1/2 ea = −1/6. Assim c pode ter qualquer valor.Problema 8.(a) 300(5x2 − 3x+ 4)299(10x− 3).(b) Primeiro rees reva 7

√· = (·)1/7. Depoisapli ando a regra da adeia,cos(

(

cos(x2) + 4)1/7

)

·

· 17

(

cos(x2) + 4)−6/7

(− sen(x2))(2x).( ) 2xe−x + 2x+ e−xx2 + e−x

(e−x + 1)2.(d) 1/3(x + t)−2/3(x2 + k)− (x+ t)1/3(2x)

(x2 + k)2.(e) 5 cos(5 ex)exx4

sen(5 ex)+ 4 log(sen(5 ex))x3.(f) 6x

((log(3x2 + 1))2 + 1)(3x2 + 1).(g) − 5earcsen(4−5x)

1− (4− 5x)2.Problema 9. g′(x) = 3

f(x) + x(f ′(x) + 1). As-sim, g′(4) = 3

f(4) + 4(f ′(4) + 1) = −12

7.Problema 10. (a) f ′(t) = −m0

K e(T0−t)/K .(b) f ′(t) = 2(ax+ b)(ad− bc)

(cx+ d)3.( ) f ′(θ) = 3aKθ2 cos(aθ3 + b).(d) f ′(t) = KeKt cos(at)− aeKt sen(at).Problema 11. (a) y′ = cos(x2) · 2x2 − sen(x2)

x2.Logo y′(

π/2) = −2/π e y(√

π/2) =√

2/π.

2.3. DERIVADA 57Assim a equação da reta tangente é: y−√2/π =−2/π(x−

π/2).(b) y′ = esen(−2x) cos(−2x)(−2).Logo y′(π) = −2 e y(π) = 1. Assim a equaçãoda reta tangente é: y − 1 = −2(x− π).Problema 12. (a) Considere f(x) = ex−(1+x).Derivando f ′(x) = ex − 1 é positiva para x > 0.Logo f é res ente para x > 0. Como f(0) = 0,a função é positiva para x > 0.(b) Considere g(x) = 2x3 − 15x2 + 60x+ 4.Como limx→+∞

g(x) = +∞ e limx→−∞

g(x) = −∞,existem pontos onde a função é positiva e ne-gativa. Pelo TVI existe pelo menos uma raiz.Note que g′(x) = 6x2 − 30x + 60 é sempre po-sitivo (para todo x ∈ R) pois é um polinmiodo segundo grau om raízes omplexas (∆ < 0 ea = 6 > 0). Assim, g é res ente para todo Re portanto injetiva. Assim a raiz é úni a pois afunção é injetiva.Problema 13. (a) Suponha que f e g represen-tam a posição dos orredores em função do tempo.Por hipóteses f(0) = g(0) ( omeçam no mesmoinstante). Suponha que eles terminaram a orridano instante T . Assim, f(T ) = g(T ) (terminaramempatados). Se h = f − g, h(0) = h(T ) = 0.Pelo TVM (ou Teorema de Rolle), existe c ∈(0, T ) tal que h′(c) = 0 = f ′(c)− g− (c), isto é,f ′(c) = g′(c).(b) Pelo TVM, f(x)− f(0) = f ′(c)x. Comox > 0 e f ′(c) ≤ 1 para todo c > 0 e f(0) = 0,f(x) = f(x)− f(0) ≤ x.( ) Seguindo a di a, omo h′i = hi para i =1, 2,

f ′(x) =h′1h2 − h1h

′2

(h2)2=

h1h2 − h1h2(h2)2

= 0.Logo f é onstante. Como f(0) =h1(0)

h2(0)= 1,

f(x) = 1 para todo x ∈ R. Logo 1 =h1(x)

h2(x), istoé, h1(x) = h2(x) para todo x ∈ R.(d) Seja h = f − g. Como h(0) = h(1) = 0,pelo Teorema de Rolle, existe c ∈ (0, 1) tal que

h′(c) = 0 = f ′(c) − g′(c). Logo f ′(c) = g′(c) eportanto as tangentes são paralelas.Problema 14.(a) h′(x) = f ′(g(−x/2))g′(−x/2)(−1/2). As-sim, h′(2) = f ′(g(−1))g′(−1)(−1/2) == f ′(2)(6)(−1/2) = −1(6)(−1/2) = 3.(b) Como h(g(x)) = x, h′(g(x))g′(x) = 1.Como g(−1) = 2, h(g(−1)) = −1 = h(2). As-sim h′(2) = h′(g(−1)) = 1/g′(−1) = 1/6.

Problema 15.O oe iente angular da reta tan-gente é ∆y

∆x=

3− 6

−1− 0= 3. Logo, f ′(−1) = 3.Note que f(−1) = 3 ou f−1(3) = −1. Logo

(f−1)′(3) =1

f ′(f−1(3))=

1

f ′(−1)=

1

3.Problema 16. f ′(x) = − sen(x5+2x+π/2)(5x4+

2). Logo f ′(0) = −2. Como g(f(0)) = 0 =

g(0), g′(0) = 1

f ′(0)= −1

22.3.3 ExtrasExtra 1. (a) w′ = 4f ′g − fg′

g2. Logo w′(2) =

4(−1)(−5) − 3(2)

(−5)2= − 4

25.(b) m′(x) = exg(3x + 2) + exg′(3x + 2)3.Logo m′(0) = g(2) + 3g′(2) = −5 + 3(5) = 10.Extra 2. (a) cos(x ex log x)(ex log x+xex log x+

ex).(b) cos(sen(senx)) cos(senx) cos x.( ) 3arctan x = elog 3 arctan x. Logo a derivadaé 3arctan x log 3

1 + x2.(d) 2

√x+ 1

4√x√

x+√x.(e) cos(cos x senx)(cos2 x− sen2x).(f) Esta função vale sen(1−x2) se −1 < x <

1. Logo a derivada neste intervalo é −2x cos(1−x2). Fora deste intervalo (em x < −1 ou x >1) a função vale sen(x2 − 1), uja derivada é2x cos(x2 − 1). Nos pontos x = ±1 a funçãopossui um bi o, e não possui derivada.(g) Primeiro es revemos em forma de função:exp(exp(x4)). A derivada é:exp(exp(x4)) exp(x4)4x3 ou 4x3ee

x4

ex4 .(h) 2 cos(2x)

√x2 + 1

sen(2x)+

log(sen(2x))x√x2 + 1Extra 3. (a) Nos pontos onde y′(x) = 3x2+4x−

4 = 0, isto é, x = −2 ou x = 2/3.(b) Rees revendo a reta 2y + 8x − 5 = 0 omo y = −4x + 5/2, observamos que o o-e iente angular é −4. Assim queremos saberquando f ′(x) = 3x2 + 4x − 4 = −4, isto é,x = 0, x = −4/3.Extra 4.Para garantir ontinuidade em x = 1devemos ter: a(1) + b = 1

1 , ou a + b = 1. Paraque as derivadas laterais sejam iguais em x = 1devemos ter 2ax = − 1

x2em x = 1, ou a = −1

2 .Assim b = 1− a = 32 .

58 CAPÍTULO 2. RESPOSTAS DOS EXERCÍCIOSExtra 5.Uma solução é: f ′(x) = g(x) + (x −a)g′(x). Como f ′(a) = g(a) + (a − a)g′(a) =g(a). O problema desta solução é que não sabe-mos se g pode ser derivada.A solução orreta é: Note que f(a) = (a −a)g(a) = 0 e f(a+ h) = (a+ h− a)g(a+ h) =hg(a + h). Assim, f(a + h) − f(a) = hg(a +

h). Logo, f(a+ h)− f(a)

h= g(a + h). Assim,

f ′(a) = limh→0

f(a+ h)− f(a)

h= lim

h→0g(a + h).Este limite é igual a g(a) pois g é ontínua em a.Assim, f ′(a) = g(a).Extra 6. (a) y′ = 2x senx + x2 cosx. Logo,

y′(π) = −π2. Assim a reta tangente é y =−π2(x− π).(b) y′ =

1√x− 2

12√x. Logo, y′((e + 2)2) =

1

2e(e + 2). Assim a reta tangente é y − 1 =

1

2e(e + 2)(x− (e+ 2)2).Extra 7. (a) y′ = − 1

x2 . Para que duas retas se-jam paralelas, basta que possua o mesmo oe- iente angular. Como o oe iente angular de2x + 3y = 0 é −2

3 , queremos determinar xR talque y′ = − 1x2 = −2

3 . Logo x = ±√3√2.(b) y′ = 2e2x. A reta tangente no ponto

c, e2c é y− e2c = 2e2c(x− c). Para que passe em(x, y) = (5/2, 0) temos que resolver: 0− e2c =2e2c(5/2 − c). Vamos obter que c = 3.Extra 8. (a) Considere f(x) = x − log x. Noteque f(1) = 1−0 = 1 > 0 e que f ′(x) = 1− 1

x > 0para x > 1. Assim a função é res ente parax > 1 e é positiva em 1. Logo f(x) > 0 paratodo x ∈ R, ou x − log(x) > 0, o que impli aque x ≥ log x.(b) Considere g(x) = −2x13 − 6x5 − x+10.Como lim

x→+∞g(x) = −∞ e lim

x→−∞g(x) = +∞,existem pontos onde a função é positiva e nega-tiva. Pelo TVI existe pelo menos uma raiz. Como

g′(x) = −26x12 − 30x4 − 1 é sempre negativa(para todo x ∈ R), g é de res ente para todo Re portanto injetiva. Assim a raiz é úni a pois afunção é injetiva.Extra 9. (a) Por hipótese existem a, b ∈ R omf(a) = f(b) = 0. Pelo TVM (ou pelo Teoremade Rolle) existe c ∈ (a, b) tal que f ′(c) = 0. Logof ′ possui uma raiz real.(b) Pelo TVM existe um c ∈ (2, 5) tal quef(5)− f(2)

5− 2= f ′(c). Logo, f(5)−f(2) = 3f ′(c).

Como por hipótese f ′(x) ≤ 4, f(5)− f(2) ≤ 12.( ) Como f ′′(x) = g′′1−g′′2 = cos(2x+log(x4+1)) − cos(2x + log(x4 + 1)) = 0 para todo x ∈R, on luímos que f ′(x) é onstante. Note quef ′(2) = g′1(2) − g′2(2) = −1 − (−1) = 0. Logof ′(x) = 0 para todo x. Assim f é onstante.Note que f(3) = g1(3) − g2(3) = 5 − 5 = 0.Logo f(x) = 0 para todo x. Con luímos queg1(x) = g2(x) para todo x ∈ R.Extra 10.Pela denição, omo f(0) = 0,

f ′(0) = limh→0

f(0 + h)− f(0)

h= lim

h→0

f(h)

h.Agora f(h) = 0 ou f(h) = hk, dependendo se

h ∈ Q ou não. Nos dois asos, |f(h)| ≤ hk.Assim, usando a ontinuidade da função módulo,|f ′(0)| =

limh→0

f(h)

h

= limh→0

f(h)

h

≤ limh→0

hk

h

= limh→0

|h|k−1 = 0.Portanto, 0 ≤ |f ′(0)| ≤ 0, ou seja, |f ′(0)| = 0 eportanto f ′(0) = 0.Extra 11.Note a beleza na simetria da resposta:(fgh)′ = f ′gh+ fg′h+ fgh′.2.3.4 DesaosDesao 1. (a) Esta função não é derivável nozero pois

f(0 + h)− f(0)

h=

h sen(1/h)

h= sen(1/h).Quando h → 0 o limite não existe.(b) Como,

g(0 + h)− g(0)

h=

h2 sen(1/h)

h= h sen(1/h),pelo Teorema do Sanduí he o limite quando h →

0 é zero. Assim, g′(0) = 0.Desao 2.Pelo binmio de Newton:(x+ h)n =

n∑

i=0

n!

i!(n− i)!xihn−i =

= xn + nxn−1h+ · · ·+ hn.Assim,(x+ h)n − xn = nxn−1h+ · · ·+ hn.

2.4. APLICAÇO DE DERIVADA 59Aqui temos termos om h, h2, . . . , hn. Dividindopor h, somente o primeiro termo não terá h:(x+h)n−xn

h = nxn−1 + (termos om h)+ hn−1.Se zermos h → 0, sobrará apenas o termonxn−1.Desao 3.O oe iente angular da reta tangenteao grá o de f no ponto x é f ′(x) = −x. O oe iente angular da reta tangente ao grá o deg no ponto x é g′(x). Queremos que g′(x) =−1/f ′(x), isto é, que g′(x) = 1/x. Logo g(x) =log x ou, de forma geral, g(x) = C + log x.Desao 4.Considere f(x) = ax2+bx+c. Assim,f ′(x) = 2ax + b. A se ante possui oe ienteangular: ax21 + bx1 − ax22 − bx2

x1 − x2=

=a(x21 − x22) + b(x1 − x2)

x1 − x2= a(x1 + x2) + b.A reta tangente no ponto médio possui oe- iente angular f ′((x1 + x2)/2) = a(x1 + x2) + bDesao 5.Cal ulando a derivada obtemos f ′(θ) =

0 para todo θ ∈ R. Logo, pelo TVM, f é ons-tante. Como f(0) = 1, f(θ) = 1 para todoθ ∈ R. Assim, cos θ + i sen θ

eiθ= 1 e obtemos oresultado.Desao 6.Note que onhe emos uma solução:

s = sen e c = cos. A questão aqui é a uni idade.2.4 Apli ação de Derivada2.4.1 Exer í ios de FixaçãoExer í io 1.O limite é 5 por L'Hospital.Exer í io 2.Não podemos apli ar L'Hospital duasvezes, somente uma vez obtendo lim

x→1

2(x− 1)

ex=

2(1− 1)

e= 0.Exer í io 3. (a) f(2.1) ≈ f(2)+f ′(2)(2.1−2) =

5 + 4(0.1) = 5.4.(b) f(1.95) ≈ f(2) + f ′(2)(1.95 − 2) = 5 +4(−0.05) = 4.8.Exer í io 4. p(π) = f(π) = −1, p′(π) = f ′(π) =− sen(π) = 0, p′′(π) = f ′′(π) = − cos(π) = 1.Exer í io 5. (a)026.juj.2010

x

y

−3 −2 −1 1 2(b)x

y

−2Exer í io 6.x

y

−3 −1 1 3

Exer í io 7. (a) e (b) Ambas verdadeiras. ( )Falso. Todos os pontos em [1, 2] são de máximoe de mínimo simultaneamente pela denição.Exer í io 8. (a) Falso. I tem que ser um inter-valo fe hado omo I = [−6, 99]. (b) Falso. Item que ser limitado e fe hado. ( ) Falso. Afunção tem que ser ontínua. (d) Falso. Mesmodes ontínua pode ter máximo. (e) Falso. Consi-dere I = R e a função ontínua f(x) =1

x2 + 1.O máximo é em x = 0.Exer í io 9. (a) Como f é ontínua em um inter-valo fe hado e limitado, podemos apli ar o TVE(Teorema do Valor Extremo de Weierstrass), Te-orema 22 da p.110, que garante que existe a.(b) Devemos omparar o valor da função nosextremos do intervalo om o valor da função nospontos ríti os. Assim omparando f(1), f(10),

f(3), f(7), determinaremos o máximo. Ou seja,o máximo será um dos pontos: 1, 3, 7 ou 10.( ) Não ne essariamente. Note que NO po-demos apli ar o TVE pois o intervalo não é limi-tado. Um exemplo é tomar uma f que vai para−∞ quando x → −∞.

60 CAPÍTULO 2. RESPOSTAS DOS EXERCÍCIOSExer í io 10. (a) a = −5. (b) b = 0. ( ) podeser em c = −1 ou c = 2. (d) d = 2.Exer í io 11. (a) maxx∈I

f(x) = 1/2, xmax = 2,minx∈I

f(x) = 1/3, xmin = 3.(b) maxx∈I

f(x) = +∞, não existe xmax =,minx∈I

f(x) = 1, xmin = 1.( )maxx∈I

f(x) = −4, xmax = −1/4, minx∈I

f(x) =

−1, xmin = −1.(d) maxx∈I

f(x) = 1, xmax = 1, minx∈I

f(x) = 0,não existe xmin =.(e) maxx∈I

f(x) = 0, minx∈I

f(x) = −∞, nãoexistem xmax nem xmin.Exer í io 12. (a) Verdadeiro, pois se é mínimo lo- al então a derivada é zero. (b) Verdadeiro, poisse é máximo no interior do intervalo, então é má-ximo lo al. ( ) Falso, pois está no extremo do in-tervalo. Pode ser zero mas não ne essariamente.(d) Falso. Um ponto om derivada zero podenão ser máximo nem mínimo, omo por exemplog(x) = (x − 3)3, que possui derivada nula emx = 3 mas não é máximo nem mínimo.Exer í io 13. (a) Verdadeiro. (b) Falso, pode sere pode não ser. Exemplo é f(x) = 3, onde TODOponto é de máximo lo al (e de mínimo lo al) em-bora f ′ = f ′′ = 0. ( ) Falso, nem todo máximolo al é máximo em um intervalo. O máximo podeo orrer no extremo do intervalo e a derivada nãopre isa ser zero neste ponto.Exer í io 14. (a) máximos lo ais: x = −2 e x =3. mínimos lo ais: x = 0.(b) Mínimo em x = 4, máximo em x = 3.( ) Mínimo em x = −3, máximo em x = 1.(d) Mínimo em x = 0, máximo em x = 3.(e) f ′′(−1.8) < 0. (f) f ′′(0) > 0. (g)f ′′(4) < 0. (h) x = −1 e x = 1.(i) mínimo lo al em x = 1. máximo lo al emx = 4. Olhe o sinal de g′ antes e depois destespontos.(j) onde g′′(x) = f ′(x) = 0? pontos de ine-xão de g: x = −2 e x = 3.Exer í io 15. (a) Como f ′(x) = 4x3 − 3x2 =x2(4x−3), os pontos ríti os são x = 0, x = 3/4.Note que o sinal da derivada é: f ′(x) < 0 parax < 3/4 e f ′(x) > 0 para x > 3/4. Assimx = 0 NO é extremos lo al. Somente x = 3/4é mínimo lo al.(b) Devemos omparar f(−1) = 2, f(2) = 8,f(3/4) = −27/256. Assim o máximo em I é emx = 2 e o mínimo em x = 3/4.

( ) Aqui basta omparar f(−1) = 2 omf(0) = 0. Assim o máximo é em x = −1 e omínimo em x = 0.(d) No extremo do intervalo x → ±∞ a fun-ção f(x) → +∞. Assim ela não tem máximo. Omínimo é no ponto ríti o x = 3/4.(e) No extremo x → −∞ a função f(x) →+∞. No extremo x = 1, f(−1) = 2. Nenhumponto ríti o perten e ao intervalo. Assim ela nãotem máximo e o mínimo é em x = −1.2.4.2 ProblemasProblema 1. (a) 4. (b) Tomando o log obtemosque se y = (ex + 3x)1/x, log y =

log(ex + 3x)

x.Apli ando L.H. obtemos que Assim log( lim

x→0+y) =

4. logo limx→0+

y = e4. ( ) 2/5. (d) Note que(ax)′ = (log a)ax. Assim o limite é log a− log b.Problema 2. (a)√65 ≈

√64+

1

2√64

(65−64) =

8+1

16. (b) log(e2−0.1) ≈ log(e2)+

1

e2(−0.1) =

2− 1

10e2. ( ) Re ordando, arctan′(x) = 1

x2 + 1.Assim, arctan′(1) = 1

2 . Assim arctan(1.2) ≈tan(1) + 1

2 (1.2− 1) =π

4+ 0.1.Problema 3. (a) Como f ′(x) = 3ax2 + 2bx+ c,os extremos lo ais vão o orrer (possivelmente) so-mente nos pontos onde f ′(x) = 0. Se a equaçãopossuir duas raízes reais distintas, o sinal de f ′passará de positivo para negativo ou vi e-versaem ada raiz: assim um ponto será de máximoe o outro de mínimo lo al. Se possuir uma raizdupla, omo a > 0, f ′(x) ≥ 0 para todo x ∈ R.Assim o ponto onde f ′ se anula não será de má-ximo nem mínimo. Finalmente se f ′ não possuirraiz real, omo a > 0, f ′(x) > 0 para todo x ∈ R.Assim a função será sempre res ente, sem extre-mos lo ais.(b) Se f não possui extremos lo ais então

f ′(x) ≥ 0 para todo x ∈ R. Assim f poderápossuir no máximo 1 raiz. Como é polinmio degrau impar, pelo TVI (porque?) possui no mínimouma raiz. Con luímos que f possui exatamente1 raiz.( ) Se f possui 2 extremos lo ais, temos queveri ar se o mínimo lo al é menor que zero ounão e se o máximo lo al é menor que zero ou não(faça uma gura). Se ambos forem menor que

2.4. APLICAÇO DE DERIVADA 61zero ou ambos maiores que zero, f admite so-mente uma raiz real. Se o máximo lo al é maiorque zero e o mínimo lo al menor que zero, f ad-mite exatamente 3 raízes reais.(d) Determine ( aso existam) os dois pontos ríti os distintos x0 < x1 de f , isto é, pontos taisque f ′(x0) = f ′(x1) = 0. Caso não existam ouexista somente um, a função possui somente umaraiz real.Como a > 0 ne essariamente x0 é máximoe x1 é minimo (basta olhar sinal de f ′, que vempositivo até x0, a negativa em (x0, x1) e voltaa ser positivo em x1. Se f(x0) > 0 > f(x1)possui 3 raízes reais, aso ontrário somente umaraiz real.Problema 4. (a) f ′(x) = 3x2 − 6x+ 3 = 3(x−1)2, uja úni a raiz é x = 1. Assim f ′(x) ≥ 0para todo x ∈ R. Logo esta função é sempre res ente. Como f ′′(x) = 6x − 6, ela tro ade on avidade em x = 1. Quando x → +∞,f(x) → +∞ e quando x → −∞, f(x) → −∞.Não possui assíntota verti al nem horizontal.Embora f ′(1) = 0, omo f ′ > 0 perto dex = 1, este ponto não é de máximo nem mínimo.

x

y

1

(b) Como g′(x) = x(x2 − 4), a derivada seanula em 0,±2. Analisando sinal de g′ (quadro desinais) on luímos que g de res e até −2, res eem (−2, 0), de res e em (0, 2), e res e de 2 emdiante. Com isso vemos que os pontos x = ±2são de mínimo lo al e x = 0 é de máximo lo al.Como g′′(x) = 3x2 − 4, a on avidade muda em± 2√

3, sendo para ima antes de − 2√

3de depois de

2√3e para baixo em (− 2√

3, 2√

3). Assim o grá oé:

x

y

−2 22√3

− 2√3Problema 5.O esboço deverá ter uma f(x) = 2para x ∈ [0, 1] pois f ′(x) = 0 neste intervalo.Ela deverá de res er para x > 1 om on avidadepara baixo pois f ′′ < 0. Entre −2 e 0 ela deverá res er pois f ′ > 0 neste intervalo. No entantoa on avidade deve ser para ima até −1 e parabaixo depois. Até o −2 ela deve deve de res er om on avidade para ima e um mínimo lo al em

x = −2 pois a derivada se anula em −2. Assimobtemos:x

y

−2 −1 1 2

Problema 6. (a) Possui duas assintotas horizon-tais: y = 2 e y = −1. Possui assintota verti alem x = 0. Possui um máximo lo al em x = −1.x

y

y = 2

y = −1−1

1

(b) Possui duas assintotas horizontais: y = 0e y = −1. Possui assintota verti al em x = 2.Possui um máximo lo al em x = 0.

62 CAPÍTULO 2. RESPOSTAS DOS EXERCÍCIOSx

y

x = 2

y = −1−2

1

2

Problema 7. (a) Intersepta os eixos em (0, 0) e(1, 0). Assintotas verti ais em x = 2 e x = −1.Assintota horizontal: y = 2. Sinal de f ′ é igualao sinal de 1 − 2x: a função res e até x = 1/2e de res e depois. Em x = 1/2 a função tem ummáximo lo al.O sinal de g′′ é igual ao sinal de (x− 2)(x+1) (note que x2 − x + 1 > 0 pois as raízes são omplexas): on avidade para ima até x = −1e depois de x = 2. Con avidade para baixo em(−1, 2).

x

y

y = 2

−1 212(b) Intersepta os eixos em (0, 1). Assintotasverti ais em x = ±1. Assintota horizontal: y =

−1. Sinal de g′ é igual ao sinal de x: a funçãode res e até x = 0 e res e depois. Em x = 0 afunção tem um mínimo lo al.O sinal de g′′ é igual ao sinal de 1−x2: on a-vidade para baixo até x = −1 e depois de x = 1.Con avidade para ima em (−1, 1).x

y

y = 1 −1 1

( ) Intersepta os eixos em (0, 0). Assintota

verti al em x = 1. Assintota horizontal: y = 0.Sinal de h′: a função de res e até x = −1, res eem (−1, 1), de res e depois de x = 1. Em x =−1 a função tem um mínimo lo al.O sinal de h′′ é igual ao sinal de x+2: on a-vidade para baixo até x = −2, Con avidade para ima depois.

x

y

−2 1Problema 8. (a) Não intersepta os eixos (nun avale zero e não está denida em x = 0). Assintotaverti al em x = 0. Assintota horizontal: y = 0.Sinal de f ′ é igual ao sinal de x − 1 pois exe x2 são sempre positivas: a função de res e atéx = 1 e res e depois de x = 1. Em x = 1 afunção tem um mínimo lo al.O sinal de f ′′ é igual ao sinal de x3 pois opolinmio x2 − 2x+ 2 possui raízes omplexas e omo oe iente de x2 é positivo, x2−2x+2 > 0para todo x ∈ R. Assim f ′′ é negativa para x < 0e positiva para x > 0. Portanto on avidade parabaixo para x < 0, Con avidade para ima parax > 0.

x

y

1(b) Note que a função está denida somenteonde 1− x2 > 0, isto é, para x ∈ (−1, 1). Inter-septa os eixos em (0, 1) e quando log(1 − x2) =−1, isto é, quando 1 − x2 = e−1. Portantoquando x2 = 1− e−1, isto é, x = ±

√1− e−1 ≈

±0.79 (pelo software Maxima). Logo inter eptao eixo x em (±0.79, 0). Assintota verti al emx = ±1 (onde temos log 0 = −∞!). Assintota

2.4. APLICAÇO DE DERIVADA 63horizontal não existe (função nem esta denidapara x > 1 nem x < −1).Sinal de f ′ é igual a de −2x para x ∈ (−1, 1)pois x2 − 1 < 0 neste intervalo. Assim a função res e para x < 0 e de res e para x > 0. Emx = 0 a função tem um máximo lo al.O sinal de f ′′. Note que o numerador 2x2+2é sempre positivo e omo o denominador é iguala (x2 − 1)2, que é sempre positivo, por ter sinalde menos na frente será sempre negativa. Assimf ′′ < 0 e a on avidade é sempre para baixo.

x

y

x = 1x = −1

√1− e−1−

√1− e−1

1

( ) Intersepta os eixos em (0, 1 + e−2). Nãopossui Assintota verti al. Assintota horizontal:y = 1.Sinal de f ′ é igual ao sinal de 3 − 2x poisexponen ial de qualquer oisa é sempre positiva.Portanto a função res e até x = 3/2 e de res edepois. Em x = 3/2 a função tem um máximolo al.O sinal de f ′′ é igual ao sinal de 4x2 − 12x+7. As raízes são: 3/2 ±

√2/2. A on avidadepara baixo em 3/2 −

√2/2, 3/2 +

√2/2), ou,aproximadamente, em (0.79, 2.20). Con avidadepara ima fora deste intervalo.

x

y

y = 1

0.79 2.2032(d) Intersepta os eixos em (0, 0). Não tem

Assintota verti al. Assintota horizontal: y = 0quando x → −∞.Sinal de f ′ é igual ao sinal de x+ 3, pois o-lo ando em evidên ia x2, que é sempre positivo,obtemos isto. Note que a derivada será zero emx = −3 e em x = 0. Note que em zero a derivadaNO tro a de sinal, ontinuando positiva. Assima função de res e até x = −3 e res e depois.Em x = −3 a função tem um mínimo lo al. Oponto x = 0 possui derivada zero (é ponto rí-ti o) mas não é máximo nem mínimo lo al pois afunção res e em torno de x = 0 (f ′(x) > 0 parax próximo mas diferente de zero).O sinal de f ′′ é igual ao sinal de x(x2+6x+6).As raízes são 0,−3±

√3.

−3−√3 ≈ −4.7 e −3 +

√3 ≈ −1.26.Fazendo quadro de sinais vamos obter que: on avidade para baixo até x = −3 −

√3 ≈

−4.7 e também no intervalo (−3 +√3, 0) ≈

(−1.26, 0). A Con avidade será para ima em(−3 −

√3, −3 +

√3) ≈ (−4.7, −1.26) e tam-bém para x > 0.

x

y

−3−4.7 −1.26Problema 9. (a) O ponto ríti o é a solução desen3(x) = cos3(x), e portanto se tan3(x) =1, ou seja, quando tanx = 1, o que o orre sex = π/4. Quando x → 0+ ou x → π/2−,f(x) → +∞. Assim o mínimo é em x = π/4 om f(π/4) = 2

√2 e NO existe máximo em

I. Portanto maxx∈I

f(x) = +∞, não existe xmax,minx∈I

f(x) = 2√2, xmin = π

4.(b) O úni o ponto ríti o é em x = 2 (f ′(2) =

0). Quando x → 0+ ou x → +∞, f(x) → +∞.Assim em I = (0, +∞) o mínimo é em x = 2 e omáximo não existe. Portanto maxx∈I

f(x) = +∞,não existe xmax, minx∈I

f(x) =5

2, xmin = 2.Em I = (0, 3], omo 2 ∈ I, o mínimo é em

x = 2 e o máximo não existe pois próximo de 0f(x) → +∞. Portanto max

x∈If(x) = +∞, não

64 CAPÍTULO 2. RESPOSTAS DOS EXERCÍCIOSexiste xmax, minx∈I

f(x) =5

2, xmin = 2.Em I = [3, 4] não tem ponto ríti o. Logo omáximo e o mínimo estão nos extremos: f(3) =

3 + 1/3 e f(4) = 4 + 1/4. Logo o mínimo éem x = 3 e o máximo em x = 4. Portantomaxx∈I

f(x) = 4 + 1/3, xmax = 4, minx∈I

f(x) =

3 + 1/3, xmin = 3.( ) Note que o termo da derivada x2 − x+2possui raízes omplexas. Como o termo de maiorgrau é x2, x−x+ 2 > 0 para todo x ∈ R. Logoa úni a raiz é x = 0, om sinal de f ′ igual aosinal de x. Como f de res e até x = 0 e res edepois, x = 0 é mínimo lo al.Assim em [−1, 1] omparamos f(−1) = 19,f(1) = 11, f(0) = 0. Portanto max

x∈If(x) = 19,

xmax = −1, minx∈I

f(x) = 0, xmin = 0.Em [1, 2], não tem ponto ríti o, basta om-parar f(2) = 64 e f(1) = 11. Portantomaxx∈I

f(x) =

64, xmax = 2, minx∈I

f(x) = 11, xmin = 1.(d) Note que f ′ é sempre positiva. Logo f ésempre res ente. Note que limx→−1−

f(x) = +∞Em I = (−1, 1] temos que limx→−1+

f(x) =

−∞. Assim não possui mínimo. O máximo seráem x = 1 om f(1) =1

2. Portanto max

x∈If(x) =

1

2, xmax = 1, min

x∈If(x) = −∞, não existe xmin =.Em I = [0, 1], omo f(0) = 0, max

x∈If(x) =

1

2, xmax = 1, min

x∈If(x) = 0, xmin = 0.Problema 10.Determine o máximo e o mínimode f(x) =

x

x4 + 3em R. Con lua que K ∈

[−1/4, 1/4].Problema 11.MODELAGEM: Se x, y são os nú-meros, y − x = 100, p = xy mínimo. Comoy = x + 100, p(x) = (x + 100)x. Queremosminimizar p(x) para x ∈ R.RESOLUÇO: Como p(x) → +∞ quandox → ±∞, o mínimo é no ponto de derivada zero.Como p′(x) = 2x + 100, x = −50 é o ponto dederivada zero, om y = −50 + 100 = 50. Logoos números são 50 e −50.Problema 12.MODELAGEM: Suponha que a do-bra tenha omprimento x. A alha terá a formade um retângulo om lado x e L− 2x (o que so-brou para base. Como o volume é propor ionala área deste retângulo, queremos o máximo def(x) = x(L− 2x) para x ∈ [0, L/2].

RESOLUÇO: Como é equação do segundograu om on avidade para baixo, o máximo é noponto de derivada zero. Como f ′(x) = L− 2x−2x = L − 4x, x0 = L/4. Assim a alha deveráter a forma de um retângulo om dimensões L/4e L/2.Problema 13.MODELAGEM: Seja r o raio do ír ulo e θ o ângulo do setor ir ular. Quere-mos maximizar a área a = θr2/2. O perímetrodeste setor é 2r mais θr. Assim, 40 = 2r + θr.Logo, θ = 40/r − 2. Logo queremos o má-ximo de a(r) = 20r − r2. Note que θ varia en-tre 0 e 2π. Como 40 = 2r + θr, para θ = 0,r = 20 e para θ = 2π, r = 20/(1 + π). Assimr ∈ [20/(1 + π), 20].RESOLUÇO: Trata-se de uma equação dosegundo grau. a′(r) = 20 − 2r. Logo a derivadaé zero em r0 = 10. Como 20/(1 + π) < 20/4 =5 < 10 (π > 3), o máximo é em r0 = 10.Problema 14.MODELAGEM: Vamos modelar in-troduzindo θ para o ângulo e B = K − h para adiferença entre a distân ia da tela ao hão e a al-tura dos olhos do espe tador. Note que se h → 0ou h → +∞ o ângulo θ → 0.Por trigonometria simples, tanϕ =

B

de tan(θ+

ϕ) =L+B

d. Assim, ϕ = arctan(B/d) e θ +

ϕ = arctan((L+ B)/d). Logo, o ângulo θ(d) =arctan((L+B)/d)− arctan(B/d).Queremos maximizar θ(d) para d ∈ (0,+∞).RESOLUÇO: Derivando obtemos

θ′(d) =L(BL+B2 − d2)

(B2 + d2)((L+B)2 + d2).Queremos determinar d0 tal que θ′(d0) = 0. Comoo denominador é sempre positivo e L > 0, a úni araiz da derivada é d0 tal que BL+B2 − d20 = 0,isto é (solução positiva) d0 = √

B2 +BL.Problema 15.MODELAGEM: Sejam x e y asdimensões do artaz. Sua área A = xy. A áreaimpressa será igual a (x − 2M)(y − 2N). Elimi-nando y = A/x obtemos que queremos maximi-zar a área impressa f(x) = (x−2M)(A/x−2N) om x ∈ [2M,A/(2N)].RESOLUÇO: Di a: Resolva o problema omA = 50,M = 2, N = 4. Vou dar a soluçãogeral. Como f ′(x) = A/x−2N−(x−2M)A/x2,os zeros da derivada são ±

AM/N . Queremossomente a solução positiva x0 =√AM/N . Noteque nos extremos a área impressa f é zero. Assim

2.4. APLICAÇO DE DERIVADA 65o máximo é de fato em x0 se nos erti armos quex0 ∈ [2M,A/(2N)].Vamos provar que de fato x0 ∈ [2M,A/(2N)].Para que o problema faça sentido a área A deveser maior que a área das margens (2M)(2N) =4MN . Assim, 4MN < A. Logo, 4M2 < AM/N ,e portanto 2M <

AM/N = x0. Por outrolado, AM/N < A2/(4N2). Logo, √AM/N =x0 < A/(2N).Problema 16.MODELAGEM ( omum aos doisitens): Seja h a altura e r o raio das semiesferas.O volume é V = 4/3πr3 + πr2h e a área desuperfí ie é A = 4πr2 + 2πrh.(a) MODELAGEM: Se xarmos a área emA, tiramos que πrh = (A − 4πr2)/2. Assim,V (r) = 4/3πr3 + r(A − 4πr2)/2. Queremosmaximar V (r) em [0,

A/(4π)] ( hegamos nestevalor tomando h = 0 na relação A = 4πr2+2πrh). RESOLUÇO: Vamos al ular o ponto rí-ti o. Como V ′(r) =A− 4πr2

2, V ′(r0) = 0 se

A = 4πr20. Assim a derivada é zero no extremodo intervalo r0 =√

A/(4π). Note que V (0) = 0e V ′(0) = A/2 > 0. Alem disso V ′(x) > 0 paratodo x ∈ [[0,√

A/(4π)]. Logo V res e neste in-tervalo e portanto r =√

A/(4π) é o ponto ondeV (r) assume o máximo.(b) MODELAGEM: O usto de fabri ação épropor ional a área de superfí ie A. Como o vo-lume V é xo, tiramos que πrh = (V−4/3πr3)/r.Assim,A(r) = 4πr2 + 2(V − 4/3πr3)/r =

6V + 4πr3

3r.Queremos minimizar A(r) para r ∈ (0, 3

3V

4π)( hegamos neste valor tomando h = 0 na relação

V = 4/3πr3 + πr2h).RESOLUÇO: Note que A(r) → +∞ quandor → 0+ ou r → +∞. Assim o mínimo o orreráem um ponto ríti o. Como A′(r) =

8πr3 − 6V

3r2.Assim a derivada se anula somente em r0 =

3

3V

4π.Pode-se onrmar que o mínimo é em r = r0 poiso sinal da derivada é sempre negativa.Problema 17. (a) MODELAGEM: Queremos mi-nimizar o quadrado da distân ia g(x) = (x−2)2+

(f(x) − 2)2 = (x − 2)2 + |6x − 2x2|. Note queo domínio de f é onde 6x − 2x2 > 0, isto é em[0, 3].

RESOLUÇO: Apli ando a denição de mó-dulo observamos que |6x − 2x2| = 6x − 2x2se x ∈ [0, 3]. Assim g(x) = −x2 + 2x + 4 sex ∈ [0, 3]. Em [0, 3], g′(x) = −2x+ 2 e g′(1) =0. Temos que omparar g(0) = 4, g(1) = 5 eg(3) = 1. Observamos que o mínimo é em x = 3 om g(3) = 1 e o máximo é em x = 1 omg(1) = 5.(b) MODELAGEM: A distân ia verti al f(x)é igual a diferença entre os y's. Assim, queremoso mínimo de f(x) = x2 +

1

x2para x ∈ R.RESOLUÇO: Note que f(x) → +∞ para

x → ±∞. Logo o mínimo será no ponto dederivada zero. Como f ′(x) = 2x− 2

x3, os pontos ríti os são 1 e −1. Como f(1) = f(−1) = 2, omínimo é em x = 1 ou x = −1.Problema 18.MODELAGEM: Sejam x, y os la-dos do retângulo. O perímetro P = 2x + 2y.Note que ligando-se o entro do ír ulo a umvérti e do retângulo obtemos um triângulo retân-gulo om lados x/2, y/2, R. Assim, por Pitágo-ras, x2 + y2 = 4R2. Logo, y =

√4R2 − x2 para

x ∈ [0, 2R]. Assim queremos o máximo e mínimode P (x) = 2x+ 2√4R2 − x2 para x ∈ [0, 2R].RESOLUÇO: Como P ′(x) = 2− 2x√

4R2 − x2.Note que P ′(x) = 0 sse 2

√4R2 − x2 = 2x. Araiz positiva será x0 = R

√2. Como x20 + y20 =

4R2, y0 = R√2. Comparando P (0) = 4R =

P (2R) e P (R√2) = R3

√2. Assim, omo 4 <

3√2, o maior perímetro será R3

√2 para o qua-drado de lado R

√2. O menor será para o retân-gulo degenerado de lados 0 e 2R, om perímetro

4R.Problema 19.MODELAGEM: Vamos xar x omosendo o ponto do eixo x que é um dos vérti- es do retângulo. Automati amente os outrosvérti es vão ser (x, y(x)), (−x, y(x)) e (−x, 0).Assim a área A = (2x)y(x) = 2(27x − x3).Note que omo as raízes da parábola são ±√27,

x ∈ [−√27,

√27] e queremos maximizar A(x) =

2(27x − x3).RESOLUÇO: Como A′(x) = 2(27 − 3x2),os pontos ríti os são x = ±3, que perten emao intervalo. Note que A(±√27) = 0. Assim omáximo será em x = 3 onde A(3) = 108. Noteque y(x) = 18. Assim as dimensões são 2x = 6por y = 18Problema 20. (a) MODELAGEM: Seja r o raioe h a altura do one ins rito na esfera. O volume

66 CAPÍTULO 2. RESPOSTAS DOS EXERCÍCIOSdo one é V =1

3πr2h. Note que omo 1

3π éum onstante, maximizar a função f = r2h é umproblema equivalente. Como é função de duasvariáveis, devemos eliminar uma delas.Ligando-se o entro da esfera até um dos pon-tos do ír ulo da base do one observamos o tri-ângulo retângulo om hipotenusa R e atetos r e

h−R. Logo, por Pitágoras, (h−R)2+ r2 = R2,Assim, r2 = 2hR − h2. Logo f(h) = h(2hR −h2). Note que h ∈ [0, 2R]. Assim queremos omáximo de f(h) para h ∈ [0, 2R].RESOLUÇO: Note que f(0) = f(2R) = 0.Como f ′(h) = 4hR − 3h2 = h(4R − 3h), ospontos ríti os são h = 0 e h = 4R/3. Como oponto zero não é de máximo, o máximo é quandoh = 4R/3.(b) MODELAGEM: Seja r o raio e h a alturado ilindro ins rito no one. O volume do ilindroé V = πr2h. Como é função de duas variáveis,devemos eliminar uma delas. Note que ortandoo one temos uma semelhança de triângulos: aaltura H do one está para R assim omo H −hestá para r. Assim, H

R=

H − h

r. Logo, r =

R(H − h)

H. Logo queremos maximizar V (h) =

πh

(

R(H − h)

H

)2. Note que h ∈ [0, H]. Assimqueremos o máximo de V (h) para h ∈ [0, H].RESOLUÇO: Note que V (0) = V (H) =

0. Como V ′(h) =πR2(H − 3h)(H − h)

H2(vaiobter-se equação do segundo grau om raízes He H/3). Como V (H) = 0, o máximo é para h =

H/3 (não pre isa al ular V (H/3) =4πHR2

27,que obtive om o Maxima).2.4.3 ExtrasExtra 1. (a) 4. (b) +∞ ( ) 5/4. (d) Note que

x1/x = (elog x)1/x = elog x/x. Quando x → +∞,log x/x → 0. logo o limite é e0 = 1.Extra 2. (a) tan(0.05) ≈ tan(0)+tan′(0)(0.05−0) = 0.05. (b) 3

√28 ≈ 3

√27 + 1/(3

3√272)(28 −

27) = 3 + 1/(27).Extra 3. (a) g′(x) = (x − 3)(x − 2). Assim afunção res e antes de x = 2, de res e em (2, 3)e res e depois de x = 3. Além disso, omog′′(x) = 2x − 5, a on avidade é para baixo atéx = 5/2 e para ima depois.

x

yg(x)

2 352

(b) f ′(x) = 4x(x − 1)(x + 1) e f ′′(x) =12x2 − 4. Note que f(x) ≥ 0 e é zero somenteem x = ±1. Pela derivada, a função de res e atéx = −1, res e em (−1, 0), de res e em (0, 1) e res e de 1 em diante. Os pontos x = ±1 são demínimo lo al. O ponto x = 0 é de máximo lo al.Quando x = 0, y = 1.Pela f ′′, a on avidade é para ima até−1/

√3,para baixo em (−1/

√3, 1/

√3) e para ima nova-mente para x > 1/

√3.

x

y f(x)

−1 11√3

− 1√3Extra 4.Extraímos do grá o informação sobre res imento e de res imento. Basta olhar ondea função do grá o é positiva e onde é negativa.Assim, f(x) de res e ate x = −2, em (0, 2) edepois de x = 4. Ela res e em (−2, 0), (2, 4).Agora se observarmos o grá o podemos ob-ter a informação sobre a derivada da função re-presentada, isto é, sobre a derivada segunda de

f . Assim, f ′′(−1) = f ′′(1) = f ′′(3) = 0. Obser-vando o sinal de f ′′ on luímos que a on avidadede f(x) é para ima até x = −1, em (1, 3). A on avidade é para baixo em (−1, 1) e depois dox = 3.

2.4. APLICAÇO DE DERIVADA 67x

y

f(x)

−2 −1 1 2 3 4

Extra 5. (a) Ponto de máximo lo al em x = 0 emínimo lo al em x = 2.x

y

1 2

3

(b) Assintota horizontal y = −3 e máximolo al em x = 2.x

y

2 4

y = −3

Extra 6. (a) Assíntotas verti ais em x = 1 e x =−3, horizontal em y = 1. Intersepta o eixo xem x = −1 e x = −2. O sinal da derivada serádado pelo polinmio −x2−10x−13, ujas raízessão: −5± 2

√3, que são aproximadamente −8.4e −1.5.

x

y

f(x) =(x+ 1)(x+ 2)

(x+ 3)(x− 1)

y = 1

−1

(b) Note que as assíntotas verti ais são x = 3e x = 1. A horizontal é y = 0. O sinal daderivada é igual ao de x2 − 3: a função de res eem (−√3,√3) e res e fora. Tem máximo lo alem −

√3 e mínimo lo al em √

3.Ignoramos a derivada segunda pois ela vai darg′′(x) = 2

x3 − 9x+ 12

(3 − x)3(x− 1)3. Note que o polin-mio do denominador é do ter eiro grau, e portantonão sabemos omo al ular a raiz ( om o Maximaobtemos que a úni a raiz real é −32/3 − 31/3.).

x

y

f(x) =x

(x− 3)(1 − x)+ 3Extra 7. (a) A função res e para x > 0 e de- res e para x < 0 pois o sinal de f ′ é deter-minado por x. A derivada se anula em x = 0,que é ponto de mínimo lo al. A assintota hori-zontal é y = 1, verti al não tem. Intersepta oseixos somente em (0, 0). Con avidade p/ imaem (−1, 1) para baixo fora.

68 CAPÍTULO 2. RESPOSTAS DOS EXERCÍCIOSx

y

f(x) =x2

x2 + 3

y = 1

−1 1

(b) A função res e para x > 0 e de res epara x < 0 pois o sinal de f ′ é determinado porx. A derivada se anula em x = 0, que é pontode mínimo lo al. A assintota horizontal é y = 1,verti al x = ±2. A on avidade é determinadapelo sinal de 4− x2 (pois o numerador é semprepositivo): on avidade p/ ima em (−2, 2) parabaixo fora.

x

y

f(x) =x2

4− x2+ 2

x = 2 x = −2

2y = 1

Extra 8. (a) Note que a função está denida so-mente para x > 0. Note que limx→0+

x log x = 0por L'Hospital. Intersepta os eixos em (0, 0) e(1, 0).Não possui assíntota verti al nem horizontal.Sinal de f ′. Note que log x = −1 quandox = e−1 ≈ 0.36. A função de res e até x = e−1,e res e depois. Em x = e−1 a função tem ummínimo lo al.O sinal de f ′′ é sempre positivo para x >0, o domínio da função. Assim a on avidade ésempre para ima.

x

y

f(x) = x log x

1e(b) Intersepta os eixos em (0, 0). Não temAssintota verti al. Assintota horizontal: y = 0.Sinal de f ′ é igual ao sinal de 1− 2x2, ujasraízes são ±1/

√2 ≈ ±0.707. Assim a função res e em (− 1√

2, 1√

2) ≈ (−0.7, 0.7). Ela de res efora deste intervalo. Em x = − 1√

2a função temum mínimo lo al e em x = 1√

2a função tem ummáximo lo al.O sinal de f ′′ é igual ao sinal de 2x(2x2− 3).Assim a on avidade para ima em (−

3/2, 0) ≈(−1.22, 0) e para x >

3/2) ≈ 1.22, Con avi-dade para baixo em (0,√

3/2) ≈ (0, 1.22) e parax < −

3/2 ≈ −1.22.x

y

f(x) = xe1−x2

1√2

−1√2

( ) Intersepta os eixos em (0, 0). Não temAssintota verti al. Assintota horizontal: y = 0.Sinal de f ′ é igual ao sinal de x(x+2). Assima função de res e em (−2, 0) e res e fora desteintervalo. Em x = −2 a função tem um máximolo al e em x = 0 tem um mínimo lo al.O sinal de f ′′ é igual ao sinal de x2 + 4x +2. Assim a on avidade para baixo em (−2 −√2, −2 +

√2) ≈ (−3.41, −0.58) Con avidadepara ima fora deste intervalo.

2.4. APLICAÇO DE DERIVADA 69x

y

f(x) = x2ex−2Extra 9. (a) Pontos ríti os são x = 0, x =

±2. Temos que omparar f(0) = 0 om f(2) =f(−2) = 16. Além disso, quando x → ±∞,f(x) → −∞.Assim, em I = R, max

x∈Rf(x) = 16, xmax = 2ou −2, min

x∈Rf(x) = −∞, não existe xmin =.Em I = [−1, 1], devemos omparar f(−1) =

f(1) = 7, f(0) = 0 (úni o ponto ríti o no in-tervalo. Logo maxx∈I

f(x) = 7, xmax = 1 ou −1,minx∈I

f(x) = 0, xmin = 0.(b) Úni o ponto ríti o é x = 0.Em I = [1, 2], que não ontém o ponto rí-ti o, devemos omparar f(1) =1

2e f(2) =

1

5.Assim max

x∈If(x) = 1/2, xmax = 1, min

x∈If(x) =

1/5, xmin = 2.Em I = [−1,+∞), omo f(x) → 0 quandox → +∞, f(0) = 1 e f(−1) = 1/2, max

x∈If(x) =

1, xmax = 0, minx∈I

f(x) = 0, não existe xmin.( ) Note que f ′(x) = − sen(x) cos(cos(x)).Logo os pontos ríti os vão ser onde senx = 0 ouonde cos(y) = 0. Assim a derivada será nula emx = 0, π pela equação senx = 0. Para a outra,y = π/2 ≈ 1.57. Como y = cos x, esta equaçãoé impossível pois nun a cos x = 1.57 > 1. Omesmo o orrerá om outros valores.Assim devemos omparar f(0) = sen 1, f(π) =sen−1 = − sen 1, f(2π) = sen 1.Assimmax

x∈If(x) = sen 1, xmax = 0, min

x∈If(x) =

− sen 1, xmin = π.Extra 10.MODELAGEM Se o triângulo equilá-tero tem lado x, o retângulo possuirá lados x e y.Como a quantidade de luz é propor ional a áreada janela, queremos maximizar a área da janelaa = xy + x2

√3/4 (área retângulo mais semi- ír ulo). Esta é uma função de duas variáveis.Utilizando a restrição que o perímetro da janela

12 = 3x+2y, obtemos que y = 6−3/2x. Assima(x) = x(6 − 3/2x) + x2

√3/4. Olhando parao perímetro 12 = 3x + 2y, vemos que os asosextremos são x = 4 (y = 0) e x = 0. Assim

queremos o máximo de a(x) para x ∈ [0, 4].RESOLUÇO: Como a′(x) = 6−(3−√3/2)x,

x0 = 6/(3−√3/2) ≈ 2.811 < 4 é o úni o ponto ríti o e perten e a [0, 4]. Nos extremos do inter-valo, a(0) = a(4) = 0.Extra 11. (a) MODELAGEM: Considere x e y omo as dimensões do retângulo. Então quere-mos maximizar a área a = xy. Como são duasvariáveis, utilizamos a restrição P = 2x + 2ypara eliminar uma delas. Assim y = P/2 − x.Logo queremos o máximo de a(x) = xP/2− x2.Note que x pode ser 0 no mínimo, mas omo

P = 2x+2y e y pode valer no mínimo 0, x podevaler no máximo P/2. Assim queremos o máximode a(x) = xP/2− x2 para x ∈ [0, P/2].RESOLUÇO: Como a′(x) = P/2−2x, x0 =P/4 é o úni o ponto ríti o e perten e ao inter-valo [0, P/2]. Como a(x) é uma parábola om on avidade para baixo, o máximo é em x0. Neste aso, omo 2x0 + 2y0 = P , 2y0 = P − 2P/4 =P/2. Assim x0 = y0 = P/4 e portanto o retân-gulo é um quadrado.(b) MODELAGEM: Considere x e y omoas dimensões do ampo, om y o lado opostoao órrego. Então queremos maximizar a áreaa = xy. Como são duas variáveis, utilizamos arestrição P = 2x + y para eliminar uma delas.Assim y = P − 2x. Logo queremos o máximode a(x) = xP − 2x2. Note que x pode ser 0 nomínimo, mas omo P = 2x+ y e y pode valer nomínimo 0, x pode valer no máximo P/2. Assimqueremos o máximo de a(x) = xP − 2x2 parax ∈ [0, P/2].RESOLUÇO: Como a′(x) = P − 4x, x0 =P/4 é o úni o ponto ríti o e perten e ao inter-valo [0, P/2]. Como a(x) é uma parábola om on avidade para baixo, o máximo é em x0 =P/4. Neste aso, omo 2x0 + y0 = P , y0 =P − 2P/4 = P/2 > x0 = P/4.( ) MODELAGEM: Considere x e y omo asdimensões do terreno, onde x é da er a refor-çada. Então queremos maximizar a área a = xy.Como são duas variáveis, utilizamos a restriçãodo usto total da er a 6000 = 3(2x) + 2(2y).Assim 6000 = 6x+4y ou 3000 = 3x+2y. Logo,y = 1500 − 3/2x. Logo queremos o máximo dea(x) = x(1500−3/2x). Note que x pode ser 0 nomínimo, mas omo 3000 = 3x+2y e y pode valerno mínimo 0, x pode valer no máximo 1000. As-sim queremos o máximo de a(x) = 1500x−3/2x2para x ∈ [0, 1000].RESOLUÇO: Como a′(x) = 1500−3x, x0 =

70 CAPÍTULO 2. RESPOSTAS DOS EXERCÍCIOS500 é o úni o ponto ríti o e perten e ao intervalo[0, 1000]. Como a(x) é uma parábola om on- avidade para baixo, o máximo é em x0 = 500.Neste aso, omo y0 = 1500 − 3/2x0 = 750.Extra 12. (a) MODELAGEM: Queremos o má-ximo de x2 + y2 mas om x + y = S. Logoqueremos maximizar f(x) = x2 + (S − x)2 parax ∈ R.RESOLUÇO: Como f ′(x) = 2x−2(S−x) =4x − 2S, o ponto ríti o é x0 = S/2. Como afunção de res e antes de x0 (f ′ < 0) e res edepois, este ponto é de mínimo lo al e global (naverdade f(x) → +∞ quando x → ±∞). Neste aso y0 = S − x0 = S/2 = x0.(b) MODELAGEM: Queremos máximo e mí-nimo de g(x) = x− x3 para x ∈ [0,+∞].RESOLUÇO: Como g′(x) = 1 − 3x2, ospontos ríti os são ±

√3/3. Mas somente x0 =√

3/3 ∈ [0,+∞]. Pelo sinal da derivada, x0 émáximo lo al e positivo. Note que g(0) = 0,lim

x→+∞g(x) = −∞. Logo o máximo é em x0 eNO existe mínimo.( ) MODELAGEM: Queremos o máximo eo mínimo de x + y om xy = P > 0. As-sim queremos o mínimo de h(x) = x +

P

x om

x ∈ (0,+∞).RESOLUÇO: Como h′(x) = 1 − P

x2, ospontos ríti os são ±

√P . Mas somente √

P ∈(0,+∞) Note que h(x) → +∞ quando x → 0ou x → +∞. Assim o mínimo é em x0 =

√P , om y0 = x0 =

√P e a soma mínima igual a

2√P . O máximo NO existe.Extra 13. (a) MODELAGEM: Seja x o lado doquadrado na base e h a altura. Logo V = hx2.A quantidade de material é propor ional a somadas áreas dos lados A = x2+4xh ( aixa aberta).Como hx =

V

x, queremos minimizar A(x) =

x2 +V

xpara x ∈ (0,+∞).RESOLUÇO: Como A′(x) = 2x − V

x2, oúni o ponto ríti o é x0 = 3

V/2. Note queA(x) → +∞ quando x → 0 ou x → +∞. Assimx0 é um ponto de mínimo (pode-se ver tambémpelo sinal da derivada: função de res e até x0 e res e depois).(b) MODELAGEM: Seja x o lado do qua-drado na base e h a altura. Logo V = hx2.A quantidade de material é propor ional a somadas áreas dos lados A = x2+4xh ( aixa aberta).

Como hx =A− x2

4, queremos maximizar V (x) =

xA− x2

4. Note que se h = 0, x =

√A. Assim

x ∈ [0,√A].RESOLUÇO: Como V ′(x) =

A− 3x2

4, ospontos ríti os são x = ±

A/3. Mas o úni o nointervalo [0,√A] é x0 =

A/3. Como V (0) =V (

√A) = 0 e V (x0) > 0 o máximo é em x0.( ) MODELAGEM: Seja x o lado do quadradona base e h a altura. Logo V = hx2. O ustoé C = 2(x2) + 4xh. Como hx =

V

x, queremosminimizar A(x) = 2x2 +

V

xpara x ∈ (0,+∞).RESOLUÇO: Como A′(x) = 4x − V

x2, oúni o ponto ríti o é x0 = 3

V/4. Note queA(x) → +∞ quando x → 0 ou x → +∞. Assimx0 é um ponto de mínimo (pode-se ver tambémpelo sinal da derivada: função de res e até x0 e res e depois).Extra 14. (a) MODELAGEM: Como a velo idadeé a mesma, o minimizar tempo é o mesmo que mi-nimizar aminho. Suponha que a distân ia de Qaté o espelho é a e de P até o espelho é b. Colo-que a origem no espelho no ponto que é a projeçãoortogonal de Q no espelho. Introduza x omo alo alização do ponto O e seja c o ponto que é aprojeção ortogonal de P no espelho. Por Pitágo-ras a distân ia total per orrida pelo raio em fun-ção de x é: d(x) =

√a2 + x2 +

b+ (c− x)2.Queremos o mínimo om x ∈ R.RESOLUÇO: Note que d(x) → +∞ quandox → ±∞. Assim o mínimo é no ponto om deri-vada zero. Como

d′(x) =x√

a2 + x2− c− x√

b+ (c− x)2.Se d′(x0) = 0 então:

x0√

a2 + x20=

c− x0√

b+ (c− x0)2.Note que isto impli a que sen θi = sen θr. Comoos ângulos são entre 0 e π/2, θi = θr.(b) Como a velo idade é a mesma, o minimi-zar tempo é o mesmo que minimizar aminho.Suponha que a distân ia de Q até a interfa eentre os meios é a e de P até a interfa e é b.Coloque a origem na interfa e no ponto que é aprojeção ortogonal de Q na interfa e. Introduza

x omo a lo alização do ponto O e seja c o ponto

2.4. APLICAÇO DE DERIVADA 71que é a projeção ortogonal de P no espelho. PorPitágoras al ulamos a distân ia em ada tre ho:√a2 + x2 e √b+ (c− x)2. O tempo será ob-tido dividindo distân ia pela velo idade. Assim otempo total per orrido pelo raio em função de xé t(x) = √

a2 + x2

v2+

b+ (c− x)2

v1. Queremoso mínimo om x ∈ R.RESOLUÇO: Note que t(x) → +∞ quando

x → ±∞. Assim o mínimo é no ponto om deri-vada zero. Comot′(x) =

x

v2√a2 + x2

− c− x

v1√

b+ (c− x)2.Se t′(x0) = 0 então:

x0

v2√

a2 + x20=

c− x0

v1√

b+ (c− x0)2.Note que isto impli a que sen θ2

v2=

sen θ1v1

.Extra 15. (a) Se y = t, x = ±√t2 + 1. Assima distân ia ao quadrado d(t) = (t − 2)2 + t2 +

1. Como d′(1) = 0, o ponto mais próximo é(±

√2, 1).(b) A distân ia ao quadrado é d(x) = (x −

4)2 + x6. Como d′(x) = 6x5 + 2x − 8. Umadas raízes é x = 1 e a função d′ é res ente (suaderivada é sempre positiva). Assim é a úni a raiz.Assim o ponto é (1, 1).( ) Se x = t, y = ±√8− 4t2, om t ∈

[−√2,√2]. A distân ia ao quadrado é d(t) =

(t − 1)2 + 8 − 4t2. Logo d′(t) = −6t − 2. As-sim d′(−1/3) = 0. Agora temos que omparard(−1/3) = 28/3 ≈ 9.33 om d(

√2) = (

√2 −

1)2 ≈ 0.17 e d(−√2) = (−

√2 − 1)2 ≈ 5.28.Logo o ponto mais perto é (

√2, 0).(d) A distân ia ao quadrado é d(x) = (x −

2)2 + x para x > 0. Assim d′(x) = 2x − 3. As-sim d′(3/2). Devemos omparar d(3/2) = 7/4 om d(0) = 4 (d(+∞) = +∞, o outro ex-tremo do intervalo). Assim o ponto mais pertoé (3/2,√

3/2).Extra 16.MODELAGEM: Seja 2x o lado do re-tângulo no diâmetro do semi ír ulo e y o outrolado. A área é A = 2xy. Note que ligando-se o entro do ír ulo a um vérti e do retângulo obte-mos um triângulo retângulo om lados x, y,R.Assim, por Pitágoras, x2 + y2 = R2. Logo,y =

√R2 − x2 para x ∈ [0, R]. Assim que-remos o máximo de A(x) = 2x

√R2 − x2 para

x ∈ [0, R].

RESOLUÇO: Como A′(x) = 2R2 − 2x2√R2 − x2

.A raiz positiva será x0 =R√2. Como x20 + y20 =

R2, y0 = x0 =R√2. Note que A(0) = A(R) = 0.Assim o máximo será em x0. Logo as dimensõessão 2x0 = R

√2 e y0 =

R√2.Extra 17.MODELAGEM: Sejam h e r os ladosdo retângulo. Quando giramos o retângulo emtorno do lado de tamanho h obtemos um ilindrode volume V = πr2h. Como P = 2h + 2r,

h = P/2−r. Assim queremos maximizar V (r) =πr2(P/2 − r). Se h = 0, r = P/2. Assim r ∈[0, P/2].RESOLUÇO: Como V ′(r) = πr(P−3r), ospontos ríti os são 0 e P/3, ambos no intervalo.Mas V (0) = V (P/2) = 0. Assim r0 = P/3 é oponto de máximo. Então h0 = P/2− r0 = P/6.Assim r0

h0= 2.Extra 18. (a) MODELAGEM: Seja r o raio e ha altura do ilindro ins rito na esfera. O volumedo ilindro é V = πr2h. Como é função de duasvariáveis, devemos eliminar uma delas.Ligando-se o entro da esfera até um dos pon-tos do ír ulo da base do ilindro observamos otriângulo retângulo om hipotenusa R e atetos

r e h/2. Logo, por Pitágoras, (h/2)2 + r2 = R2,Assim, r2 = R2 − h2/4. Logo V (h) = π(R2 −h2/4)h. Note que h ∈ [0, 2R]. Assim queremoso máximo de V (h) para h ∈ [0, 2R].RESOLUÇO: Note que V (0) = V (2R) = 0.Como V ′(h) = π(R2 − 3h2/4), o ponto ríti opositivo é h0 = 2R/

√3 que perten e ao intervalo(√3 > 1). Este será o ponto de máximo pois afunção é positiva em h0.(b) MODELAGEM: Seja r o raio e h a alturado one ins rito no one. O volume do one é

V =1

3πr2h. Note que omo 1

3π é um onstante,maximizar a função f = r2h é um problema equi-valente. Como é função de duas variáveis, deve-mos eliminar uma delas. Por semelhança de tri-ângulos, H

H − h=

R

r. Logo, h = H(1 − r/R).Assim queremos maximizar f(h) = r2H(1−r/R)para r ∈ [0, R].RESOLUÇO: Como f ′(r) = H/Rr(2R −

3r). As raízes são 0 e 2R/3, ambas no intervalo[0, R]. Mas f(0) = f(R) = 0. Assim o máximoé em r0 = 2R/3.Extra 19.MODELAGEM: Chame de x a distân-

72 CAPÍTULO 2. RESPOSTAS DOS EXERCÍCIOS ia da base da es ada até a base da er a, de ya distân ia da base do prédio até o ponto ondea es ada en osta no prédio e de d o tamanho daes ada. Por Pitágoras (x + L)2 + y2 = d2. Porsemelhança de triângulos, H/x = y/(x+L). As-sim, y = H(x+L)/x. Utilizando Pitágoras obte-mos que d2(x) = (x+L)2(1+H2/x2). Queremosminimizar d2(x) para x ∈ (0,+∞).RESOLUÇO: É fá il ver que d2(x) → +∞nos extremos deste intervalo. Assim o mínimo éno ponto onde a derivada se anula. Derivandod2(x) obtemos 2(L+ x)(x3 − LH2)

x3. As raízesreais são x0 = −L (des artada pois está fora dointervalo de minimização) e x0 =

3√LH2. Por-tanto a menor distân ia é d(

3√LH2).2.4.4 DesaosDesao 1.Para todos a resposta é k.Desao 3.O quadrado da distân ia entre o ponto

(s, f(s)) e (a, b) é d(s) = (s− a)2 +(f(s)− b)2.Assim o mínimo será no ponto onde f ′(s0) =0 = 2(s0−a)+2(f(s0)− b)f ′(s0), isto é quandof(s0)− b = −1/f ′(s0)(s0−a). Como f ′(s0) é o oe iente da reta tangente no ponto (s0, f(s0),−1/f ′(s0) é o oe iente da reta normal nesteponto. Assim obteremos a identidade f(s0) −b = −1/f ′(s0)(s0 − a) se, e somente se, (a, b)perten er a reta normal neste ponto. Portanto oponto mais perto de (a, b) no grá o de f é oponto onde a reta normal inter epta (a, b).Desao 4.Queremos minimizar a distân ia aoquadrado: f(x) = (x−x0)

2+(y(x)−y0)2, onde

y(x) é denida impli itamente por ax+ by(x) +c = 0. Derivando impli itamente, a+by′(x) = 0.Como f ′(x) = 2(x − x0) + 2(y(x) − y0)y

′(x),vamos obter w ∈ R tal que f ′(w) = 0. Resol-vendo: (w − x0) + (y(w)− y0)y′(w) = 0. Como

y′ = −a/b e y(x) = −(ax+ c)/b, temos que re-solver. (w−x0)+(−(ax+x)/b−y0)(−a/b) = 0.Com o Maxima obtemos que:w =

−aby0 + b2x0 − ac

b2 + a2.Como (w−x0)+ (y(w)− y0)y

′(w) = 0, (y(w)−y0)y

′(w) = −(w − x0) e y′ = −a/b,(y(w) − y0)

2(w − x0)2(a2/b2).Logo,

f(w) = (w − x0)2(1 + a2/b2).

Agora om o Maxima obtemos quef(w) =

(ax0 + by0 + c)2

a2 + b2.Desao 5. Soma das áreas dos trapézios é (1 +

1/x)/2(x − 1) + (1/x + 1/a)/2(a − x). Noteque esta área é maior que log a. Assim queremosminimizar f(x) = (a−1) x2+a2−a2 ax − log a. Cal u-lando f ′(x) = (a−1) x2−a2+a

2 ax2 . Logo f ′(x0) = 0 ex0 > 0 se x0 =

√a.Desao 6. (a) MODELAGEM: Seja r o raio e ha altura do one ins rito na esfera. Área lateraldo one é A = πrl, onde l é o omprimento dalateral do one, que por Pitágoras satisfaz l2 =

h2 + r2. Maximizar A é o mesmo que maximizarA2 = π2r2l2 = π2r2(h2 + r2).Ligando-se o entro da esfera até um dos pon-tos do ír ulo da base do one observamos o tri-ângulo retângulo om hipotenusa R e atetos r eh−R. Logo, por Pitágoras, (h−R)2+ r2 = R2,Assim, r2 = 2hR − h2.Assim queremos maximizarA2(h) = π2(2hR−h2)2hR = 2π2Rh2(2R − h) para h ∈ [0, 2R].RESOLUÇO: A2(h) vale zero nos extremosdo intervalo. A derivada de A2(x) em relação ax é 2π2Rh(4R − 3h). Assim os pontos ríti ossão h = 0 e h = 4R/3. Logo o máximo é emh = 4R/3, o mesmo resultado que obtemos semaximizarmos o volume ao invés da área.(b) MODELAGEM: Seja r o raio e h a alturado ilindro ins rito no one. A área lateral do ilindro é A = 4πr2 + 2πrh. Como é funçãode duas variáveis, devemos eliminar uma delas.Note que ortando o one temos uma semelhançade triângulos: a altura H do one está para Rassim omo H − h está para r. Assim, H

R=

H − h

r. Logo, h = H(1− r/R). Logo queremosmaximizar A(r) = 4πr2 + 2πrH(1 − r/R) para

r ∈ [0, R].RESOLUÇO: Note que A(0) = 0 e queA(R) = 4πR2 é andidato a máximo. ComoA′(r) = 2π/R(HR−r(2H−4R)), o úni o ponto ríti o é r0 =

HR

2H − 4R.Agora pode-se provar que r0 ∈ [0, R] se, esomente se, H ≥ 4R. Neste aso o máximo seráem r0 e A(r0) =

πH2R

2H − 4R( al ulei om Ma-xima, não pre isa al ular).

2.4. APLICAÇO DE DERIVADA 73Por outro lado, se H < 4R então r0 6∈ [0, R]e o máximo será em r = R, om área igual aA(R) = 4πR2.Desao 7.MODELAGEM: Suponha que o raiodo ír ulo é 1. Colo ando a origem no entro do ír ulo queremos partir de θ = 0 hegar em θ =π. Devemos nadar em linha reta de θ = 0 até ϕ edepois orrer na beira do lago até θ = π. O per- urso nadando possui omprimento igual a distân- ia entre (cos 0, sen 0) = (1, 0) e (cosϕ, senϕ).Assim distân ia de nado ao quadrado: (cosϕ −1)2 + (senϕ)2. A distân ia orrendo é o ar o de ír ulo de ângulo π−ϕ. Colo ando omo velo i-dade 1 e 2 para nadar e orrer respe tivamente, otempo t(ϕ) =

(cosϕ− 1)2 + (senϕ)2 + (π −ϕ)/2. Note que ϕ ∈ [0, π].RESOLUÇO: t′(x) = −

√2−2 cos(x)−2 sen(x)

2√

2−2 cos(x).Assim queremos determinar x0 tal que√

2− 2 cos x0 = 2 sen x0. Assim, x0 = 0 oux0 = 2π/3. Comparando t(0) = π/2 ≈ 1.57,t(π) = 2 e t(2π/3) = π/6 +

√3 ≈ 2.25. Assimo melhor é orrer diretamente para o outro ladodo lago e levar tempo t(0).Desao 8.Queremos que o ír ulo (x − a)2 +

(y(x) − b)2 = r2 os ule a urva. Derivando im-pli itamente obtemos que (x−a)+ (y− b)y′ = 0e 1 + (y − b)y′′ + (y′)2 = 0. Para que o ír uloos ule, quando x = c: y = η, y′ = η1, y′′ = η2.Temos que determinar a, b, r tais que:

(c− a)2 + (η − b)2 = r2,(c− a) + (η − b)η1 = 0,1 + (η − b)η2 + (η1)

2 = 0.Resolvendo, obtemos r (se quiser a e b também).Veja [Ha, p.299 e [Co, p.333 e p.283.2.4.5 ⋆Problemas (Taxas Rela iona-das)Problema 1.O volume V (r) = 4/3πr3. AssimV ′(r) = 4πr2. Seja r(t) o raio do balão emfunção do tempo. Por hipótese r′(t) = −15 paratodo t. Seja f(t) = V (r(t)) a variação no volumeem função do tempo. Então f ′(t) será a taxa om o ar estará saindo do balão. Então, f ′(t) =V ′(r(t))r′(t) = −15(4πr2(t)). Assim se r(t) =9, f ′(t) = −15(4π81) = −4860.Problema 2. Seja O o ponto no solo verti al-mente abaixo do balão. Se h(t) é altura do solo,x(t) distân ia de O até o arro, e d(t) a distân ia

balão- arro, por Pitágoras h(t)2 + x(t)2 = d(t)2.Logo 2hh′ + 2xx′ = 2dd′. Após 4 segundos:h(4) = 48 + 4 · 3 = 60, x(4) = 0 + 20 · 4 =80. Por Pitágoras, d(4) = 100. Como h′(4) =3 e x′(4) = 20. substituindo na equação (ob-tida por derivação implí ita) obtemos: 2(60)(3)+2(80)(20) = 2(100)d′(4). Logo a variação da dis-tân ia vale d′(4) = 17, 8m/s.Problema 3. Seja d a distân ia da fonte até aparede e r o raio da área iluminada. Como oângulo é de 90o, a o triângulo retângulo ujos atetos são r e d é isós eles. Assim r = d. LogoA(t) = πr(t)2 = πd(t)2. Assim, A′ = 2πdd′.Logo A′ = 2πk(−a) = −2kaπ.Problema 4.Por semelhança de triângulos, se r éo raio de água do tanque om h metros de profun-didade, h(t) = r(t)H/R. Assim o volume v(t) =4/3πr2(t)h(t). Assim v′(t) = 4/3π(2rr′h+r2h′)e h′ = r′H/R. Assim se v′ = V , h = L entãor = LR/H e r′ = h′R/H.V = 4/3π(2LR/Hh′R/HL+(LR/H)2h′). Bastaresolver para h′.Problema 5. Seja x a distân ia da base da es adaaté a parede e y a altura do topo da es ada. PorPitágoras, x2+y2 = L2. Assim, 2xx′+2yy′ = 0.Como x′ = V e quando y = H, x =

√L2 −H2,

V√L2 −H2 +Hy′ = 0. Basta resolver para y′,a velo idade do topo: y′ = −H/(V

√L2 −H2).Problema 6. (a) Seja a(r) = πr2 a área do ír- ulo. Então a′(r) = 2πr. Assim f(t) = a(r(t)) éa variação da área em função do tempo e f ′(t) =

a′(r(t))r′(t) = 2πr(t)r′(t). Logo se r′(t) = k,f ′(t) = 2kπr(t).(b) Seja V (x) = x3 o volume do ubo. EntãoV ′(x) = 3x2. Assim, f(t) = V (x(t)) é a vari-ação do volume em função do tempo e f ′(t) =V ′(x(t))x′(t) = 3x2(t)x′(t). Logo se x′(t) = k,f ′(t) = 3kx2(t) = k/2(6x2(t)), onde 6x2(t) é aárea de superfí ie.( ) Aqui V (r) = kr3 (k = 4/3π). AssimV ′(r) = 3kr2. Assim, f(t) = V (x(t)) é a vari-ação do volume em função do tempo e f ′(t) =V ′(r(t))r′(t) = 3kr2(t)r′(t). Suponha que f ′(t) =C4πr2(t). Então, C4πr2(t) = 3kr2(t)r′(t). Logo,r′(t) =

C4π

3k, uma onstante.

74 CAPÍTULO 2. RESPOSTAS DOS EXERCÍCIOS2.4.6 ⋆Problemas (Derivação Implí- ita)Problema 1. (a) y′(1) = −5/38 e a reta tan-gente é y = 5− 5/38(x − 1).(b) Derivando impli itamente obtemos 2x =−2y + 2y′x(x− y)2

. Substituindo obtemos que y′ = 1.Assim a reta tangente é y = x+ 1.Problema 2.Derivando impli itamente obtemosque y′ =y − x2

y2 − x. Assim a reta tangente seráhorizontal quando y′ = 0, isto é, quando x = y2.Substituindo em x3 + y3 = 3xy, obtemos que

y6 = 2y2, ujas raízes reais são y = 0 e y =3√2. Obtemos o x orrespondente substituindona equação x3 + y3 = 3xy.A reta será verti al quando y′ = ±∞. Assimbasta que o denominador y2 − x se anule, isto é,

x = y2. Substituindo em x3 + y3 = 3xy, obte-mos, de forma análoga, x6 = 2x2, ujas raízesreais são x = 0 e x = 3√2. Obtemos o y orres-pondente substituindo na equação x3+y3 = 3xy.Problema 3.A derivada implí ita é 2x− 2yy′ +

1

2√xy

(y + xy′) = 0.(a) Queremos y′(2) = f ′(2). Substituindox = 2 e y = 2, obtemos que4 − 4y′(2) +

1

4(2 + 2y′(2)) = 0. Logo, y′(2) =

f ′(2) =9

7.(b) y − 2 =

9

7(x− 2).( ) g′(x) =f ′(x)x− f(x)

x2. Logo g′(1) =

18/7 − 2

4=

1

7.Problema 4. (a) De res ente, f ′(1) = −1,

f ′′(1) = −19/3.(b) De res ente, f ′(1) = −π/2, f ′′(1) = π.Problema 5.Derivando impli itamente, obtemos4x3− y−xy′+4y3y′ = 0. Queremos determinaros pontos onde y′ = 0. Assim obtemos que y =4x3. Substituindo na equação x4−xy+y4 = 253obtemos a equação 256x12 − 3x4 = 253. Porinspeção obtemos as raízes x = ±1. As outrasraízes são omplexas (graças ao Maxima!). Assimem x = 1, y = 4x3 = 4 e em x = −1, y = 4x3 =−4. O máximo é y = 4 e o mínimo é y = −4.Use um software para plotar a função implí itae veri ar esta resposta. Este método fun iona

pois a urva denida pela equação é limitada.Problema 6.Como (1, 1) perten e a urva, 1 +a = b. A derivada implí ita é: 2xy + x2y′ +2ayy′ = 0. Logo em x = 1, y = 1, 2 + y′(1) +2ay′(1) = 0 ou (2a+1)y′(1) = −2. Logo y′(1) =−2

2a+ 1. Queremos que seja igual ao oe ienteangular de 4x + 3y = 7, que é −4/3. Assim

y′(1) =−2

2a+ 1= −4/3. Logo, a =

1

4e b =

1 + a =5

4.2.5 Integral2.5.1 Exer í ios de FixaçãoExer í io 1. (a) Falso. A função pode ser posi-tiva num intervalo e negativo em outro de modoque as áreas se an elam. Exemplo ∫ 2π

0senx dx =

0 mas sen(x) não é zero para todo x.(b) Verdadeiro, pelo Lema 8 da p.139 (mo-notoni idade da integral).( ) Falso. Podemos mudar a integral num nú-mero nito que o valor da integral será mantido.Exer í io 2. (a) Pela Denição 25 da p.139,∫ −1

2f(x) dx = −

∫ 2

−1f(x) dx = −5.(b) Utilizando a linearidade, 5+2(−3) = 5−

6 = −1.( ) Pela Denição 25 da p.139, a integral é 0(mesmos limites de integração).(d) Pelo Lema 8 da p.139, item ( ),∫ 2

−1(· · · ) =

∫ 0

−1(· · · ) +

∫ 2

0(· · · ).Assim,

∫ 2

0f(x) dx =

∫ 2

−1f(x) dx−

∫ 0

−1f(x) dx =

= 5− 7 = −2.(e) Note que g(t) é onstante na integral emds. Assim,∫ 0

−1f(s)g(t) ds = g(t)

∫ 0

−1f(s) ds = g(t)7.026.jul.2010

2.5. INTEGRAL 75Assim,∫ 2

−1

(∫ 0

−1f(s)g(t) ds

)

dt =

∫ 2

−1(g(t)7) dt

= (−3)7 = −21.(f) Mudar a função em um úni o ponto NOaltera o valor da integral. Assim ∫ 2

−1h(x) dx =

∫ 2

−1f(x) dx = 5.Exer í io 3. (a) F (0) = 0, F (1) = 2 (área doretângulo), F (2) = F (1)+1 = 3, F (3) = F (2)−

1/2 = 5/2.(b) F vai res er onde f é positiva, pois aárea vai aumentar. Assim, F res e em (0, 2) edepois de x = 5 e de res e em (2, 5).( ) Máximo lo al em x = 2 pois estava res- endo e passa a de res er e mínimo lo al emx = 5 pois estava de res endo e passa a res- er.Exer í io 4. (a) h(2) = ∫ 2

2(. . .) = 0.(b) Pelo TFC, h′(x) =(5− x)5

x4 + 6. Assim osinal de h′ é igual ao sinal de 5−x. Logo h′(x) >

0 (e h res e) se x < 5 e h de res e para x > 5.( ) somente em x = 5 a derivada é zero.Como h′ é positiva antes e negativa depois, x = 5é máximo lo al.Exer í io 5.Pelo Corolário 10 da p.141,∫ 1

−1h(s) ds = g(1) − g(−1) =

= (Ke+B −C)− (Ke−1 −B − C) =

= K(e− e−1) + 2B.Exer í io 6. (a) a primitiva é x4/2−x3 +5x. Oresultado é 9/2.(b) No intervalo [0, 1], y2 − 1 é negativo.Assim, |y2 − 1| = 1 − y2. Logo, a primitiva éy − y3/3. O resultado é 2/3.( ) Note que a integral é em t. Logo, x é onstante nesta integral. Assim,∫

(3x+ et − 7x sen t) dt = 3xt+ et + 7x cos t.Exer í io 7.(a) ∫ 5

−∞e−s2 ds = lim

k→−∞

∫ 5

kf(s) ds.(b) ∫ 2

0

dx

log(5− x2)= lim

k→2−

∫ k

0

dx

log(5− x2).

( ) ∫ +∞

0

dy

1 + y4= lim

k→+∞

∫ k

0

dy

1 + y4.(d) ∫ 2

1

dx

x10 − 1= lim

k→1+

∫ 2

k

dx

x10 − 1.Exer í io 8. (a) Tome x = t/c. Logo dx = dt/c.Logo, dt = c dx. Assim, quando t = ac, x = a equando t = bc, x = b.(b) Tome u = x− c. Logo du = dx. Assim,quando x = a + c, u = a e quando x = b + c,

u = b.Exer í io 9. (a) Tome u = K − 3x. Então du =

−3dx. Assim devemos integrar ∫ 4√u(−du/3) =

−4/15u5/4. Logo a integral é −4/15(K−3x)5/4 .(b) Tome u = x3. Então du = 3x2dx. Assimdevemos integrar ∫ cos u du = senu = sen(x3).( ) Tome u = log t. Então du = dt/t. Assimdevemos integrar ∫ u2du = u3/3 = log3 t/3.(d) Tome u = 3−2x2. Então, du = −4x dx.Assim devemos integrar∫ √

u (−du/4) = −u3/2

6= −(3− 2x2)3/2

6.(e) Tome u = cos θ. Então, du = − sen θ dθ.Assim devemos integrar

− du√u= −2

√u = −2

√cos θ.(f) Tome u = 5 senx. Então, du = 5cos x dx.Assim devemos integrar

eu (du/5) =eu

5=

e5 senx

5.Exer í io 10. (a) Tome u = log x e dv = x.Assim, du = dx/x e v = x2/2. Logo,

x log x dx =x2 log x

2−∫

x2

2

dx

x.Esta última integral a ∫ x

2dx =

x2

4. Logo,obtemos que

x log x dx =x2 log x

2− x2

4.(b) (Veja exemplo de integral de arcsen nap.149). Tomando u = arctan x e dv = 1 · dx,

du =dx

x2 + 1e v = x. Logo,

arctan x dx = x arctan x−∫

x

x2 + 1dx.

76 CAPÍTULO 2. RESPOSTAS DOS EXERCÍCIOSAgora vamos resolver a integral tomando z =x2 + 1, dz = 2x dx. Logo,∫

x√x2 + 1

dx =

dz

2z=

log z

2=

log(x2 + 1)

2.Juntando tudo obtemos,

arctan x dx = x arctan x− log(x2 + 1)

2.Exer í io 11. (a) Tome u = 3−2x. Assim du =

−2dx. Logo,∫

(3− 2x)4 dx =

u4(−du

2) = −u5

10=

= −(3− 2x)5

10.Substituindo os limites de integração:

∫ 2

1(3− 2x)4 dx = −(3− 2x)5

10

2

1

=

= −(3− 2(2))5

10+(3− 2(1))5

10=

1

10+

1

10=

1

5.(b) Tome u = −x/4. Assim du = −dx/4.Logo,

eu(−4) du = −4eu = −4e−x/4.Substituindo os limites de integração:∫ +∞

log 3e−x/4 dx = −4e−x/4|+∞

log 3 =

= 4(14√3− e−∞/4) =

44√3.( ) A primitiva é −cos(2θ)

2. Logo,

∫ π

π/4sen(2θ) dθ = −cos(2π)

2+

cos(π/2)

2= −1

2.(d) A primitiva é − 1

2s2. Logo a integral vale

lims→+∞

− 1

2s2+

1

2= 0 +

1

2=

1

2.

2.5.2 ProblemasProblema 1.Primeiro esbo e o grá o:x

y

1 2 3 4 5−1

1

4

Agora al ule as integrais determinando asáreas om sinal.(a) Área do triângulo igual a 4 menos a áreado quadrado igual a 1. Logo a integral é 4−1 = 3.(b) Área do retângulo ( om sinal negativo):−2.( ) Área do trapézio igual a 5/2 menos a áreado retângulo igual a 2 mais área do triângulo iguala 1/2. Logo a integral é 5/2 − 2 + 1/2 = 2.Problema 2. (a) Basta apli ar a (monotoni idadeda integral) do Lema e observar que ∫ b

aM dx =

M

∫ b

adx = M(b− a).(b) Novamente, note que

sen(qualquer oisa) ≥ −1. Como log é res- ente, seu menor valor em [e, 5e] é log e = 1.Assim a função é limitada inferiormente por m =−4. De forma análoga ao item (a), limitamos aintegral por baixo por m(6e−e) = 5em = −20e.Problema 3.Denindo h(x) =

∫ x

0sen(t2) dt,queremos determinar a derivada de g(x) = h(e2x).Pelo Teorema 5.2 da p.140 (TFC), h′(x) = sen(x2).Utilizando a regra da adeia, a derivada de g(x) =

h(e2x) é g′(x) = h′(e2x)(e2x)′ = sen((

e2x)2)2e2x.Logo, g′(x) = sen(e4 x)2e2x.Problema 4.Pelo Teorema 5.2 da p.140 (TFC)e pela regra da adeia,

y′(x) =2x cos(x2 − π)

2 + sen(x2 − π)+ cos(x2). Logo,

y′(√π) =

√π + 1. Assim a equação da retatangente é y − log(2) =

√π(x − √

π) ou y =√πx+ log(2) − π.Problema 5. (a) Dena

H(y) =

∫ y

1et

3

dt e G(k) =

∫ k

4cos(1 + s2) ds.

2.5. INTEGRAL 77Agora pelo TFC, H ′(y) = ey3 e G′(k) = cos(1+

k2). Assim, omo f(y) = G(H(y)), pela regra da adeia, f ′(y) = G′(H(y))H ′(y). Logo, f ′(1) =G′(H(1))H ′(1). ComoH(1) = 0, f ′(1) = cos(1)e.(b) Dena J(x) =

∫ x

8log(t3 + 1) dt. As-sim, g(y) = ∫ 5

yJ(x) dx = −

∫ y

5J(x) dx. Logo,pelo TFC, g′(y) = −J(y) = −

∫ y

8log(t3+1) dt.Assim, g′(8) = 0.Problema 6. (a) A primitiva é x1−p

1− p. Logo aintegral vale 1

p− 1+ lim

N→+∞N1−p

1− p. Para que olimite seja nito (na realidade será zero) o expo-ente de 1−p deve ser negativo. Assim, 1−p < 0,o que impli a que 1 < p. Assim a integral seránita se p > 1 e valerá 1

p− 1.(b) Novamente a primitiva é x1−p

1− p. Logo aintegral vale 1

1− p+lim

k→0

k1−p

1− p. Para que o limiteseja nito (na realidade será zero) o expoente de

1− p deve ser positivo. Assim, 1− p > 0, o queimpli a que p < 1. Assim a integral será nita se0 < p < 1 e valerá 1

1− p.Problema 7. (a) Substitua u =

√k.R: 2 sen(√k) + C.(b) Substitua u = 1− 3x2.R: −√

1− 3x2 + C( ) Deverá ser feita a substituição u = 3x+1.Depois uma integração por partes tomando z = xe dw = sen(u).R :

sen(3x+ 1)− 3x cos(3x+ 1)

9+ C.(d) Tome u = senx, du = cos x dx. Assim aintegral se transforma em ∫ udu =

u2

2.R: sen2x

2+ C.(e) Substitua u =

√t. Depois obterá umaintegral do tipo ∫ u senu du, que deverá ser re-solvida integrando por partes.R: 2 sen(√x)− 2√x cos(

√x) + C.(f) Veja té ni a do Exemplo na página 149(integrar por partes duas vezes seguidas).R: 2/5 e2 x cos(x) + 1/5 e2 x sen(x) +C(g) Tome u = log x. Logo, du = dx/x.Como x = eu, dx = eu du. Portanto temos que

integrar ∫ eu sen(u) du. Veja té ni a do Exem-plo na página 149 (integrar por partes duas vezesseguidas).R :

x (sen (log (x))− cos (log (x)))

2+ C(h) Substitua u = 3

√s. Depois obterá umaintegral do tipo ∫ ueu du, que deverá ser resolvidaintegrando por partes.R: 2 (3

√s− 1) e3

√s

9+ C.(i) Substitua u = ex. Vai obter ∫ du

1 + u2=

arctan u.R: arctan(ex) + C.Problema 8. (a) Primitiva: −e−x2

2. R: 1− 1/e

2.(b) Primitiva: −(3s + 1)e−3s

9. R: 1− 4e−3

9.( ) Primitiva: − 1

2(log x)2. R: 1/2.(d) Primitiva: −2 se−1/2 s − 4 e−1/2 s; R: 4.(e) Primitiva: 2(ex + 1)3/2

3. R: (16−4

√2)/3.(f) Primitiva: log(1 + x2)

2. R: log(2)/2.(g) Primitiva: −2(1 + 1/x)3/2

3. R: 37

24

√2.Problema 9. (a) Separe na integral de 0 até 2 de

x2(2− x) e de 2 até 4 de x2(x− 2). R: 24.(b) Separe na integral de 1/2 até 1 de − log se de 1 até 2 de log s. R: (3 log(2) − 1)/2.( ) Note que ey − 1 > 0 se y > 0. Logoes−1 − 1 > 0 se s − 1 > 0 e aso ontrário seránegativo. Assim al ule −2 até 1 ∫ 1

−21−es−1 ds, uja primitiva é es−1 − s e some om ∫ 2

1es−1 −

1 ds, uja primitiva é s− es−1 − s. R: e+ e−3.Problema 10. (a) Integrando obtemos quey(x) =

4x3/2 + 6√x

3+ C. Como queremos que

y(1) = 10/3 + C = 0, C = −10/3. Assim,y(x) =

4x3/2 + 6√x

3− 10

3.(b) Substitua u = x2 + 1. Vamos obter

y(x) =ex

2+1

2+ C. Como queremos que y(1) =

e2

2+ C = e2, C =

e2

2. Assim, y(x) = ex

2+1

2+

e2

2.

78 CAPÍTULO 2. RESPOSTAS DOS EXERCÍCIOSProblema 11. (a) A primitiva é 4x3/4

3. R: 32/3.(b) Tome u = log x e faça a substituição. Aprimitiva é − 1

2 log2 x. R. 1/2( ) Tome u = 3 − 2ex e faça a substituição.A primitiva é −3− 2ex

2. R: log 3

2.Problema 12. (a) Note que trata-se de um limitedo tipo 0 vezes innito. Assim, es revendo omoo quo iente da integral por ex2 podemos apli arL'Hospital. Derivando a integral om o TFC ob-temos que o limite é igual ao limite

limx→+∞

log(x9 + 3)

2xex2.Colo ando 1

2xem evidên ia e apli ando L'Hospitalmais uma vez vamos obter o limitelim

x→+∞9x8

(x9 + 3)2xex2.Agora omo exponen ial vai mais rápido para in-nito que polinmio (ou apli ando L'Hospital umas8 vezes mais), on luímos que o limite vale 0.(b) Derivando os dois lados, utilizando o TFC,obtemos que e−xf ′(x) = 3 ou f ′(x) = 3ex. As-sim, integrando, obtemos que f(x) = 3ex + C.Como f(0) = 1 = 3 + C, C = −2. Logo,

f(x) = 3ex − 2.2.5.3 ExtrasExtra 1.Pelo TFC, Si′(x) = sen(x)

x. A derivadaé zero em x = kπ om k ∈ Z∗ (sem o zero, poispelo limite fundamental, Si′(0) = 1). Para saberse é máximo ou mínimo temos que ver o sinal daderivada antes e depois destes pontos. Para x >

0 vamos ter os máximos lo ais em x = 2kπ + πpara k ∈ N. Para x < 0 vamos ter os máximoslo ais em x = 2kπ para k ∈ N.Extra 2. (a) Pelo TFC, f ′(x) = log(ex + x −1)ex

2 . Logo, f ′(1) = e. Como f(1) = 0 (pois∫ 1

1(· · · ) = 0), a reta tangente é y = e(x− 1).(b) Pelo TFC, h′(x) = ex

x2 + 1. Logo, h′(2) =

e2

5. Como h(2) = 7, a reta tangente é y − 7 =

e2

5(x− 2).Extra 3. Sabemos que a a eleração a(t) é igual aderivada da velo idade v(t) que é igual a derivada

da posição x(t). Assim, v′ = a, x′ = v. Comov′(t) = a(t) = Aω2 cos(ωt),v(t) =

a(t) dt =

Aω2 cos(ωt) dt =

= Aω sen(ωt) + C.Como v(0) = 0 = Aω sen(ω0) +C = C, C = 0.Como x′(t) = v(t),x(t) =

v(t) dt =

Aω sen(ωt) dt =

= −A cos(ωt) + C.Como x(0) = −A cos(ω0) + C = 0 = −A + C.Assim, C = A. Logo, x(t) = −A cos(ωt) +A.Extra 4. Seja h(y) =

∫ y

0

sen(t)

t2dt. Note que

f(s) = h(s2)−h(s) (porque?). Pelo TFC, h′(y) =sen(y)

y2. Assim, pela regra da adeia. f ′(s) =

2sh′(s2)− h′(s) =2s sen(s2)

s4− sen(s2)

s4.Extra 5.Pelo Teorema 5.2 da p.140 (TFC),

F ′(x) =x2 − 1

x2 + 1. O sinal de F ′ será determinadopelo numerador pois o denominador é sempre po-sitivo.(a) F é res ente em x > 1 e x < −1; F éde res ente em (−1, 1).(b) F ′′(x) =

4x

(x2 + 1)2. Assim a on avi-dade é para ima em x > 0 e para baixo em

x < 0.( ) A derivada é zero em ±1. Mas o mínimolo al é em x = 1 pois a on avidade do grá aé para ima neste ponto. O máximo lo al é emx = −1 onde a on avidade é para baixo.Extra 6.Tome u = 2x + 1. Então du = 2dx.Logo quando x = 0, u = 1; quando x = −1, u =

−1. Logo, ∫ 0

−1f(2x + 1) dx =

∫ 1

−1f(u) du/2.Agora, pelas propriedades da integral,

∫ 1

−1=

∫ 3

−1−∫ 3

1.Assim ∫ 1

−1f(u) du/2 = (1/2)(7− 3) = 4/2 = 2.Extra 7. (a) Integrando obtemos que

y(θ) =sen(5x)

5+ 3x+ C. Como y(π) = 15π +

2.6. APLICAÇÕES DA INTEGRAL 79C = 5π, C = −10π. Logo, y(θ) =

sen(5x)

5+

3x− 10π.(b) Integrando obtemos quey(x) =

log(2x+ 1)

2+C. Como y(0) = 0+C =

3, C = −3. Logo, y(x) = log(2x+ 1)

2− 3.Extra 8.Temos que y′(x) = √

x+ 1 e que y(0) =1. Integrando obtemos quey(x) =

2(x+ 1)3/2

3+ C. Como y(0) = 2/3 +

C = 1, C = 1/3. Logo, y(x) =2(x+ 1)3/2

3+

1/3.Extra 9. (a) Como x2−3x+2 = (x−1)(x−2),separe em três integrais: ∫ 1

0(x2 − 3x+ 2) dx =

5/6, ∫ 2

1(−x2+3x−2) dx = 1/6 e ∫ 4

2(x2−3x+

2) dx = 14/3. R: 5/6 + 1/6 + 14/3 = 17/3.(b) Separe na integral de ∫ 0

−3

√1− x dx =

14/3 e ∫ 3

0

√1 + x dx = 14/3. R: 14/3+14/3 =

28/3.( ) Como x2 − 2x = x(x − 2), separe naintegral de ∫ 0

−2x(x2 − 2x) dx = −28/3 mais

∫ 2

0x(2x− x2) = 4/3dx R: −28/3 + 4/3 = −8.Extra 10. (a) 1/3x3 log(x)− 1/9x3 + C.(b) 2 (x

3

2 log x

3− 2x

3

2

9

)

+ C.( ) x2(2(log x)2 − 2 log x+ 1)

4+ C.(d) sen(4x) + 4x

8+ C.(e) − 1

ex − 1+ C.(f) e3 sen(x)+4

3+ C.(g) sen(e3 + 3) + C.Extra 11. (a) Primitiva −e1/x. R: e2 − e.(b) Primitiva (x2 − 1)ex

2

2. R: log(2)− 1.( ) Primitiva 2√x(log x−2). R: 8 log(2)−4.(d) Primitiva 2(sen θ)3/2

3. R: 2/3.(e) Primitiva−2 (1−y)

52

5 + 4 (1−y)32

3 −2√1− yR: 16/15.(f) Primitiva −2 cos(

√x). R: −4.

(g) Primitiva log(ex + 4). R: log(7/6).(h) Primitiva √t2 + 1. R: √2− 1.Extra 12. (a) A primitiva é −1 + log x

x. R: 1.(b) A primitiva é 1

4− x. R: 1

2.2.5.4 Desaos2.5.5 ⋆Problemas (Integração por Fra-ções Par iais)Problema 1.Completando o quadrado e olo- ando em evidên ia onstantes, vai apare er umaintegral do tipo ∫ dy

1 + y2= arctan y. Colo andoos limites (arctan(+∞) = π/2 e arctan(−∞) =

−π/2) vamos obter a resposta 2π√4c− b2

.Problema 2. (a) 1

b− alog

(

x− b

x− a

).(b) log(x− a)− log(x)

a2+

1

ax.Problema 3.Dena D = aB− bA e X = ax2+

2bx+ c.(a) A

2alog |X|+ D

2a√∆

log

ax+ b−√∆

ax+ b+√∆

.(b) A

2alog |X|+ D

a√−∆

arctan

(

ax+ b√−∆

).( ) − D

a(ax+ b)+

A

alog |ax+ b|.2.6 Apli ações da Integral2.6.1 Exer í ios de FixaçãoExer í io 1. (a) Uma função é a translação daoutro por 2 unidades. Assim a área é igual a

∫ 5

0(ex + 2− ex) dx =

∫ 5

02 dx = 2 · 5 = 10.(b) Está área é igual ∫ π/2

−π/2cos(x) dx = 2.Exer í io 2. (a) ∫ 2

1(g(x) − f(x)) dx.(b) ∫ 1

−1(f(x)−g(x)) dx+

∫ 2

1(g(x)−f(x)) dx.Exer í io 3. (a) ∫ 1

−1π[f(x)]2 dx.027.jul.2010

80 CAPÍTULO 2. RESPOSTAS DOS EXERCÍCIOS(b) ∫ 2

1π[g(x)]2 dx−

∫ 2

1π[f(x)]2 dx.( ) ∫ 6

3π[g−1(y)]2 dy −

∫ 2

1π[f−1(y)]2 dy.Exer í io 4.∫ 3

−2(g(y) − f(y)) dy.Exer í io 5.Pelo Teorema 26 da p.167 o volumede Ω é ∫ 4

−2A(s) ds.Exer í io 6.Pela denição de média,K =

1

b− a

∫ b

ag(x) dx.Pela monotoni idade da integral (Lema 8 da p.139), omo g(x) ≤ 5,

∫ b

ag(x) dx ≤

∫ b

a5 dx = 5(b− a).Assim, K ≤ 1

b− a5(b − a) = 5. De formaanáloga, pela monotoni idade da integral, omo

g(x) ≥ −4,∫ b

ag(x) dx ≥

∫ b

a−4 dx = −4(b− a).Assim, K ≥ 1

b− a(−4)(b − a) = −4.2.6.2 ProblemasProblema 1. (a) A interseção o orre quando y =

x2 = x− x2, ou seja, quando 2x2 − x = x(2x−1) = 0. Assim a interseção é em x = 0 e x = 1/2.Logo a área é igual a ∫ 1/2

0(x − x2 − x2) dx =

x2

2− 2x3

3

1/2

0

=1

24(b) A interseção o orre é quando cos x =senx, que o orrerá dentro de um i lo do seno([0, 2π]) em π/4 e π + π/4 = 5π/4. Assim aárea é igual a ∫ 5π/4

π/4(senx− cos x) dx =

√8.Problema 2. (a) A interseção de y = x3 = x+6é em x = 2 e y = 8. A interseção de 2y = x e

y = x+ 6 é em (−4, 2). Assim o esboço é:

x

y

y − x = 6

y − x3 = 0

2y + x = 02−4

2

8

Assim a área é:∫ 0

−4((x+6)−(−x/2)) dx+

∫ 2

0((x+6)−(x3)) dx.Como ∫ 0

−4((x+6)−(−x/2)) dx = 12 e ∫ 2

0((x+

6)− (x3)) dx = 10, a área é 22.(b) Para fa ilitar, o primeiro passo é tro arx om y e resolver o problema: Cal ule a regiãodelimitada por por x2 = 2y + 4 e por x = y − 2.Assim, y =

x2

2− 2 e y = x + 2. A interseçãoo orrerá quando y =

x2

2− 2 = x + 2, isto é se

x = −2 ou se x = 4. Assim a área é igual a∫ 4

−2((x+ 2)− (x2/2− 2)) dx = 18.

x

y

y =x2

2− 2

y = x+ 2

−2 4Resolvendo o problema original (y2 = 2x+ 4e y = x − 2) e integrando em x teríamos quees rever omo soma de duas integrais (verique):∫ 0

−22√2x+ 4 dx+

∫ 6

0(√2x+ 4− (x− 2)) dx =

=16

3+

38

3= 18.( ) Fazendo o esboço observamos que a in-terseção o orrerá em x = 0 e em x = 1. Assim a

2.6. APLICAÇÕES DA INTEGRAL 81área é igual a ∫ 1

0(1−√

x) dx =1

3. Outra possi-bilidade é integrar em y. Como y =√x, x = y2.Assim, a área é ∫ 1

0y2 dy =

1

3.x

y

y =√x

y = 1

x = 01Problema 3. Sua área será determinada por

∫ 4

−4(4+

16− 4x2−4) dx−∫

√2

−√2(6−x2−4) dx.A primeira integral é igual a metade da área do ír ulo de raio 4: 8π. R: 8π − 8

√2

3.Problema 4.Note que a região é limitada supe-riormente por y = x+1. Assim, rodando no eixo

x o volume será ∫ 1

0π(x+1)2 dx =

3. O sólidoobtido será um tron o de one.Girando em torno do eixo y vamos obter um ilindro de 1 e altura 2 menos o sólido obtidogirando x = y − 1 (já que y = x + 1) para y ∈

[1, 2]. O ilindro possui volume 2π (πr2h, omr = 1 e h = 2). Devemos subtrair ∫ 2

1π(y −

1)2 dy =π

3. Assim o volume é 2π − π

3=

3.Problema 5. (a) Primeiro o esboço.

x

y

y = 3√x

Ω

y = 2

8Sua área é igual a∫ 8

0(2− 3

√x) dx = 4.O volume será al ulado omo a diferença entredois volumes:

π

∫ 8

022 dx−π

∫ 8

0( 3√x)2 dx = 32π−π

96

5=

64π

5.

(b) Primeiro o esboço.x

y

Ωy = e−x

x = 1Sua área é igual a∫ +∞

1e−x dx =

1

e.O volume é igual a

π

∫ +∞

1(e−x)2 dxπ

∫ +∞

1(e−2x) dx =

π

2e2.Problema 6.Primeiro o esboço:x

y

x = y2x = y

1

1

x = −1

y = −1(a) Note que y =√x. Como a rotação é emtorno de y = −1, o raio maior é 1+√

x e o menoré 1 + x, ao invés de √x e x se fosse rotação emtorno de y = 0 (eixo x). O volume será dado peladiferença de volumes:

π

∫ 1

0(√x+ 1)2 dx− π

∫ 1

0(x+ 1)2 dx =

17π

6− 7π

3=

π

2.(b) De forma análoga, o raio maior é y+1 e omenor y2+1. O volume será dado pela diferençade volumes:

π

∫ 1

0(y + 1)2 dy − π

∫ 1

0(y2 + 1)2 dy =

3− 28π

15=

15.

82 CAPÍTULO 2. RESPOSTAS DOS EXERCÍCIOSProblema 7.Note que A = (1, e) pois está na urva y = ex2 , e portanto, y = e1

2

= e1 = e.Por outro lado B está parábola. Como y = 0,x = 2. Assim B = (2, 0). Assim a equação dareta L (que passa em A e B) é y = −ex + 2e,ou x = 2 − y/e. A função y = ex

2 intersepta oeixo y em y = 1.O volume será igual ao volume do tron o de one obtido ao girar a reta L para y ∈ [0, e] menoso volume ao girar y = ex2 para y ∈ [1, e] e menoso volume ao girar a parábola para y ∈ [0, 1].Invertendo as funções, omo y = ex

2 , log y =x(y)2. Assim o integrando será x2 = log y. Como4y = (x−2)2, e na região (veja gura) x−2 ≤ 0,√

(x− 2)2 = −(x − 2) = 2 − x. Assim, 2√y =2− x. Logo, x = 2− 2

√y.Assim o volume é igual a

π

∫ e

0(2− y/e)2 dy−

− π

∫ 1

0(2− 2

√y)2 dy − π

∫ e

1log y dy.Problema 8.A primeira oisa a ser observada éque a resposta NO é o volume da esfera menos ovolume do ilindro de raio a. Isto porque o nal do ilindro retirado pelo furo é arredondado (estána superfí ie da esfera).A esfera é o sólido de revolução de y = f(x) =√

r2 − x2 em torno do eixo x. Como o bura otem raio a (veja gura), o valor x = k para quef(k) = a =

r2 − x20 será k =√r2 − a2.

x

y

aa

y =√r2 − x2

k−kAssim o volume será dado porπ

∫ k

−k(√

r2 − x2)2 dx = 2kπr2 − 2πk3

3,onde k =

√r2 − a2.Problema 9.Como y = ±

√r2 − x2, o lado doquadrado para ada x é 2

√r2 − x2. A área de

ada orte A(x) = 4(r2 − x2). Assim, o volumeé∫ r

−r4(r2 − x2) dx =

16

3r3.Problema 10.Faça a gura e observe que a in-terseção é em (1, 1) e (0, 0). O lado do qua-drado para ada x é x−x2. A área de ada orte

A(x) = (x− x2)2. Assim, o volume é∫ 1

0(x− x2)2 dx =

1

30.Problema 11. (a) k2

3.(b) 2

π.2.6.3 ExtrasExtra 1. (a) Começamos om o esboço:

x

y

y = 1/x

y = x2

1

x = 2x = −1

y = −2Assim a área será:∫ 1

−1(x2 − (−2)) dx +

∫ 2

1(1/x− (−2)) dx =

14

3+ log(2) + 2 =

20

3+ log 2.(b) Note que as raízes da equação do segundograu são 0 e 2π. O esboço é:

x

y

y = 6πx− 3x2

y = cos(x)− 12π

2.6. APLICAÇÕES DA INTEGRAL 83Assim a área é∫ 2π

0(6πx− 3x2 − (cos(x)− 1)) dx = 4π3 + 2π.( ) A interseção o orrerá se y = x2 = 8−x2,isto é, quando 2x2 = 8, em x = ±2.Assim a área é ∫ 2

−2((8− x2)− x2) dx =

64

3.

x

y y = x2

y = 1− x2Extra 2. (a) Começamos pelo esboço.x

yy = x3 − x

y = sen(πx)

1−1

Assim a área será, por simetria, o dobro daárea para x ∈ [0, 1], ou seja,2

∫ 1

0(sen(πx)− (x3 − x)) dx =

π + 8

2π=

1

2+

4

π.(b) Começamos pelo esboço.

x

yy = x3 − 3x2 + 2x

y = 3x− x2 − 21−1 2

Assim al ulamos a área somando duas inte-grais:∫ 1

−1(x3 − 3x2 + 2x− (3x− x2 − 2)) dx+

∫ 2

1(3x− x2 − 2− (x3 − 3x2 + 2x)) dx =

8

3+

5

12=

37

12.Extra 3.

f(ab) =

∫ ab

1

dx

x=

∫ a

1

dx

x+

∫ ab

a

dx

x.Tome u = x/a. Assim, du = dx/a. Mudandovariável vamos obter ∫ b

1du/u = f(b).Extra 4. (a) duas parábolas, uma om on avi-dade para baixo, outra para ima. Ambas se in-terse tam, e possuem omo raízes ±√

c.(b) ∫ √c

−√c(c−x2−2x2+2c) dx = 4c3/2 = 25.Logo, c = 4.Extra 5. (a) O volume será dado por

π

∫ +∞

e

log(x)− 1

x2dx = − log x

x

+∞

e

e.(b) O volume será dado por:

∫ e2

1log2 x dx =

= x(log2 x− 2 log x+ 2)∣

e2

1= 2e2 − 2Extra 6. (a) Quando x = 1, y = 1/9. Noteque x =

1/y − 5 na região. Assim, o volumerodando em x será dado porπ

∫ 2

0

dx

(x2 + 5)2.O volume rodando em y é

π

∫ 1/9

0(2)2 dy + π

∫ 1/5

1/9(1/y − 5) dy.(b) A interseção é em (0, 0) e (4, 2). Ambosvolumes serão determinados subtraindo volumes.

84 CAPÍTULO 2. RESPOSTAS DOS EXERCÍCIOSO volume rodando em x será dado porπ

∫ 4

0x dx− π

∫ 4

0

x2

4dx.Invertendo as relações obtemos x = y2 e x = 2y.O volume rodando em y será dado por

π

∫ 2

04y2 dy − π

∫ 2

0y4 dy.( ) Note que embora as raízes de (6−x)2 = xsejam x = 4 e x = 9, a úni a interseção (soluçõesde 6 − x =

√x) é em x = 4, y = 2 (porquedes artei x = 9?). A reta y = 6− x intersepta oeixo x em x = 6O volume rodando em x será dado por

π

∫ 4

0x dx+ π

∫ 6

4(6− x)2dx.Invertendo as relações obtemos que x = 6 − y e

x = y2. O volume rodando em y será dado porπ

∫ 2

0(6− y)2 dy − π

∫ 2

0y4dy.Extra 7.Note que y(x) = ±2

√x. Logo, para ada x, o lado do quadrado é 4√x. A área de ada orte A(x) = 16x. Assim, o volume é

∫ 9

016x dx = 648.Extra 8.De forma análoga a um exer í io anterioronde determinamos o volume de uma esfera omum furo. Por Pitágoras, r2 = a2 + (r − h)2.Rodando a gura em 90 graus, pensando naesfera omo o sólido de revolução de y = f(x) =√

r2 − x2 em torno do eixo x e denindo k =r − h, o volume da alota seráπ

∫ r

k(√

r2 − x2)2 dx =π

3(2r3 − 3kr2 + k3).Substituindo k = r− h, obtemos que o volume é

π(h2r − h3/3). Com mais alguma manipulaçãotambém obtemos que o volume é πh

6(3a2 + h2).Extra 9.O volume total é

π

∫ 2

0(x+ 1) dx = 4π.

O volume até x = a éπ

∫ a

0(x+ 1) dx = π

a2 + 2a

2.Igualando a2 + 2a

2=

1

24, obtemos que a =

√5−

1 (a solução no intervalo [0, 1], pois a outra−√5−

1 está fora.Extra 10.Observe que y(x) = ±b√

1− (x/a)2 om x ∈ [−a, a]. Assim o volume éπ

∫ a

−ab2(1− (x/a)2) dx =

4πab2

3.2.6.4 DesaosDesao 1. Suponha que os ilindros possuem omoeixos os eixos x e y. Agora para ada plano

z = s, que é perpendi ular aos eixos, a inter-seção é um quadrado de lado 2L. Por Pitágoras,r2 = L2 + s2, ou, L(s) = √

r2 − s2. A área de ada quadrado é A(s) = (2L(s))2 = 4(r2 − s2).Assim o volume é∫ r

−r4(r2 − s2) ds =

16

3r3.Pro ure na internet Interse tion of Two Cylinders.Desao 2. Introduzindo oordenadas e olo andoum ír ulo na origem e outro em (0, L), obtemosque

x2 + y2 = R2 e x2 + (y − L)2 = r2.Vamos denotar por (x0, y0) uma solução positivadeste sistema. Suponha, sem perda de generali-dade, que r < R. Geometri amente (faça umagura) é laro que para que o orra uma interse-ção,L− r ≤ R ≤ L+ r.Resolvendo para y0 subtraindo as duas equaçõesobtemos que

y20 − (y0 − L)2 = R2 − r2.Assim,y0 =

R2 − r2 + L2

2L.Como r < R, y0 ≥ 0. Agora podemos denir

x0 =√

R2 − y20.

2.6. APLICAÇÕES DA INTEGRAL 85Vamos al ular a área da lúnula omo a dife-rença entre duas integrais. Para isto denimosyR(x) =

R2 − x2 e yr(x) = L+√

r2 − x2,funções que delimitam a lúnula. Assim a sua áreaé igual a∫ x0

−x0

(yr(x)− yR(x)) dx.Por simetria a área pode ser al ulada por2

∫ x0

0(yr(x)− yR(x)) dx.Agora pre isamos da primitiva de √

a2 − x2:∫

a2 − x2 dx =x

2

a2 − x2+a2

2arcsen(x/a).Apli ando esta fórmula e substituindo√R2 − x20por y0 e √r2 − x20 por |y0 − L| obtemos que aárea é

x0(2L+ |y0 − L| − y0)+

+ r2 arcsen(x0/r)−R2 arcsen(x0/R).Embora formalmente orreta, temos que ve-ri ar que a as funções arcsen estão bem deni-das. Para isto temos que veri ar que (porque?)0 ≤ x0 ≤ R e 0 ≤ x0 ≤ r. Para isto vamosprovar que

y20 ≤ R2, (y0 − L)2 ≤ r2.Isto impli a, pelas equações dos ír ulos, que0 ≤ x0 ≤ R e x0 ≤ r.Vamos provar que y0 ≤ R, o que impli a que

y20 ≤ R2. Cal ulando (veja denição de y0)y0−R =

R2 − r2 + L2 − 2RL

2L=

(R− L)2 − r2

2L.Pela ondição de existên ia de interseção (L−r ≤

R ≤ L+ r), −r ≤ R−L ≤ r. Logo |R−L| ≤ r.Assim, (R − L)2 ≤ r2. Apli ando isto obtemosquey0 −R ≤ r2 − r2

2L= 0,ou seja, y0 ≤ R.Vamos provar que (y0 − L)2 ≤ r2. Bastamostrar que −r ≤ y0 − L ≤ r. Cal ulando (vejadenição de y0)

y0 − L =(R − L)(R+ L)− r2

2L.

Como já provamos que R− L ≤ r,y0 − L ≤ r(R+ L)− r2

2L=

r(R+ L− r)

2L.Como R− r ≤ L,

y0 − L ≤ r.Agora na outra direção. FALTA. . . Prezadoleitor: Me mande o nal desta solução.2.6.5 ⋆Problemas (Substituição Tri-gonométri a)Problema 1. (a) Tome x = sec t. Então dx =sec t tan tdt e √

x2 − 1 = tan t. Assim amos om ∫

sec t dt = log | sec t + tan t|. Ressubsti-tuindo obtemos a resposta.R: log |x+√x2 − 1|.(b) Tome x = sec t. Então dx = sec t tan tdte √

x2 − 1 = tan t. Assim amos om∫

tan2t dt =

(sec2 t− 1) dt = tan t− t. Res-substituindo obtemos a resposta.R: √x2 − 1− arctan(√x2 − 1).2.6.6 ⋆Problemas (Comprimento deCurvas no Plano)Problema 1. (a) √3.(b) Vamos ter que al ular ∫ √

1 + x2

xdx.Fazendo substituição hiperbóli a, obtemos a pri-mitiva √

1 + x2 − arcsenh(1/x).R: √5−√2 + arcsenh(1)− arcsenh(1/2).Problema 2.Ver texto.2.6.7 ⋆Problemas (Área de Superfí- ie de Sólido de Revolução)Problema 1. (a) π

27(103/2 − 1).(b) π

32(18

√5− log(2 +

√5)).( ) π(√2 + log(1 +

√2)).